You are on page 1of 89

PRACTICE TEST 13

Read the following passage and mark the letter A, B, C, or D on your answer sheet to
indicate the correct answer to each of the questions from 1 to 7.
One of the highest honors for formalists, writers, and musical composers is the Pulitzer Prize.
First awarded in 1927, the Pulitzer Prize has been won by Ernest Hemingway, Harper Lee, John F.
Kennedy, and Rodgers and Hammerstein, among others. As with many famous awards, this prize
was named after its founder, Joseph Pulitzer.
Joseph Pulitzer‟s story, like that of many immigrants to the United States, is one of hardship,
hard work and triumph. Born in Hungary, Joseph Pulitzer moved to United States in 1864. He
wanted to be a reporter, but he started his American life by fighting in the American Civil War. After
the war, Pulitzer worked for the German - language newspaper, the Westliche Post. His skills as a
reporter were wonderful, and he soon became a partial owner of the paper.
In 1978, Pulitzer was able to start a newspaper of his own. Right from the first edition, the
newspaper took a controversial approach to new. Pulitzer wanted to appeal to the average reader,
so he produced exciting stories of scandal and intrigue. Such an approach is commonplace today,
but in Pulitzer’s time it was new and different. The approach led to the discovery of many
instances of corruption by influential people. Pulitzer’s paper became very famous and is still
produced today.
The success of Joseph Pulitzer‟s newspaper made him a very wealthy man, so he wanted to give
something back to his profession. Throughout his later years, he worked to establish university
programs for the teaching of journalism, and he funded numerous scholarships to assist journalism
students. Finally, he wanted to leave a legacy that would encourage writers to remember the
importance of quality. On his death, he gave two million dollars to Columbia University so they
could award prizes to great writers.
The Pulitzer Prize recipients are a very select group. For most, winning a Pulitzer Prize is the
highlight of their career. If an author, journalist, or composer you know has won a Pulitzer Prize,
you can be sure they are at the top of their profession.
Question 1. Why does the writer mention "John F. Kennedy" in line 3?
A. He was one of the inventors of the famous awards.
B. He was one of the winners of the Pulitzer Prize.
C. He was one of the people who selected the Pulitzer winners.
D. He was in one of the scandals reported on by Joseph Pulitzer.
Question 2. According to the reading passage, why did Joseph Pulitzer invent the Pulitzer Prize?
A. to encourage people to remember his name and success
B. encourage writers to remember the importance of quality
C. to encourage journalism students to achieve their goals
D. to encourage work of the Pulitzer winners
Question 3. The word “partial” in the passage is closest in meaning to……
A. in part only B. brand new C. one and only D. very important .
Question 4. According the passage, who receives the Pulitzer Prize?
A. Columbia University graduates B.journalism students
C.noted writers and composers D.most newspaper reporters
Question 5. According to the reading passage, how did Joseph Pulitzer appeal to the average reader?
A. He wrote about famous writers of journalism and literature
B. He wrote stories about the war
C. He produced his own newspaper
D. He produced exciting stories of scandal and intrigue
1
Question 6. Which sentence about Joseph Pulitzer is true according to the reading passage?
A. He received a scholarship when he was a university student.
B. He was rich even when he was young
C. He was a reporter during the American Civil War
D. He immigrated to the United States from Hungary
Question 7. Which sentence about Joseph Pulitzer is NOT true according to the reading passage?
A. Joseph Pulitzer was the first writer to the win the prize in 1917.
B. The prize is aimed at encouraging a writer‟s career.
C. Joseph Pulitzer left money to award to the prizewinners.
D. Receiving the prize is one of the highest honors for writers.

Mark the letter A,B,C or D on your answer sheet to indicate the underlined part that
needs correction on each of the followwing questions
Question 8. United Nations is aimed at develop friendly relations among nations based on
A B
respect for the principle of equal rights ans sefl-determination of peoples
C D
Question 9. All members shall give the United Nations every assistance in any action it does
A B C
in accordance with the present Charter.
D
Question 10. Cutural diversity is important because most countries, workplaces, and schools
A
increasingly consist of various culture, racial and ethnic group.
B C D

Mark the letter A, B, C, or D on your answer sheet to indicate the word whose
underlined part differs from the other three in pronunciation in each of the following
questions.
Question 11. A. rhinoceros B. biologist C. reserve D. digest
Question 12. A. form B. shortcoming C. chore D. sector

Mark the letter A, B, C, or D on your answer sheet to indicate the correct answer to
each of the following questions
Question 13. The atmosphere at the meeting was very and everyone was on first name terms
A. formal B. informal C. formality D. informality
Question 14. Mark enjoys having fun by causing trouble. He's very _________ boy.
A. strong willed B. mischievous C. obedient D. well behaved
Question 15. Many young people nowadays are prepared to getting married to pursue their
professional careers.
A satisfy B. sacrifice C. prefer D. confide
Question 16. My boss's planeat 10.15 tomorrow morning, but I cannot pick him up.
A. arrives B. is arriving C. will be arriving D. arrived
Question 17. Jack asked Jil interested in any kinds of sports.
A. if she were B. if were she C. if was she D. if she was
Question 18. Whenever problemswe discuss frankly and find solutions quickly.
A. make up B. come up C. put up D. turn up

2
Question 19. Children should be taught that they have to……….. everything after they use it.
A. put away B. pick off C. collect up D. catch on
Question 20. Vietnamese parents normally do not let their children make a decision their own
future career.
A. in B. of C. on D. for
Question 21. The police have just round the man and his car were swept away during the heavy
storm last week.
A. that B. which C.whose D.when
Question 22. the more terrible the terrorism will become.
A. The more weapons are powerful C. The more powerful weapons are
B. The weapons more powerful D. Weapons are the more powerful
Question 23. We are concerned with the problem of energy resourceswe must also think of our
environment.
A. despite B. though C. however D. but
Question 24. When finding a new house, parents should………….all the conditions for their
children‟s education and entertainment.
A. take into account B. make all the conditions
C. get a measure of D. put into effect

Mark the letter A, B, C or D on your answer sheet to indicate the most suitable response
to complete each of following exchanges.
Question 25. Two friends Diana and Anne are talking about their upcoming exams.
Diana.” Our midterm exams will start next Tuesday, are you ready?”
Anne.” ”
A. I‟m half ready. B. God save you.
C. Thank you so much D. Don’t mention it!
Question 26. Mary is talking to her professor in his office.
Mary.” Can you tell me how to find material for my science report, professor?”
Professor. “_ ”
A. I like it that you understand. B. Try your best, Mary.
C. You can borrow books from the library. D. You mean the podcasts from other students?

Mark the letter A, B, C or D on your answer sheet to indicate the word that differs from
the other three in the position of primary stress in each of the following questions.
Question 27. A. maximum B. vacancy C. terrorist D. investment
Question 28. A. vertical B. contractual C. domestic D. outstanding

Mark the letter A, B, C or D on your answer sheet to indicate the word (s) OPPOSITE in
meaning to the underlined word (s) in each of the following questions.
Question 29. Language teaching in the United State is base on the idea that the goal of language
acquisition is communicative complete.
A. Not good at socializing. C. unable to understand
B. Excellent in orating in front of others D. incapable of working with words
Question 30. This is new washing machine is not a patch on our old one. These clothes are still
dirty.
A. To be expensive B. To be strange C. to be broken D. to be better

3
Mark the letter A, B, C or D on your answer sheet to indicate the word (s) CLOSEST in
meaning to the underlined word (s) in each of the following questions
Question 31. Today, American English is particularly influential, due to the USA’s dominance of
cinema, television, popular music, trade and technology (including the Internet)
A. complete mastery B. overwhelming superiority
C. complete control D. profound effect
Question 32. Students motivation for learning a language increase when they see connection
between what they do in classroom and what they hope to do with the language in the future.
A. the reason for which someone does something
B. the action that someone takes to deal with something
C. the eagerness that someone has to do something
D. the excitement with which one is filled when doing something.

Mark the letter A, B, C or D on your answer sheet to indicate the sentence that is closest
in meaning to each of the following questions
Question 33. “What are you going to do with such a long list of books, Dane?” asked Sarah
A. Sarah was curious why Dane had such a long list of books.
B. Sarah asked Dane what he was going to do with such a long list of books.
C. Sarah could not understand why Dane was borrowing such a long list of books.
D. Sarah warned Dane not to borrow such a long list of books.
Question 34. “Are you going to the cinema with us tonight, Susan?” asks her friends.
A. Susan‟s friend asked her whether she went to the cinema with them that night
B. Susan‟s friend asked her if she was going to the cinema with them that night
C. Susan‟s friend would like to invite her to go to the cinema with them that night
D. Susan‟s friend would rather her went to the cinema with them that night.
Question 35. The Internet has enabled most people to get contact in a matter of moment
A. Most of people have been able to get in contact by the Internet in a matter of moments.
B. Most of people have got in contact as enable in a matter of moments by the Internet
C. On the Internet, most of people are able to get in contact a matter of moments.
D. On the Internet, most of people can find their contact a matter of moments

Read the following passage and mark the letter A, B, C or D on your answer sheet to
indicate the correct word or phrase that best fist each of the numbered blanks
Tim Samaras is a storm chaser. His job is to find tornadoes and follow them. When he gets close to
a tornado, he puts a special tool (36)________ a turtle probe on the ground. This tool measures
things like a twister‟s temperatune, humidity, and wind speed. With this information, Samaras
can lean what causes tornadoes to develop. If meteorologists understand this, they can warn
people (37)________ twisters sooner and save lives.
How does Samaras hunt tornadoes? It‟s not easy. First, he has to find one. Tornadoes are too small
to see using weather satellites. So Samaras can‟t rely on these tools to find a twister. (38)________
, he waits for tornadoes to develop.
Once Samaras sees a tornado, the chase begins. But a tornado is hard to follow. Some
tornadoes change (39) __ several time – for example, moving east and then west and then east
again. When Samaras finally gets near a tornado, het puts the turtle probe on the ground. Being
this close to twister is (40)________ . He must get away quickly.
Question 36. A. called B. know C. made D. meant
Question 37. A. with B. about C. at D. for
4
Question 38. A. Rather B. Still C. Instead D. Yet
Question 39. A. progression B. movement C. dimension D. direction
Question 40. A. terrifly B. terrifying C. terrified D. terrifies

Mark the letter A, B, C, or D to indicate the sentence that best combines each pair of
sentences in the following question.
Question 41. The teacher has done his best to help all students. However, none of them made any
effort on their part
A. The teacher has done his best to help all student, then, none of them made any effort on their part
B. Although the teacher has done his best to help all students, none of them made any effort on their part
C. Because the teacher has done his best to help all students, , none of them made any effort on their part
D. If the teacher has done his best to help all students, , none of them made any effort on their part
Question 42. “ Finish your work. And then you can go home”.
A. “You can‟t go home until you finish your work”
B. “You finish your work to go home as early as you can”
C. “When you go home, finish your work then”
D. “Because you have finished your work, you can go home”.

Read the following passage and mark the letter A, B,C or D on your answer sheet to
indicate the correct answer to each of the questions from 43 to 50
Clara Barton became known as “ The Angel of the Battlefield” during the American Civil
War. Born in Oxford, Massachusetts in 1821, Clara Barton‟s interest in helping soldiers on the
battlefield began when she was told army stories from her father. Another event that influenced
her decision to help soldiers was an accident her brother had. His injuries were cared for by Barton
for 2 year. At the time, she was only 11 years old. Barton began teaching school at the age of 15.
She taught for 18 years before she moved to Washington, D.C in 1854.
The civil war broke out 6 years later. Immediately, Barton started war service by
helping the soldiers with their needs. At the battle of Bull run, Clara Barton received permission
from the government to take care of the sick and hurt. Barton did this with great empathy and
kindness. She acknowledged each soldier as a person . her endurance and courage on the
battlefield were admired by many. When the war ended in 1865 , she used 4 years of her life to
assist the government in searching for soldiers who were missing during the war.
The search for missing soldiers and years of hard work made her feeble physically. In
1869, her doctors recommended a trip to Europe for a rest. While she was on vacation , she
became involved with the International Red Cross, an organization set up by the Geneva
Convention in 1864. Clara Barton realized that the Red Cross would be a best help to the United
States. After she returned to the United States, she worked very hard to create an American Red
Cross. She talked to government leaders and let American people know about the Red Cross. In
1881, the National Society of the Red Cross was finally established with its headquarters in
Washington, D.C. Clara Barton managed its activities for 23 years.
Barton never let her age stop her from helping people. At the age of 79, she helped food
victims in Galveston, Texas. Barton finally resigned from the Red Cross in 1904. She was 92 years
old and had truly earned her title “The Angel of the Battle”.
Question 43. According to the paragraph 1, which of the following is true of the young Barton
Clara?
A. She helpep her father when he was a soldier
B. She suffered from an accident when she was 11
5
C. She helped her brother who hurt in an accident
D. She made a decision to live with her brother for 2 years
Question 44. The phrase broke out in paragraph 2 is closest in meaning to
A. extended B. broken down C. closed D. began
Question 45. The word this in paragraph 2 refers to
A. recognized each soldier as a person B. cooker for soldiers
C. took care of the sick and hurt D. received permission
Question 46. The word acknowledged in paragraph 2 could best be replaced by
A. nursed B. recognized C. pleaded D. believed
Question 47. What can be inferred about the government?
A. It did not always agree with Clara barton
B. It did not have the money to help Clara Barton
C. It showed Clara Barton great empathy and kindness
D. It had respect for Clara Barton
Question 48. What does the author mention about the American Red Cross?
A. It was disapproved again and again by the Geneva Convention
B. Barton tried to have it set up in America
C. The American people were not interested in the Red Cross
D. It was first established in the Unites States
Question 49. What is the main idea of the passage?
A. Clara Barton helped wounded soldiers and she was the founder of the Red Cross
B. Clara Barton was a kind and strong woman who helped people in need
C. Clara Barton becam a nurse during the American Civil War
D. Clara Barton worked for disaster victims until she was old
Question 50. What can be the best title of the reading passage?
A. The angel of the Battlefield C. The American Civil War
B. The American Red Cross D. The International Red Cross
~~THE END~~

PRACTICE TEST 14
Mark the letter A, B, C, or D on your answer sheet to indicate the word whose
underlined part differs from the other three in pronunciation in each of the following
questions.
Question 1. A. chemistry B. chocolate C. speech D. lunch
Question 2. A. book B. foot C. good D. boot

Mark the letter A, B, C, or D on your answer sheet to indicate the word that differs from
the other three in the position of primary stress in each of the following questions.
Question 3. A. calculation B. economics C. photography D. conservation
Question 4. A. industry B. adventure C. consider D. eventual

Mark the letter A, B, C, or D on your answer sheet to indicate the underlined part that
needs correction in each of the following questions.
Question 5. Bill is often late for class, which makes his teachers angrily.
A. which B. angrily C. for D. is
Question 6. In the end of the party, Mary had to wash the dishes alone again, as usual .
6
A. as usual B. wash C. In D. alone again
Question 7. The oceans,which cover two-thirds of earth’s surface , are the object of study for
oceanographers.
A. The oceans B. of earth’s surface C. object D. for
Mark the letter A, B, C, or D on your answer sheet to indicate the correct answer to each
of the following questions.
Question 8. He ______ for two weeks. He is trying to give it up.
A. isn’t smoking B. hasn’t smoked C. doesn’t smoke D. didn’t smoked
Question 9. He is a _______ boy. He is often kind and helpful to every classmate.
A. lovely B. obedient C. frank D. caring
Question 10. Many of pictures ______ from outer space are presently on display in the public
library.
A. that sent B. sending C. sent D. to sending
Question 11. Do you have ______ minutes? I’d like to ask you some questions.
A. a little B. few C. little D. a few
Question 12. Instead of buying a new pair of shoes, I had my old ones ______.
A. repaired B. repairing C. to repair D. repair
Question 13. Don’t forget ______ me as soon as you arrive at the airport.
A. called B. call C. calling D. to call
Question 14. How ______ homework did the teacher assign?
A. much B. plenty C. little D. many
Question 15. Peter has been studying for almost three years and he will have his degree and
return to his country in ______ six months.
A. others B. another C. other D. the other
Question 16. The director ______ the men to turn to work immediately.
A. ordered B. insisted C. demanded D. suggested
Question 17. __ number of boys were swimming in the lake, but I didn’t know _____ exact
number of them.
A. A/the B. A/an C. The/the D. The/an
Question 18. During his _____, he lived with his uncle’s family in the United States.
A. childish B. child C. childlike D. childhood
Question 19. I like her, ______ she can be very annoying at times.
A. however B. therefore C. even though D. despite

Mark the letter A, B, C, or D on your answer sheet to indicate the most suitable
response to complete each of the following exchanges.
Question 20. Tom: “Hello, nice to see you again.” Jerry: “______.”
A. Me too B. I don’t think you are C. Neither am I D. You shouldn’t be so
Question 21. Mike: “Would you like a cup of coffee?” Ann: “______.”
A. Yes, please B. No, I don’t C. Yes, I do D. No, I don’t like

Mark the letter A, B, C, or D on your answer sheet to indicate the word(s) CLOSEST in
meaning to the underlined word(s) in each of the following questions.
Question 22. The old houses were knocked down to make way for a new block of flats.
A. pulled apart B. struck through C. demolished D. abolished
Question 23. After Mary finishes her degree, she intends to work in her father's company.
A. an institution for business studies B. a university or college course
7
C. an amount or level of something D. a unit for measuring angles
Mark the letter A, B, C, or D on your answer sheet to indicate the word(s) OPPOSITE in
meaning to the underlined word(s) in each of the following questions.
Question 24. Simon admitted that his joke on his girlfriend’s clothes was deliberate.
A. planned B. unintentional C. unknown D. unwanted
Question 25. The boy wasn't allowed to have any friends, so he became an introvert.
A. someone who is active and confident, and who enjoys spending time with other people
B. someone who is quiet and shy, and does not enjoy being with other people
C. someone who believes that good things will never happen to him and other people
D. someone who wears funny clothes and does silly things to make people laugh

Mark the letter A, B, C, or D on your answer sheet to indicate the sentence that is
closest in meaning to each of the following questions.
Question 26. The garage serviced my car last week.
A. I got my car serviced last week. B. My car was to be serviced last week.
C. The garage’s service to my car was good. D. Last week I went to the garage by car.
Question 27. The train drivers' strike made it hard for us to get to work.
A. We found it difficult to get to work because of the train drivers' strike.
B. When the train drivers were having a strike, we could get to work.
C. We were unable to get to work when the train drivers' strike happened.
D. The train drivers' strike made it impossible for us to get to work.
Question 28. The train should be here any minute now.
A. We know that the train is always here on time. B. The train has not arrived here yet.
C. The train is known to be here in a minute. D. We are expecting the train to arrive soon.

Mark the letter A, B, C, or D on your answer sheet to indicate the sentence that best
combines each pair of sentences in the following questions.
Question 29. He is very intelligent. He can solve all the problems in no time.
A. So intelligent is he that he can solve all the problems in no time.
B. He is very intelligent that he can solve all the problems in no time.
C. An intelligent student is he that he can solve all the problems in no time.
D. So intelligent a student is he that he can solve all the problems in no time.
Question 30. We cut down many forests. The Earth becomes hot.
A. The more forests we cut down, the hotter the Earth becomes.
B. The more we cut down forests, the hotter the Earth becomes.
C. The more forests we cut down, the Earth becomes hotter.
D. The more we cut down forests, the Earth becomes hotter.

Read the following passage and mark the letter A, B, C, or D on your answer sheet to
indicate the correct word or phrase that best fits each of the numbered blanks
By the 1950s many American families owned television sets. During television's first 20 years,
deaf people missed most of the fun. They could not hear what was being said and had to
(31)_____.
Deaf people who watched television liked sports and action shows, but they were disappointed
with other programs. If there was a lot of (32)_____, deaf viewers couldn't follow the plot. Even
the most skilled lip readers could only catch part of the talking. This frustrated many deaf people.

8
In the late 1960s, a man started experimenting. Malcom Norwood thought that deaf people
could enjoy television programs, too. He wanted to develop captions for the programs. Norwood
worked for the federal government's Media Services and Captioned Films Division at the Bureau of
Education of the Handicapped. Norwood surveyed many hearing Americans. He wanted to see how
they felt about seeing captions on the television screen. Too many people were against the
(33)_____. Norwood realized he had to develop another way of captioning - one that would not
bother hearing people.
In October of 1971, Norwood's (34)_____ signed a contract with WGBH-TV, a public television
station in Boston. WGBH was hired to experiment with captions. They agreed to make a captioned
television program for Norwood. That program was made. It was shown on television and at a
special convention. The type of captions made by WGBH could be seen on any television. No special
equipment was needed. These were called "open captions."
Later, a new machine was invented. This device was made to send signals on a special part of
the television picture. The signals could be captions. If a family had another kind of machine in
their home or in their TV set, then the captions (or signals) would appear on their television screen.
Without the machine, no captions would be seen. That special machine is called a decoder. It
receives the signals (35)_____from the television station. Captions that require a decoder are called
"closed captions".
Question 31. A. forecast B. find C. guess D. wait
Question 32. A. scene B. dialogue C. talk D. conversation
Question 33. A. law B. questions C. actors D. idea
Question 34. A. students B. channels C. company D. office
Question 35. A. moved B. gone C. transmitted D. sent

Read the following passage and mark the letter A, B, C, or D on your answer sheet to
indicate the correct answer to each of the questions from 36 to 42.
Mandatory volunteering made many members of Maryland's high school class of 97
grumble with indignation. They didn't like a new requirement that made them take part in the
school's community service program.
Future seniors, however, probably won't be as resistant now that the program has
broken in. Some, like John Maloney, already have completed their required hours of approved
community service. The Bowie High School sophomore earned his hours in eighth grade by
volunteering two nights a week at the Larkin-Chase Nursing and Restorative Center in Bowie.
He played shuffle board, cards, and other games with the senior citizens. He also helped plan
parties for them and visited their rooms to keep them company.
John, fifteen, is not finished volunteering. Once a week he videotapes animals at the
Prince George's County animal shelter in Forestville. His footage is shown on the Bowie public
access television channel in hopes of finding homes for the animals.
"Volunteering is better than just sitting around," says John, "and I like animals; I don't
want to see them put to sleep."
He's not the only volunteer in his family. His sister, Melissa, an eighth grader, has
completed her hours also volunteering at Larkin-Chase.
"It is a good idea to have kids go out into the community, but it's frustrating to have to
write essays about the works," she said. It makes you feel like you're doing it for the requirement
and not for yourself."
The high school's service learning office, run by Beth Ansley, provides information on organizations
seeking volunteers so that students will have an easier time fulfilling their hours.
9
"It's ridiculous that people are opposing the requirements," said Amy Rouse, who this
summer has worked at the Ronald McDonald House and has helped to rebuild a church in Clinton.
"So many people won't do the service unless it's mandatory," Rouse said, "but once they
start doing it, they'll really like it and hopefully it will become a part of their lives - like it has
become a part of mine."
Question 36. The best title of the passage could be ______.
A. "Students Who Volunteer to Work with Senior Citizens"
B. "Students Who Earn Extra Money after School"
C. "A Volunteer Program at Bowie High School"
D. "The High School Class of 1977"
Question 37. The word "frustrating" in paragraph 7 is closest in meaning to ______.
A. interesting B. happy C. satisfying D. upset
Question 38. The word "it" in paragraph 7 refers to ______.
A. completing requirements B. writing essays
C. doing volunteer work D. A. going out in the community
Question 39. From paragraphs 6 and 7, we can infer that Melissa Maloney ______.
A. volunteers because it's a requirement
B. is frustrated by her volunteer job
C. volunteers because it makes her feel good
D. doesn't like to write essays about her volunteer work
Question 40. According to the last two paragraphs, Amy Rouse thinks that ______.
A. most people don't like volunteering, so they don't want to do it
B. most people will discover they enjoy volunteering if they try it
C. the volunteer program shouldn't be mandatory
D. people should be able to choose whether they want to volunteer
Question 41. Which of the following volunteer activities is NOT mentioned in the passage?
A. Videotaping animals in a shelter. B. Rebuilding a church.
C. Tutoring children. D. Visiting elderly people.
Question 42. In the passage, the author gives the explanation of the concept of mandatory
volunteer programs by ______.
A. describing one volunteer program B. classifying different types of volunteer programs
C. arguing in favor of volunteer programs D. comparing two volunteer programs

Read the following passage and mark the letter A, B, C, or D on your answer sheet to
indicate the correct answer to each of the questions from 43 to 50.
The mineral particles found in soil range in size from microscopic clay particles to large boulders.
The most abundant particles - sand, silt, and clay - are the focus of examination in studies of soil
texture. Texture is the term used to describe the line composite sizes of particles in a soil sample,
typically several representative handfuls . To measure soil texture, the sand, silt, and clay
particles are sorted out by size and weight. The weights of each size are then expressed as a
percentage of the sample weight.
In the field, soil texture can be estimated by extracting a handful of sod and squeezing the damp
soil into three basic shapes: (1) cast, a lump formed by squeezing a sample in a clenched fist; (2)
thread, a pencil shape formed by rolling soil between the palms; and (3) ribbon, a flatfish shape
formed by squeezing a small sample between the thumb and index finger. The behavioral
characteristics of the soil when molded into each of these shapes, if they can be formed at all,
provides the basis for a general textural classification. The behavior of the soil in the hand test is
10
determined by the amount of clay in the sample. Clay particles are highly cohesive, and when
dampened , behave as a plastic. Therefore the higher the clay content in a sample, the more
refined and durable the shapes into which it can be molded.
Another method of determining soil texture involves the use of devices called sediment sieves,
screens built with a specified mesh size. When the soil is filtered through a group of sieves, each
with a different mesh size, the particles become grouped in corresponding size categories. Each
category can be weighed to make a textural determination. Although sieves work well for silt, sand,
and larger particles, they are not appropriate for clay particles. Clay is far too small to sieve
accurately; therefore, in soils with a high proportion of clay, the fine particles are measured on the
basis of their settling velocity when suspended in water. Since clays settle so slowly, they are easily
segregated from sand and silt. The water can be drawn off and evaporated, leaving a residue of
clay, which can be weighed.
Question 43. The author mentions “several representative handfuls” in the passage in order
to show ______.
A. the range of soil samples B. the requirements for an adequate soil sam
C. the process by which soil is weighed D. how small soil particles are
Question 44. It can be inferred that the names of the three basic shapes mentioned in paragraph
2 reflect ______.
A. the way the soil is extracted B. the need to check more than one handful
C. the difficulty of forming different shapes D. the results of squeezing the soil
Question 45. The word “dampened” in the passage is closest in meaning to _____.
A. damaged B. stretched C. moistened D. examined
Question 46. It can be inferred from the passage that a soil sample with little or no clay in it
______.
A. does not have a durable shape B. is not very heavy
C. does not have a classifiable texture D. may not hold its shape when molded
Question 47. The word “they” in the passage refers to ______.
A. larger particles B. sieves C. categories D. clay particles
Question 48. It can be inferred from the passage that the sediment sieve has an advantage over
the hand test in determining soil texture because ______.
A. the sieve allows for a more exact measure B. less training is required to use the sieve
C. using the sieve takes less time D. the sieve can measure clay
Question 49. During the procedure described in paragraph 3, when clay particles are placed into
water they ______.
A. stick to the sides of the water container B. dissolve quickly
C. take some time to sink to the bottom D. separate into different sizes
Question 50. The word “fine” in the passage is closest in meaning to ______.
A. many B. excellent C. tiny D. various
~~ THE END ~~

PRACTICE TEST 15
Mark the letter A, B, C, or D on your answer sheet to indicate the word whose
underlined part differs from the other three in pronunciation in each of the following
questions.
Question 1. A. sender B. complete C. penalty D. bedroom
Question 2. A. reformed B. sacrificed C. dissolved D. ploughed
11
Mark the letter A, B, C, or D on your answer sheet to indicate the word that differs from
the other three in the position of primary stress in each of the following questions
Question 3. A. mathematics B. history C. literature D. physics
Question 4. A. economy B. village C. interest D. industry

Mark the letter A, B, C, or D on your answer sheet to indicate the underlined part that
needs correction in each of the following questions
Question 5. Supposed that (A) you failed (B) your driving test, would (C) you take (D) it again?
Question 6. I often look into (A) new words in (B) the dictionary whenever (C) I don’t know
their meanings (D)
Question 7. When I came (A) to the theatre, I saw a handsome American young (B) man
standing (C) on the stage (D)

Mark the letter A, B, C, or D on your answer sheet to indicate the correct answer to
each of the following questions
Question 8. The harder you work, you‟ll pass the exam.
A. the best B. the worst C. the better D. the worse
Question 9. Our school about 600 new students every year.
A. admits B. allows C. accepts D. gets
Question 10. There a number of reasons for the fall of the Roman Empire.
A. are said to have been B. said to be
C. are said being D. was said being
Question 11. Let’s go to the beach - it’s a day to sit indoors.
A. nice enough B. too nice C. so nice D. such nine
Question 12. I don‟t like when I am not there.
A. criticizing B. being criticized C. to criticize D. to be criticized
Question 13. If I __ in her position, I could turn down the job.
A. am B. had been C. will be D. were
Question 14. I thought you said she was going away the next Sunday, ?
A. wasn't it B. didn't you C. wasn't she D. didn't I
Question 15. I my hand to draw her attention but she took no ______ of me.
A. waved/ notice B. shook/ noticed C. waved/ attention D. shook/ attention
Question 16. These days everybody is aware _________the danger of smoking.
A. up B. of C. on D. with
Question 17. I can’t speak English,
A. neither can sheB. she can’t either C. she can’t too D. A & B are correct
Question 18. Gestures such as waving and handshaking are forms of ___communication.
A. direct B. regular C. verbal D. non-verbal
Question 19. When Mr.Kim needs a car, he uses
A. his son’s B. that one of his son C. car of his son D. he son’s own car

Mark the letter A, B, C, or D on your answer sheet to indicate the most suitable
response to complete each of the following exchanges
Question 20. Jane: Thank you for a lovely evening. Barbara:
A. You are welcome B. Have a good day C. Thanks D. Cheer!
Question 21. A: _______ B: Oh, thank you. I just got it yesterday.
A. When have you got this beautiful dress?
12
B. You’ve just bought this beautiful dress, haven’t you?
C. How a beautiful dress you’re wearing!
D. That’s a beautiful dress you have on!

Mark the letter A, B, C, or D on your answer sheet to indicate the word(s) CLOSEST in
meaning to the underlined word(s) in each of the following questions
Question 22. Tourists today flock to see the two falls that actually constitute Niagara falls.
A. come by plane B. come in large numbers
C. come out of boredom D. come without knowing what they will see
Question 23. A lack of qualifications can be a major obstacle to finding a job.
A.impediment B. encouragement C. impetus D. assistance

Mark the letter A, B, C, or D on your answer sheet to indicate the word(s) OPPOSITE in
meaning to the underlined word(s) in each of the following questions
Question 24. Children must sit on an parent’s lap unless an empty seat is available.
A. unoccupied B. booked C. disused D. abandoned
Question 25. Smoking has been banned in public places in some countries.
A. made illegal B. limited C. restricted D. given way

Mark the letter A, B, C, or D on your answer sheet to indicate the sentence that is
closest in meaning to each of the following questions.
Question 26. "Shall I make you a coffee?" the girl said to the man.
A. The girl wanted to make a coffee for the man.
B. The girl was asked to make a coffee for the man.
C. The girl promised to make a coffee for the man.
D. The girl offered to make a coffee for the man
Question 27. “I was not there at the time,” he said.
A. He denied to have been there at the time.
B. He denied that he had not been there at the time.
C. He denied to be there at the time.
D. He denied having been there at the time.
Question 28. No matter how hard he tried, Mike could not make sense of his economics textbook.
A. Mike could not understand his economics textbook because he hardly tried to at all.
B. In spite of his efforts, Mike was unable to understand the contents of his economics textbook.
C. If Mike had studied harder, he would have been able to comprehend what was in his economics
textbook.
D. It was impossible for Mike to understand his economics textbook without making a great effort.

Mark the letter A, B, C, or D on your answer sheet to indicate the sentence that best
combines each pair of sentences in the following questions.
Question 29. Mary was sick. She didn’t leave the meeting until it ended.
A. In spite of the fact that Mary’s sickness, she didn’t leave the meeting until it ended.
B. Mary didn’t leave the meeting until it ended despite she was sick.
C. When the meeting ended, Mary left because she was sick.
D. Though sick, Mary didn’t leave the meeting until it ended.
Question 30. She heard the news of the death of her mother. She fainted.
A. On hearing the news of her dead mother, she fainted.
13
B. She fainted and then she heard the news of the death of her mother.
C. She fainted so she heard the news of the death of her mother.
D. On hearing the news of her mother’s death, she fainted.

Read the following passage and mark the letter A, B, C, or D on your answer sheet to
indicate the correct word or phrase that best fits each of the numbered blanks
However objective we believe ourselves to be, most of us do not judge a product solely on
itsmerits, considering quality, value and style before making a decision. (31) _________, we are
easily influenced by the people around us.
There is nothing (32) _______ with this. It is probably a smarter way to
make decisions than (33) _____ on only our own opinions. But it does make life hard for
companies. They have long understood that groups of friends and relatives tend to buy the same
products, but understanding the reasons has been tricky. It is because they are so similar with (34)
________ to how much money they make and what television ads they watch that they
independently (35) _______ at the same decision
Question 31. A. What’s more B. Instead C. Unlike D. In place
Question 32. A. wrong B. silly C. bad D. daft
Question 33. A. basing B. trusting C. supposing D. relying
Question 34. A. connection B. regard C. relation D. concern
Question 35. A. reach B. come C. arrive D. get
Read the following passage and mark the letter A, B, C, or D on your answer sheet to
indicate the correct answer to each of the questions from 36 to 42.
PHOBIAS CONTROL THE EYES
People with an overpowering fear of something may not be able to consciously control the
direction of their gaze when faced with a picture that provokes that fear. Those with a fear of
spiders, or arachnophobia, may not be able to avoid looking toward a spider in the grass, while
those without the fear can. To reach this finding, 26 college students - half of whom had
arachnophobia - were outfitted with a device that tracks eye movements. The participants were
then shown a four-by-four grid of flowers on a computer screen, where a picture of a grey-brown
spider and a grey-brown mushroom each occupied one space on the grid.
In one trial, the students were asked to press the spacebar when they spotted a mushroom,
but to ignore any spiders. In a second, the students were asked to press the key when they saw
the spider, but to avoid looking at mushrooms. When instructed to ignore the spider and spot the
mushroom, the students with arachnophobia could not avoid looking at the spider - causing them
to notice the mushroom three-tenths of a second more slowly than the control group. Additionally,
when the students were asked to search for the spider, the arachnophobic students found the
spider and pressed the space bar about three-tenths of a second more quickly than the non-phobic
students.
These findings show that the attention processes of people with a powerful fear of spiders may
be beyond their conscious control. This suggests that treatments that focus on conscious thought
processes may not work for such phobias. Instead, therapists could focus on treatments such as
systematic desensitization.
Question 36. What can we infer that the word phobia means?
A. a new type of spider B. a fear of something
C. a scientific experiment D. the name of a flower
Question 37. Why was the device to track eye movement needed?
A. to make sure the students were looking at the computer screen
14
B. to move the spider’s location around
C. to verify where the students were looking
D. to make the students look only in one place
Question 38. What do the results of the experiment tell us about people with phobias?
A. They should always avoid the object of their fear.
B. They don’t have to try to change their thoughts.
C. They can’t consciously choose how to react to the feared object.
D. All of the above
Question 39. It can be inferred that provokes in paragraph I is closest in meaning to
A. eases B. treats C. reminds D. causes
Question 40. It can be inferred that spotted in paragraph 2 is closest in meaning to
A. ignored B. feared C. saw D. instructed
Question 41. The word their in paragraph 3 refers to .
A. findings B. processes C. spiders D. none of the above
Question 42. What is the purpose of the passage?
A. to prove that phobias are a myth
B. to help people with phobias
C. to relate research findings about phobias affecting eye control
D. to relate research findings about phobias controlling eyesight.

Read the following passage and mark the letter A, B, C, or D on your answer sheet to
indicate the correct answer to each of the questions from 43 to 50
One of the seven wonders of the ancient world, the Great Pyramid of Giza was a
monument of wisdom and prophecy built as a tomb for Pharaoh Cheops in 2720 B.C . Despite its
antiquity, certain aspects of its construction makes it one of the truly wonders of the world. The
thirteen- acre structure near the Nile river is a solid mass of stone blocks covered with limestone.
Inside are the number of hidden passageways and the burial chamber of the Pharaoh. It is the
largest single structure in the world. The four sides of the pyramid are aligned almost exactly on
true north, south, east and west-an incredible engineering feat. The ancient Egyptians were sun
worshippers and great astronomers, so computations for the Great Pyramid were based on
astronomical observations.
Explorations and detailed examinations of the base of the structure reveal many
intersecting lines. Further scientific study indicates that these represent a type of timeline of events
– past, present and future. Many of the events have been interpreted and found to coincide with
known facts of the past. Others are prophesied for future generations and are currently under
investigation. Many believe that pyramids have supernatural powers and this one is no exception.
Some researchers even associate it with extraterrestrial beings of ancient past. Was this
superstructure made by ordinary beings, or one built by a race far superior to any known today?
Question 43. On what did the ancient Egyptians base their calculations?
A. Observation of the celestial bodies B. Advanced technology
C. Advanced tools of measurement D. knowledge of the earth‟s surface
Question 44. What has research of the base revealed?
A. there are cracks in the foundation B. The lines represent important events
C. Tomb robbers have stolen the Pharaoh’s body D. A superior race of people built in
Question 45. Extraterrestrial beings are .
A. living beings from other planets B. astronomers in the ancient times
C. researchers in Egyptology D. very strong workers
15
Question 46. What was the most probable reason for providing so many hidden passages ?
A. To allow the weight of the pyramid to settle evenly
B. To permit the high priests to pray at night
C. To enable the Pharaoh‟s family to bring food for his journey to the afterlife
D. To keep grave robbers from finding the tomb and the treasure buried with the pharaoh
Question 47. In the second passage , the word “prophesied‟ is closest in meaning to
A. affiliated B. precipitated C. terminated D. foretold
Question 48. Why is the Great Pyramid of Giza considered one of the seven wonders of the world?
A. It is perfectly aligned with the four cardinal points of the compass and contains many
prophecies
B. It was selected of the tomb of Pharaoh Cheops
C. It was built by a super race
D. It is very old
Question 49. The word feat in the first paragraph is closet in meaning to .
A. accomplishment B. Appendage C. festivity D. structure
Question 50. What do the intersecting lines in the base symbolize?
A. Architects’ plan for the hidden passages
B. Pathways of the great solar bodies
C. Astrological computations
D. Dates of important events taking place throughout time

~~THE END~~

PRACTICE TEST 16

16
17
18
19
PRACTICE TEST 17

Mark the letter to indicate the word whose underlined part differs from the other three
in pronunciation .
Question 1: A. stopped B. cooked C. laughed D. started
Question 2. A. allow B. tomorrow C. slowly D. below

Mark the letter to indicate the word that differs from the other three in the position of
the primary stress
Question 3: A. capture B. picture C. ensure D. pleasure
Question 4: A. business B. opponent C. specialize D. agency

Mark the letter A, B, C, or D on your answer sheet to indicate the correct answer to
each of the following questions.
Question 5: College students are becoming less dependent_____________ their teachers.
A. on B. with C. of D. to
Question 6: She is the woman about_____________ I told you.
A. which B. whose C. whom D. that
Question 7: How long ago_____________ to learn French?
A. would you start B. did you start C. were you starting D. have you started
Question 8: _____________ he was tired, he still watched the final match on TV.
A. Though B. However C. Despite D. Because
Question 9: Nowadays, most students use_______ calculators in their studies and exams.
A. electrical B. electronic C. electricity D. electric
Question 10: It__________ it is parents' responsibility to take good care of their children.
A. commonly says that B. is commonly saying
C. commonly to be said that D. is commonly said that
Question 11: Neither Tom nor his brothers ____ willing to help their mother with the housework.
A. was B. are C. has been D. is
Question 12: ______________ that she does not want to stay any longer.
A. So homesick does Beth feel B. Beth feels such homesick
C. Homesick though Beth may feel D. A little homesick does Beth feel
Question 13. _____________ harder, you would have passed the exam.

A. If you studied B. If had you studied


C. Had you studied D. Were you studied
Question 14. A house in London costs ______ a similar house in Liverpool.
A. twice as much as B. twice as much more
C. as much as twice D. twice as many as
Question 15 “John shouldn’t have behaved so badly.”, said Janet.
A. Janet doesn't like John's behaviors. B. Janet was angry with John.
C. Janet disliked John. D. Janet objected to John's behaviors.
Question 16: I hope you will_____________ notice of what I am going to tell you.
A. gain B. keep C. get D. take

20
Choose A, B, C, or D on your answer sheet to indicate the word or phrase CLOSEST in
meaning .
Question 17: Don't be concerned about your mother's illness; she'll recover soon.
A. surprised at B. worried about C. embarrassed at D. angry with
Question 18: Although they hold similar political views, their religious beliefs present a striking
contrast.
A. significant difference B. interesting resemblance
C. minor comparison D. complete coincidence

Mark the letter A, B, C, or D on your answer sheet to indicate the word or phrase that is
OPPOSITE in meaning
Question 19: This shouldn’t be too taxing for you.
A. comfortable B. demanding C. easy D. relaxing
Question 20: The Germany’s war hysteria has accounted for its people’s hostility towards foreigners.
A. disease B. ceremony C. malaria D. serenity

Mark the letter A, B, C, or D on your answer sheet to indicate the most suitable respond
to complete each of the following exchanges
Question 21: Lucy: “You look really nice in that red sweater!” Sue: “___________”
A. Don’t mention it. B. How dare you? C. I’m afraid so. D. Thank you.
Question 22 : - Jack: "What's wrong with you?" - Jill: "_______________."
A. I'm having a slight headache B. Thank you very much
C. Yes, I was tired yesterday D. No, I don't care

Mark the letter A, B, C, or D on your answer sheet to show the underlined part that
needs correction .
Question 23: The assumption that smoking has bad effects on our health have been proved.
A B C D
Question 24: Rock music was original a mixture of country music and rhythm and blues.
A B C D
Question 25: Wind is the motion that occurs when lightest air rises and cooler heavier air replaces it.
A B C D
Mark the letter A, B, C, or D on your answer sheet to indicate the sentence that is
closest in meaning .
Question 26. Their chances of success are small.
A. They will certainly be successful. B. It’s possible that they will achieve success.
C. They have no chances of being successful. D. It’s not very likely that they will succeed.
Question 27. Kate works for an organization which collects money to help orphans .
A. The organization which Kate works for collects money to help orphans.
B. The organization where Kate works for collects money to help orphans.
C. The organization for that Kate works collects money to help orphans.
D. Money of orphans is collected in the organization where Kate works.
Question 28. The Minister said that he had not done anything improper.
A. The Minister denied doing anything improper.
B. The Minister denied that he would do anything improper.
C. The Minister refused to have done anything improper.
21
D. The Minister refused to do anything improper.

Mark the letter A, B, C, or D on your answer sheet to indicate the sentence that best
combines each pair of sentences in the following questions
Question 29: We’d better leave them a note. It’s possible they’ll arrive later .
A. If they arrive late, we’d better leave them a note.
B. We’d better leave them a note as they possibly arrive later.
C. They’ll probably arrive later so that we’d better leave them a note.
D. We’d better leave them a note in case they arrive later.
Question 30: Women still cover their heads in some countries. They did so in the past .
A. In the past, women cover their heads but they do so today in some countries.
B. Women still cover their heads in some countries as they did in th past.
C. Women still cover their heads in some countries similar to what they did so in the past.
D. Women still cover their heads in some countries as they did so in the past.

Read the following passage adapted and choose the correct answer (corresponding to A, B,
C, or D) to each of the questions that follow.
Today we take electricity for granted and perhaps we do not realize just how useful this
discovery has been. Steam was the first invention that replaced wind power. It was used to drive
engines and was passed through pipes and radiators to warm rooms. Petrol mixed with air was the
next invention that provided power. Exploded in a cylinder, it drove a motor engine. Beyond these
simple and direct uses, those forms have not much adaptability. On the other hand, we make use
of electricity in thousands of ways. From the powerful voltages that drive our electric trains to the
tiny current needed to work a simple calculator, and from the huge electric magnet in steel works
that can lift 10 tons to the tiny electric magnet in a doorbell, all are powered by electricity. An
electric current can be made with equal ease to heat a huge mass of molten metal in a furnace or
to boil a jug for a cup of coffee. Other than atomic energy, which has not as yet been harnessed to
the full, electricity is the greatest power in the world. It is flexible, and so adaptable for any task for
which it is wanted. It travels so easily and with incredible speed along wires or conductors that it
can be supplied instantly over vast distances. To generate electricity, huge turbines or generators
must be turned. In Australia they use coal or water to drive this machinery. When dams are built,
falling water is used to drive the turbines without polluting the atmosphere with smoke from coal.
Atomic power is used in several countries but there is always the fear of an accident. A tragedy
once occurred at Chernobyl, in Ukraine, at an atomic power plant used to make electricity. The
reactor leaked, which caused many deaths through radiation. Now scientists are examining new
ways of creating electricity without harmful effects to the environment. They may harness the tides
as they flow in and out of bays. Most importantly, they hope to trap sunlight more efficiently. We
do use solar heaters for swimming pools but as yet improvement in the capacity of the solar cells to
create more current is necessary. When this happens, electric cars will be viable and the world will
rid itself of the toxic gases given off by trucks and cars that burn fossil fuels.
Question 31: The author mentions the sources of energy such as wind, steam, petrol in the first
paragraph to____.
A. suggest that electricity should be alternated with safer sources of energy
B. discuss which source of energy can be a suitable alternative to electricity
C. emphasize the usefulness and adaptability of electricity
D. imply that electricity is not the only useful source of energy

22
Question 32: Before electricity, what was sometimes passed through pipes to heat rooms?
A. Hot wind. B. Steam. C. Gas. D. Petrol.
Question 33: What does the author mean by saying that electricity is flexible?
A. It can be adapted to various uses. B. It is cheap and easy to use.
C. It is used to drive motor engines. D. It can be made with ease.
Question 34: The main forms of power used to generate electricity in Australia are___________.
A. atomic power and water B. water and coal
C. sunlight and wind power D. wind and gas
Question 35: The word "they" in the last paragraph refers to _________________.
A. new ways B. harmful effects C. scientists D. the tides
Question 36: Electric magnets are used in steel works to _____________________.
A. boil a jug of water B. test the steel for strength
C. lift heavy weights up to ten tons D. heat the molten steel
Question 37: The best title for this passage could be___________.
A. “Why Electricity Is So Remarkable” B. “Types of Power Plants”
C. “Electricity: Harmful Effects on Our Life” D. “How to Produce Electricity”

Choose the word or phrase ( A, B, C or D ) that best fits the blank space in the following
passage.
When you first apply for a job, you (38) ______not succeed in getting it. It’s always a good
(39)______to ask them to explain to you what prevented from beating the other candidates. Don’t
complain about the situation, but ask them to advise you (40)______what you can do better next
time. Perhaps the interviewer disapproved of or disagreed with something you said. Perhaps they
just glanced at your application and saw something that made it easy to choose between you and
another candidate. Don’t regard it as a failure, but recognize it as a chance to learn more.
(41)______you don’t worry too much about it and continue to believe in yourself, you’ll
(42)______find the chance you’ve been waiting for. Then, your family and friends will be able to
congratulate you on your success!
Question 38: A. might B. would C. won’t D. must
Question 39: A. means B. opinion C. idea D. method
Question 40: A. about B. of C. over D. in
Question 41: A. As far as B. By far C. So far D. As long as
Question 42: A. in the end B. lastly C. at last D. eventually

Read the following passage and mark the letter A, B, C, or D on your answer sheet to
indicate the correct answer to each of the questions from 43 to 50.
Bees, classified into over 10,000 species, are insects found in almost every part of the world
except the northernmost and southernmost regions. One commonly known species is the
honeybee, the only bee that produces honey and wax. Humans use the wax in making candles,
lipsticks, and other products, and they use the honey as a food. While gathering the nectar and
pollen with which they make honey, bees are simultaneously helping to fertilize the flowers on
which they land. Many fruits and vegetables would not survive if bees did not carry the pollen from
blossom to blossom.
      Bees live in a structured environment and social structure within a hive, which is a nest with
storage space for the honey. The different types of bees each perform a unique function. The
worker bee carries nectar to the hive in a special stomach called a honey stomach. Other workers
make beeswax and shape it into a honeycomb, which is a waterproof mass of six-sided
23
compartments, or cells. The queen lays eggs in completed cells. As the workers build more cells,
the queen lays more eggs.
All workers, like the queen, are female, but the workers are smaller than the queen. The male
honeybees are called drones; they do no work and cannot sting. They are developed from
unfertilized eggs, and their only job is to impregnate a queen. The queen must be fertilized in order
to lay worker eggs. During the season when less honey is available and the drones is of no further
use, the workers block the drones from eating the honey so that they will starve to death.
Question 43: Which of the following is the best title for this reading?
A. The Honeybee – Its Characteristics and Usefulness
B. The Many Species of Bees
C. The Useless Drone
D. Making Honey
Question 44: The word “species” in the first sentence probably means ______.
A. mates B. killers C. varieties D. enemies
Question 45: It is stated in paragraph 2 that a hive is ________.
A. a type of honey B. a storage space C. a type of bee D. a nest
Question 46: According to the passage, the drone ________.
A. collects less honey than workers
B. mates with the queen and has no other purpose
C. comes from eggs fertilized by other drones
D. can be male or female
Question 47: The author implies that __________.
A. drones are never females B. bees are unnecessary in the food chain
C. drones are completely dispensable D. the queen can be a worker
Question 48: According to the passage, honey is carried to the hive in a honey stomach by the _____.
A. queens B. drones C. males D. workers
Question 49: The word “They” in the last paragraph refers to _____.
A. workers B. queens C. honeybees D. drones
Question 50: It can be inferred from the passage that beeswax is _________.
A. absorbent B. complex in structure C. pliable D. sweet

~~ THE END ~~

PRACTICE TEST 18
Mark the letter A, B, C, or D on your answer sheet to indicate the word whose
underlined part differs from the other three in pronunciation in each of the following
questions.
Question 1: A. employed B. called C. expressed D. prepared
Question 2: A. although B. weather C. healthy D. themselves

Mark the letter A, B, C, or D on your answer sheet to indicate the word that differs from
the other three in the position of primary stress in each of the following questions.
Question 3: A. preference B. attraction C. advantage D. infinitive
Question 4: A. picturesque B. mathematician C. refugee D. cigarette

Mark the letter A, B, C, or D on your answer sheet to indicate the underlined part that

24
needs correction in each of the following questions.
Question 5: The man, of whom the red car is parked in front of our house, is a famous doctor in
this town. A B C D
Question 6: If either of you take a vacation, we will not be able to finish the work.
A B C D
Question 7: The better you are at English, more chance you have to get a job with international
organizations. A B C D

Mark the letter A, B, C, or D on your answer sheet to indicate the correct answer to each
of the following questions.
Question 8: Thousand of antibiotics , but only about thirty are in common use today.
A. have developed B. are developing
C. have been developed D. have been developing
Question 9: Everyone hopes to succeed in life, ?
A. haven’t they B. don’t they C. hasn’t he D. doesn’t he
Question 10: When his business failed, he started again from .
A. beginning B. blank C. scratch D. introduction
Question 11: Can you help me sort these clothes into different sizes?
A. for B. between C. with D. out
Question 12: “Lac Troi”, is a newly composed song of Son Tung MTP, has become a musical
phenomenon throughout Vietnam these days.
A. who B. which C. what D. that
Question 13: By the household chores, he can help his mother after going home from school.
A. ordering B. doing C. having D. making
Question 14: This factory produced motorbikes in 2008 as in the year 2006.
A. as twice many B. as twice as many C. twice as many D. as many as twice
Question 15: It’s essential that every student the exam before attending the course.
A. passes B. would pass C. passed D. pass
Question 16: You must these instructions very carefully.
A. bring out B. carry out C. carry on D. get up to
Question 17: There is a huge amount of associated with children’s TV nowadays.
A. merchandising B. manufacturing C. produce D. sales
Question 18: The doctor told him that the he would gain from a healthy diet would be well
worth the sacrifice.
A. profits B. advantages C. benefits D. welfare
Question 19: Our teacher is a person. If she says she will do something, you know that she will do.
A. depending B. independent C. dependable D. dependent

Mark the letter A, B, C, or D on your answer sheet to indicate the most suitable response to
complete each of the following exchanges.
Question 20: Mark and Susan are now at the music club.
- Mark: “I didn't know you could play the guitar so well, Susan. Your tune was lovely!”
- Susan: “ ”
A. I don't like your saying.
B. Thank you very much, I'm afraid.
C. You must be kidding. I need to practise more.
D. You're telling a lie.
25
Question 21: Linda and Lucy are planning to go out together after class.
- Linda: “What time should we meet?”
- Lucy: “ .”
A. How about meeting again. B. Oh, that's not a good idea.
C. Is 6:30 all right? D. I'd like to go to the library.

Mark the letter A, B, C, or D on your answer sheet to indicate the word or phrase that is
CLOSEST in meaning to the underlined part in each of the following questions.
Question 22: According to World Bank statistics, many developing countries, such as Brazil,
Ecuador, and Indonesia, had lost almost half of their rainforests by 1991.
A. numerical data B. report results C. conclusions D. quantities
Question 23: If we had taken his sage advice, we wouldn’t be in so much trouble now.
A. sturdy B. willing C. wise D. eager

Mark the letter A, B, C, or D on your answer sheet to indicate the word or phrase that is
OPPOSITE in meaning to the underlined part in each of the following questions.
Question 24: He had never experienced such discourtesy towards the president as it occurred at
the annual meeting in May.
A. measurement B. encouragement C. politeness D. rudeness
Question 25: He decided not to buy the fake watch and wait until he had more money.
A. forger B. original C. faulty D. authentic

Mark the letter A, B, C, or D on your answer sheet to indicate the sentence that is closest in
meaning to each of the following questions.
Question 26: “If I were you, I would go to the doctor,” David said to Claudia.
A. David advised Claudia not to go to the doctor.
B. David told Claudia that he would go to see the doctor.
C. David advised Claudia to go to the doctor.
D. David told Claudia to become a doctor.
Question 27: People say that six out of the seven wonders of the ancient world were destroyed.
A. Six out of the seven wonders of the ancient world are said to have been destroyed.
B. It was said that six out of the seven wonders of the ancient world were destroyed.
C. Six out of the seven wonders of the ancient world are said to have destroyed.
D. Six out of the seven wonders of the ancient world is said to have destroyed.
Question 28: We have worked as volunteers at this SOS village for 3 months.
A. It's 3 months since we worked as volunteers at this SOS village.
B. It's 3 months since we stopped working as volunteers at this SOS village.
C. It's 3 months since we didn't work as volunteers at this SOS village.
D. We last worked as volunteers at this SOS village 3 months ago.

Mark the letter A, B, C, or D on your answer sheet to indicate the sentence that best combines each
pair of sentences in the following questions.
Question 29: The old man is working in this factory. I borrowed his bicycle yesterday.
A. The old man whom I borrowed his bicycle yesterday is working in this factory.
B. The old man whom is working in this factory I borrowed his bicycle yesterday.
C. The old man whose bicycle I borrowed yesterday is working in this factory.
D. The old man is working in this factory which I borrowed his bicycle yesterday.
26
Question 30: The girl forgot to set the alarm clock. Therefore, she is in a hurry now.
A. The girl is not in a hurry now although she forgot to set the alarm clock.
B. The girl is not in a hurry now in spite of forgetting to set the alarm clock.
C. The girl forgot to set the alarm clock because she is in a hurry now.
D. The girl is in a hurry now because she forgot to set the alarm clock.

Read the following passage and mark the letter A, B, C, or D on your answer sheet to indicate
the correct word or phrase that best fits each of the numbered blanks.
Public transport plays a central role in any (31) urban transport system. In developing
countries where at least 16 cities are expected to have more than 12 million people each by the end
of this decade, failing to give priority (32) public transport would be disastrous. The term ‘public
transport’ covers many different types of vehicles, but most commonly refers to (33) ________ and
trains. Rail services fall into four major categories, rapid rail (also called the underground, tube,
metro, or subway), which operates on exclusive (34)________or on elevated tracks; trams, which
move with other traffic on regular streets; light rail, which is a quieter, more modern version of trams
that can run either on exclusive way or with other traffic; and suburban or regional trains, which
connect a city with surrounding areas. The recent trend in many cities is toward light rail over ‘heavy’
rapid rail systems. Whereas metros require exclusive rights-of-way, which often means building costly
elevated or underground lines and stations, light rail can be build on regular city streets.
Public transport modes vary in fuel use and emissions and in the space they require, but if carrying
reasonable numbers of passengers, they all perform (35)________ than single-occupant private cars
on each of these counts.
Question 31: A. efficient B. ancient C. rural D. western
Question 32: A. about B. to C. with D. along
Question 33: A. cars B. bicycles C. buses D. horses
Question 34: A. tunnels B. burrows C. caves D. dungeons
Question 35: A. faster B. slower C. worse D. better

Read the following passage and mark the letter A, B, C, or D on your answer sheet to indicate
the correct answer to each of the questions from 36 to 42.
Becoming a teacher demands not only knowledge in an academic field but also a personal
commitment to lifelong learning, and enthusiasm for sharing knowledge with other people. To
become one of those noble educators in the USA, one has to satisfy several basic requirements.
First and foremost, it is a prerequisite to have bachelor's degree in education. In the event that a
candidate already has a bachelor's degree in another field, a teacher preparation program is needed.
But that is not all. Almost every school in the USA understands that real classroom teaching
experience is a vital part of a teacher's training. Before taking over a class, a person typically needs
to coplete a training program, including working as a supervised student teacher.
People who want to become university teachers need master's degrees. Getting a master's
degree is a necessity, but if it is gained too early, there may be concerns that the candidate lacks the
real-world experience to go with it. In fact, very few schools want to hire novices with little or no
classroom experience and even if they are accepted, they are usually ill-paid. One wise solution to
the issue is for future postgraduates to start working as teachers before going on to gain their
master's degree.
Besides knowledge and experience, certain personal qualities are also required. A teacher should
be positive, prepared, focused, and most importantly, patient. Being a teacher involves being aware
of the fact that learning sometimes be hard work, even for the most motivated students. Also,
27
teaching can at times be tiring and frustrating, so teaching candidates have to practise being patient
with themselves.
In short, as in other careers, teaching requires a combination of qualifications, experience, and
personal qualities. Teaching candidates meeting mandatory requirements are always in demand in
the USA.
Question 36: The text is mainly about .
A. the importance of teachers
B. the advantages and disadvantages of being a teacher in the USA
C. the difference of teaching career
D. the basic requirements of being a teacher in the USA
Question 37: According to the text, future postgraduates should start working as teachers
_________.
A. after gaining their master's degree
B. before studying for their master's degree
C. during the time they are studying for their master's degree
D. before studying for their bachelor's degree
Question 38: The word “vital” in paragraph 2 is closest in meaning to .
A. very useless B. very easy C. very important D. very interesting
Question 39: According to the text, teaching requires a a combination of many things EXCEPT
___________.
A. qualifications B. personal qualities C. experience D. appearance
Question 40: According to the text, the most imporatnt quality of a teacher is .
A. being patient B. being to work hard C. being prepared D. being a role model
Question 41: The word “they” in paragraph 3 refers to .
A. postgraduates B. novices C. schools D. teachers
Question 42: According to the text, all of the following sentences are true EXCEPT____ .
A. Those who want to become university teachers need master's degrees.
B. A teachers needs to be aware of the fact that learning can sometimes be hard work.
C. A great number of schools in the USA want to hire novices with little or no classroom experience.
D. In the USA, before one takes over a class, a training program is typically necessary to be
completed.

Read the following passage and mark the letter A, B, C, or D on your answer sheet to indicate
the correct answer to each of the questions from 43 to 50.
The general principles of dynamics are rules that demonstrate a relationship between the
motions of bodies and the forces that produce those motions. Based in large part on the work of his
predecessors, Sir Isaac Newton deduced three laws of dynamics, which he published in his famous
Principal.
Prior to Newton, Aristotle had established that the natural state of a body was a state of rest,
and that unless a force acted upon it to maintain motion, a moving body would come to rest. Galileo
had succeeded in correctly describing the behavior of falling objects and in recording that no force
was required to maintain a body in motion. He noted that the effect of force was to change motion.
Huygens recognized that a change in the direction of motion involved acceleration, just as did a
change in speed, and further, that the action of a force was required. Kepler deduced the laws
describing the motion of planets around the sun. It was primarily from Galileo and Kepler that
Newton borrowed.
In short, Newton’s laws of Motion are: a body at rest remains at rest, and a body in motion
28
remains in motion along a straight line, unless acted upon by an unbalanced force, if an unbalanced
force acts upon a body, the momentum of the body changes in proportion to the force and in the
same direction as the force, to every action or force, there is an equal and opposite reaction.
Question 43: What was the main purpose of this passage?
A. To demonstrate the development of Newton’s laws
B. To establish Newton as the authority in the field of physics
C. To discredit Newton’s laws of motion
D. To describe the motion of planets around the sun
Question 44: The word “predecessors” in paragraph 1 refers to .
A. those who came before B. those who provided help
C. those who published their work D. those who agreed with the ideas
Question 45: The phrase “prior to” could best be replaced by which of the following?
A. after B. with C. before D. simultaneously
Question 46: The word “it” in paragraph 2 refers to .
A. rest B. body C. state D. motion
Question 47: According to Huygens, when was acceleration required?
A. For either a change in direction or a change in speed
B. Only for a change in speed
C. Only for a change in direction
D. Neither for a change in direction nor for a change in speed
Question 48: According to this passage, Newton based his laws primarily upon the work of
_________ .
A. Galileo and Copernicus B. Ptolemy and Copernicus
C. Huygens and Kepler D. Galileo and Kepler
Question 49: The word “momentum” in the last paragraph is closest in meaning to .
A. weight B. speed C. shape D. size
Question 50: Which of the following describes Inertia, or the principles of bodies at rest?
A. Newton’s first law B. Newton’s third law
C. Newton’s law of motion D. Newton’s law of dynamics
~~ THE END ~~

PRACTICE TEST 19
Mark the letter A, B, C, or D on your answer sheet to indicate the word whose
underlined part differs from the other three in pronunciation in each of the following
questions.
Question 1: A. brush B. rush C. push D. crush
Question 2: A. approached B. sacrificed C. unwrapped D. obliged

Mark the letter A, B, C, or D on your answer sheet to indicate the word that differs from
the other three in the position of primary stress in each of the following questions.
Question 3: A. preview B. commit C. recipe D. index
Question 4: A. continental B. considerate C. territorial D. economic

29
Mark the letter A, B, C, or D on your answer sheet to indicate the underlined part that
needs correction in each of the following questions.
Question 5: Americans have begin leaning towards the idea that marriage is more of an option,
rather than a milestone on the path to adulthood.
A. is B. towards C. have begin D. on the path
Question 6: Once you realize that there are ways to move on from awkward situations with grace
and confidence, you’ll be on your way to embrace social interactions instead of dreading them.
A. embrace B. Once C. are D. grace
Question 7: Our brains are incredibly agile machines, and it’s hard to think of anything they do
more efficient than recognize faces.
A. recognize B. anything C. efficient D. are

Mark the letter A, B, C, or D on your answer sheet to indicate the correct answer to
each of the following questions.
Question 8: By the end of next year, George ________ English for 2 years.
A. would learn B. will have learnt C. will have D. has learnt
Question 9: Neither Peter nor his parents ________ going to spend the summer abroad.
A. is B. are C. was D. has been
Question 10: The Lake District, ________ was made a national park in 1951, attracts a large
number of tourists every year.
A. which B. that C. where D. what
Question 11: As a millionaire who liked to show off her wealth, Mrs. Smith paid ________ we
asked.
A. four times as much as B. four time much than
C. four time as many as D. four times much as
Question 12: ________ my mother’s encouragement, I wouldn’t have made such a daring
decision.
A. In spite B. Providing C. Until D. But for
Question 13: Is there anything wrong ________ your bicycle?
A. with B. of C. in D. about
Question 14: There’s a lot more to Willie than one would think: still waters run ________
A. deep B. deeply C. deepness D. depth
Question 15: His emotional problems ________ from the attitudes he encountered as a child, I
think.
A. stem B. flourish C. root D. sprout
Question 16: I’m not keen on ________ control of the project to a relatively newcomer.
A. undertaking B. charging C. entrusting D. allotting
Question 17: People who are educated overseas can ________ recognition of training and get
employment easily.
A. gain B. grow C. train D. see
Question 18: The young lady sat still in the afternoon breeze, with her hair_____ her back.
A. running over B. fallen against C. flowed down D. streaming down
Question 19: He was completely ________ by her tale of hardship.
A. taken away B. taken in C. taken up D. taken down

Mark the letter A, B, C, or D on your answer sheet to indicate the most suitable
response to complete each of the following exchanges.
30
Question 20: Monica and Mathew are on the way home.
~ Monica: “I think we should recycle these bags. It will help protect the environment.”
~ Mathew: “________”
A. I can't agree with you more. B. It's rubbish. We shouldn't use it.
C. Never mind. D. You can say that again.
Question 21: Two friends Stephanie and Scott are preparing for their first lesson at school.
~ Stephanie: “Oh, no! I left my book at home. Can I share yours?”~ Scott: “________”
A. Yes, I do too. B. No, thanks. C. No, not at all! D. Yes, sure!

Mark the letter A, B, C, or D on your answer sheet to indicate the word that is CLOSEST
in meaning to the underlined part in each of the following questions.
Question 22: For many people, business socialising is a very important aspect of working life - but
some people find it quite tricky, especially if English isn’t their first language.
A. strict B. difficult C. endurable D. ingenious
Question 23: The puddles had coalesced into a small stream.
A. cooled B. burned C. united D. exploded

Mark the letter A, B, C, or D on your answer sheet to indicate the word that is
OPPOSITE in meaning to the underlined part in each of the following questions.
Question 24: After five days on trial, the court found him innocent of crime and he was released.
A. innovative B. naive C. benevolent D. guilty
Question 25: Affluent families find it easier to support their children financially.
A. Impoverished B. Wealthy C. Privileged D. Well-off

Mark the letter A, B, C, or D on your answer sheet to indicate the sentence that is
closest in meaning to each of the following questions.
Question 26: Slightly more than twenty-five percent of the students in the class come from
Spanish speaking countries.
A. The percentage of the students speaking Spanish fell by twenty-five percent.
B. Seventy-five percent of the students in the class speak Spanish.
C. A considerable proportion of the students in the class are Spanish.
D. A small minority of the students of in the class are Hispanic.
Question 27: “Send this urgent document immediately!” the officer told the soldier .
A. The officer ordered the soldier to deliver the urgent document instantly.
B. The officer advised the soldier to send the urgent document right away.
C. The officer requested that the soldier rush out due to the document’s urgency.
D. The officer recommended the soldier leave right away because of the urgent document.
Question 28: Had he known more about the Internet, he’d have invested in some computer
companies.
A. Knowing about the Internet helped him invest in some computer companies.
B. He didn’t know much about the Internet so he didn’t invest in any computer companies.
C. Knowing about the Internet, he would have invested in some computer companies.
D. He would have invested in some computer companies without his knowledge of the Internet.

Mark the letter A, B, C, or D on your answer sheet to indicate the sentence that best
combines each pair of sentences in the following questions.

31
Question 29: Overeating is a cause of several deadly diseases. Physical inactivity is another cause
of several deadly diseases.
A. Both overeating and physical inactivity result from several deadly diseases.
B. Not only overeating but also physical inactivity may lead to several deadly diseases.
C. Overeating and physical inactivity are caused by several deadly diseases.
D. Apart from physical activities, eating too much also contributes to several deadly diseases.
Question 30: He was successful because he was determined to pursue personal goals. He was not
talented.
A. It was his determination to pursue personal goals, not talent, that contributed to his success.
B. In addition to his determination, his talent ensured his success in pursuing his goals.
C. His success lay in his natural ability, not in his determination to pursue personal goals.
D. His determination to pursue personal goals made him successful and talented.

Read the following passage and mark the letter A, B, C, or D on your answer sheet to
indicate the correct word or phrase that best fits the numbered blanks.
Many people today would like the traditional two-parent family back, that is to say, they want a
man and a woman to (31)_______ for life; they also think the man should support the family and
the woman should stay home with the children. However, few families now (32)_______into this
category. In fact, if more women decide to have children on their own, the single-parent household
may become more typical than the traditional family in many countries. Also, unmarried couples
may decide to have more children – or they might take in foster children or (33)_______ And
because people are staying single and living longer (often as widows), there may be more one-
person households. (34)________ the other hand, some people believe similar events happen again
and again in history: if this is true, people may go back to the traditional (35)________or nuclear
family of the past.
Others think the only certainty in history is change: in other words, the structure of the future
family could begin to change faster and faster and in more and more ways.
Question 31: A. marry B. wed C. engage D. gather
Question 32: A. belong B. crowd C. group D. fall
Question 33: A. bring up B. feel C. adapt D. adopt
Question 34: A. In B. At C. On D. For
Question 35: A. extended B. extensive C. enlarged D. big

Read the following passage and mark the letter A, B,C, or D on your answer sheet to
indicate the correct answer to each of the questions from 36 to 42.
Bacteria are extremely small living things. While we measure our own sizes in inches or
centimetres, bacterial size is measured in microns. One micron is a thousandth of a millimetre; a
pinhead is about a millimetre across. Rod-shaped bacteria are usually from two to four microns
long,while rounded ones are generally one micron in diametre. Thus if you enlarged a rounded
bacterium a thousand times, it would be just about the size of a pinhead. An adult human
magnified by the same amount would be over a mile (1.6 kilometres) tall.
Even with an ordinary microscope, you must look closely to see bacteria. Using a
magnification of 100 times, one finds that bacteria are barely visible as tiny rods or dots One cannot
make out anything of their structure. Using special stains, one can see that some bacteria have
attached to them wavy - looking "hairs" called flagella. Others have only one flagellum. The flagella
rotate, pushing the bacteria though the water. Many bacteria lack flagella and cannot move about

32
by their own power while others can glide along over surfaces by some little understood
mechanism.
From the bacterial point of view, the world is a very different place from what it is to humans. To
a bacterium, water is as thick as molasses is to us. Bacteria are so small that they are influenced by
the movements of the chemical molecules around them. Bacteria under the microscope, even those
with no flagella, often bounce about in the water. This is because they collide with the water
molecules and are pushed this way and that. Molecules move so rapidly that within a tenth of a
second, the molecules around a bacterium have all been replaced by new ones. Even bacteria
without flagella are thus constantly exposed to a changing environment.
Question 36: Which of the following is the main topic of the passage?
A. The characteristics of bacteria B. How bacteria reproduce
C. The various functions of bacteria D. How bacteria contribute to disease
Question 37: Bacteria are measured in ________
A. inches B. centimeters C. microns D. millimeters
Question 38: Which of the following is the smallest?
A. A pinhead B. A rounded bacterium C. A microscope D. A rod-shaped bacterium
Question 39: The word “ordinary” in the second paragraph refers to ________
A. usual B. distinctive C. original D. extraordinary
Question 40: According to the passage, someone who examines bacteria using only a microscope
that magnifies 100 times would see ________
A. tiny dots B. small “hairs” C. large rods D. detailed structures
Question 41: The relationship between a bacterium and its flagella is most nearly analogous to
which of the following?
A. A rider jumping on a horse's back B. A ball being hit by a bat
C. A boat powered by a motor D. A door closed by a gust of wind
Question 42: The author compares water to molasses, in order to introduce which of the following
topics?
A. The bacterial content of different liquids
B. What happens when bacteria are added to molasses
C. The molecular structures of different chemicals
D. How difficult it is for bacteria to move through water.

Read the following passage and mark the letter A, B,C, or D on your answer sheet to
indicate the correct answer to each of the questions from 43 to 50.
Since water is the basis of life, composing the greater part of the tissues of all living things, the
crucial problem of desert animals is to survive in a world where sources of flowing water are rare.
And since man’s inexorable necessity is to absorb large quantities of water at frequent intervals, he
can scarcely comprehend that many creatures of the desert pass their entire lives without a single
drop.
Uncompromising as it is, the desert has not eliminated life but only those forms unable to
withstand its desiccating effects. No moist-skinned, water-loving animals can exist there. Few large
animals are found. The giants of the North American desert are the deer, the coyote, and the
bobcat. Since desert country is open, it holds more swift-footed running and leaping creatures than
the tangled forest. Its population is largely nocturnal, silent, filled with reticence, and ruled by
stealth. Yet they are not emaciated. Having adapted to their austere environment, they are as
healthy as animals anywhere else in the word.

33
The secret of their adjustment lies in the combination of behaviour and physiology. None could
survive if, like mad dogs and Englishmen, they went out in the midday sun; many would die in a
matter of minutes. So most of them pass the burning hours asleep in cool, humid burrows
underneath the ground, emerging to hunt only by night. The surface of the sun-baked desert
averages around 150 degrees, but 18 inches down the temperature is only 60 degrees.
Question 43: The title for this passage could be ________
A. “Desert Plants” B. “Life Underground”
C. “Animal Life in a Desert Environment” D. “Man’s Life in a Desert Environment”
Question 44: Man can hardly understand why many animals live their whole life in the desert, as
________
A. sources of flowing water are rare in a desert
B. water is an essential part of his existence
C. water composes the greater part of the tissues of living things
D. very few large animals are found in the desert
Question 45: The phrase “those forms” in the passage refers to all of the following
EXCEPT________
A. water-loving animals B. the coyote and the bobcat
C. moist-skinned animals D. many large animals
Question 46: According to the passage, creatures in the desert ________
A. are smaller and fleeter than forest animals
B. live in an accommodating environment
C. are more active during the day than those in the tangled forest
D. are not as healthy as those anywhere else in the world
Question 47: The author mentions all the following as examples of the behaviour of desert
animals EXCEPT ________
A. they sleep during the day B. they dig home underground
C. they are noisy and aggressive D. they are watchful and quiet
Question 48: The word “emaciated” in the passage mostly means ________
A. wild B. cunning C. unmanageable D. unhealthy
Question 49: The word “them” means ________
A. animals B. people C. water D. minutes
Question 50: We can infer from the passage that ________
A. living things adjust to their environment B. water is the basis of desert life
C. desert life is colourful and diverse D. healthy animals live longer lives
~~THE END~~

PRACTICE TEST 20
Mark the letter A, B, C, or D on your answer sheet to indicate the word whose
underlined part differs from the other three in pronunciation in each of the following
questions.
Question 1: A. route B. scout C. doubt D. trout
Question 2: A. ruins B. economics C. leftovers D. details

Mark the letter A, B, C, or D on your answer sheet to indicate the word that differs from
the other three in the position of primary stress in each of the following questions.
Question 3: A. maintain B. retain C. fountain D. entire
34
Question 4: A. referee B. electrician C. manufacture D. immortal

Mark the letter A, B, C, or D on your answer sheet to indicate the underlined part that
needs correction in each of the following questions.
Question 5: The officials of the Board of Elections asked that each voter present their registration
card and a valid Texas driver’s license before receiving a ballot.
A. receiving B. driver’s C. their D. present
Question 6: For such a demanding job, you will need qualifications, soft skills and having full
commitment.
A. will need B. such C. qualifications D. having full compliment
Question 7: Several people have apparent tried to change the man’s mind, but he refuses to
listen.
A. apparent B. Several C. mind D. to listen

Mark the letter A, B, C, or D on your answer sheet to indicate the correct answer to
each of the following questions.
Question 8: The restaurants on the island are expensive, so it’s worth ________ a packed lunch.
A. take B. to take C. taken D. taking
Question 9: ________ is caused by a virus was not known until 1911.
A. That measles B. Measles C. What if measles D. As measles
Question 10: ________ smoking is a causative factor of many diseases, there is no ban on
tobacco advertising.
A. In spite of B. However C. Although D. Therefore
Question 11: Simple sails were made from canvas ________ over a frame.
A. stretched B. was stretched C. a stretch D. it was stretched
Question 12: Last year she earned ________ her brother.
A. twice as much as B. twice more than C. twice as many as D. twice as more as
Question 13: Helen is allergic ________ seafood, so she never tries these delicious dishes.
A. down B. up C. to D. on
Question 14: Fiona loves reading __________ novels in her leisure time to help her unwind from
her hectic life.
A. escaping B. escapist C. escapable D. escapism
Question 15: The politician tried to arouse the crowd, but most of them were ________ to his
arguments.
A. closed B. careless C. indifferent D. dead
Question 16: After nine months without any rain, the country was facing one of the worst
________ in the last fifty years.
A. draughts B. floods C. eruptions D. droughts
Question 17: We must push the piano to the corner of the hall to ________ our party tonight.
A. make place for B. take up room to C. give place to D. make room for
Question 18: On Christmas Eve, the ________ family gathers for dinner, usually at my
grandmother’s house.
A. mere B. total C. entire D. complete
Question 19: John lost the ________ bicycle he bought last week and his parents were very angry
with him because of his carelessness.
A. beautiful Japanese blue new B. beautiful new blue Japanese
C. new beautiful blue Japanese D. Japanese beautiful new blue
35
Mark the letter A, B, C, or D on your answer sheet to indicate the most suitable
response to complete each of the following exchanges.
Question 20: Phuong Thao is a student in Ms. Lan’s writing class. She is asking for Ms. Lan’s
comments on her last essay.
~ Phuong Thao: "You must have found reading my essay very tiring."
~ Ms. Lan: "________ I enjoyed it."
A. Just in case B. You’re welcome C. Not in the least D. At all costs
Question 21: Ann’s mother is cooking in the kitchen.
~ Ann: "Do you need any help?"
~ Ann’s mother: "________"
A. No, thanks. I can manage. B. I haven’t got a clue.
C. That’s all for now. D. That’s fine by me.

Mark the letter A, B, C, or D on your answer sheet to indicate the word that is CLOSEST
in meaning to the underlined part in each of the following questions.
Question 22: Many scientists agree that global warming poses great threats to all species on
Earth.
A. fears B. annoyances C. risks D. irritations
Question 23: Because of cutbacks in council spending, plans for the new swimming pool had to be
shelved.
A. stopped B. cancelled C. disapproved D. delayed

Mark the letter A, B, C, or D on your answer sheet to indicate the word that is
OPPOSITE in meaning to the underlined part in each of the following questions.
Question 24: Because Jack defaulted on his loan, the bank took him to court.
A. failed to pay B. paid in full C. had a bad personality D. was paid much money
Question 25: Population growth rates vary among regions and even among countries within the
same region.
A. stay unchanged B. fluctuate C. remain unstable D. restrain

Mark the letter A, B, C, or D on your answer sheet to indicate the sentence that is
closest in meaning to each of the following questions.
Question 26: I am sure he did not know that his brother graduated with flying colours.
A. That his brother graduated with flying colours must have been appreciated by him.
B. He cannot have known that his brother graduated with very high marks.
C. He should not have been envious of his brother’s achievement.
D. He may not know that his brother is flying gradually up in a colourful baloon.
Question 27: “We lost the last game because of the referee,” said the team captain.
A. The team captain refused to tell the referee about their loss in the last game.
B. The team captain admitted to the referee that they had lost in the last game.
C. The team captain blamed the referee for their loss in the last game.
D. The team captain said that without the referee, they might have lost the last game.
Question 28: The president placed his car at my disposal as a bonus for my good work .
A. To get rid of the car, the president decided to sell it to me, his good worker, at a bonus price.
B. I was willing to drive the president’s car as a compliment for my good performance at work.
C. In order to praise me as a good worker, the president took me home in his own car.
36
D. To show his appreciation for my good work, the president allowed me to use his car whenever I
liked.

Mark the letter A, B, C, or D on your answer sheet to indicate the sentence that best
combines each pair of sentences in the following questions.
Question 29: I had never seen here before. However, I recognized her from a photograph.
A. Although I had never seen her before, I recognized her from a photograph.
B. I recognized her from a photograph before I had never seen her.
C. Although I had never seen her before but I recognized her from a photograph.
D. After I had seen her, I recognized her from a photograph.
Question 30: They drove fifteen miles off the main road. Also, they had nothing to eat for the day.
A. They drove fifteen miles off the main road until they had something to eat for the day.
B. They neither drove fifteen miles off the main road nor had anything to eat for the day.
C. Not only did they drive fifteen miles off the main road, they also had nothing to eat for the day.
D. Driving fifteen miles off the main road, they eventually had something to eat for the day.

Read the following passage and mark the letter A, B, C, or D on your answer sheet to
indicate the correct word or phrase that best fits each of the numbered blanks.
Freya Stark (1893-1993) was an extraordinary woman who ventured into remote areas where
few Europeans had ever (31) ________ foot. Born in Paris, brought up in Italy, and educated in
London, she was already multilingual before deciding to learn Arabic at the end of 30. She travelled
(32)________ through Greece, Italy and Cyprus. However, her passion was for exploring ancient
lands of the Middle East. She was determined to make contact with the tribes that populated the
area before their traditional way of life vanished forever. She led expeditions into the most
dangerous areas and despite the risks (she was even thrown into a military prison on one
occasion), she always returned safe and (33)________. During her travels, she completed a trek
across the Valley of the Assassins, famous not only for its rough terrain but also for its murderous
inhabitants, and followed in the (34) ______ of Alexander, the Great. In her long career she
produced maps, discovered lost cities, and worked as a spy and propagandist, although it is as a
travel writer that she is best remembered. She died at the age of 100, (35) ________ most women
of her generation in more ways than one.
Question 31: A. placed B. got C. set D. put
Question 32: A. deliberately B. privately C. extensively D. broadly
Question 33: A. again B. back C. round D. sound
Question 34: A. departure B. footsteps C. time D. trail
Question 35: A. outstaying B. outweighing C. outliving D. outnumbering

Read the following passage and mark the letter A, B,C, or D on your answer sheet to
indicate the correct answer to each of the questions from 36 to 42.
What makes it rain? Rain falls from clouds for the same reason as anything falls to Earth. The
Earth's gravity pulls it. But every cloud is made of water droplets or ice crystals. Why doesn't rain or
snow fall constantly from all clouds? The droplets or ice crystals in clouds are exceedingly small.
The effect of gravity on them is minute. Air currents move and lift droplets so that the net
downward displacement is zero, even though the droplets are in constant motion.
Droplets and ice crystals behave somewhat like dust in the air made visible in a shaft of
sunlight. To the casual observer, dust seems to act in a totally random fashion, moving about
chaotically without fixed direction. But in fact dust particles are much larger than water droplets
37
and they finally fall. The cloud droplet of average size is only 1/2500 inch in diametre. It is so small
that it would take sixteen hours to fall half a mile in perfectly still air, and it does not fall out of
moving air at all. Only when the droplet grows to a diametre of 1/125 inch or larger can it fall from
the cloud. The average raindrop contains a million times as much water as a tiny cloud droplet. The
growth of a cloud droplet to a size large enough to fall out is the cause of rain and other forms of
precipitation. This important growth process is called "coalescence."
Question 36: What is the main topic of the passage?
A. The mechanics of rain B. The climate of North America
C. How gravity affects agriculture D. Types of clouds
Question 37: The word “minute” in the first paragraph is closest in meaning to ________
A. second B. tiny C. slow D. predictable
Question 38: Why don' t all ice crystals in clouds immediately fall to earth?
A. They are balanced by the pressure of rain droplets.
B. The effect of gravity at high altitude is random.
C. They are kept aloft by air currents.
D. The heat from the sun's rays melts them.
Question 39: The word “visible” in the second paragraph is closest in meaning to _______
A. uncertain B. vague C. invisible D. obvious
Question 40: What can be inferred about drops of water larger than 1/125 inch in diametre?
A. They never occur.
B. They are not affected by the force of gravity.
C. In still air they would fall to earth.
D. In moving air they fall at a speed of thirty-two miles per hour.
Question 41: In this passage, what does the term “coalescence” refer to ________
A. The gathering of small clouds to form larger clouds
B. The growth of droplets
C. The fall of raindrops and other precipitation
D. The movement of dust particles in the sunlight
Question 42: What is the diametre of the average cloud droplet?
A. 1/16 inch B. 1/125 inch C. 1/2500 inch D. One million of an inch
Read the following passage and mark the letter A, B,C, or D on your answer sheet to
indicate the correct answer to each of the questions from 43 to 50.
The Harlem Renaissance, a movement of the 1920’s, marked the twentieth century’s first
period of intense activity by African Americans in the field of literature, art, and music in the United
States. The philosophy of the movement combined realism, ethnic consciousness, and Americanism.
Encouraged by the example of certain Americans of European descent such as Thomas Eakins,
Robert Henri, and George Luks, who had included persons of African descent in their paintings as
serious studies rather than as trivial or sentimental stereotypes, African American artists of this
period set about creating a new portrayal of themselves and their lives in the United States. As they
began to strive for social and cultural independence. Their attitudes towards themselves changed,
and, to some extent, other segments of American society began to change their attitudes towards
them, thus, thought the Harlem Renaissance was a short-lived movement, its impact on American
art and culture continues to the present.
The district in New York City know as Harlem was the capital of the movement. In 1925 an
issue of Survey Graphic magazine devoted exclusively to Harlem and edited by philosopher Alain
Locke became the manifesto of the African American artistic movement. Locke strongly suggested
that individuals, while accepting their Americanism, take pride in their African ancestral arts and
38
urged artists to look to Africa for substance and inspiration. Far from advocating a withdrawal from
American culture, as did some of his contemporaries, Locke recommended a cultural pluralism
through which artists could enrich the culture of America. African Americans were urged by Locke
to be collaborators and participators with other Americans in art, literature, and music; and at the
same time to preserve, enhance, and promote their own cultural heritage.
Artists and intellectuals from many parts of the United States and the Caribbean had been
attracted to Harlem by the pulse and beat of its unique and dynamic culture. From this unity
created by the convergence of artists from various social and geographical backgrounds came a
new spirit, which, particularly in densely populated Harlem, was to result in greater group
awareness and self-determination. African American graphic artists took their place beside the
poets and writers of the Harlem Renaissance and carried on efforts to increase and promote the
visual arts.
Question 43: What does the passage mainly discuss?
A. African American paintings in the 1920’s.
B. An arts movement of the 1920’s.
C. The influence of Alain Locke on African American art.
D. Some ways in which African culture inspired American literature, art and music.
Question 44: According to the passage, Tomas Eakins, Robert Henri, and George Luks were
important because of ________
A. the philosophical contributions they made to the Harlem Renaissance
B. their development of a new style of African American art
C. the way in which they depicted African Americans in their paintings
D. their independence from European artistic traditions
Question 45: The word “them” in the first paragraph refers to ________
A. Americans of European descent B. paintings
C. African American artists D. attitudes
Question 46: According to the passage, African American artists of the 1920’s differed from
earlierAfrican American artists in terms of their feelings about _________
A. themselves B. other artists C. their impact on American art D. stereotypes
Question 47: The word “urged” in the second paragraph is closest is meaning to _______
A. prepared B. defined C. permitted D. encouraged
Question 48: In mentioning “the pulse and beat” of Harlem during the 1920’s, the author is
characterizing the district as one that ________
A. depended greatly on its interaction with other parts of the city
B. grew economically in a short period of time
C. was an exciting place to be
D. was in danger of losing population
Question 49: The word “convergence” in the last paragraph is closest in meaning to ____
A. gathering B. promotion C. expression D. influence
Question 50: According to the passage, all of the following were true of Harlem in the 1920’s
EXCEPT ________
A. Some Caribbean artists and intellectuals lived there.
B. It attracted people from various regions of United States.
C. It was one of the most expensive neighborhoods in New York City.
D. It was a unique cultural centre.

~~THE END~~
39
PRACTICE TEST 21
Mark the letter A, B, C, or D on your answer sheet to indicate the word whose
underlined part differs from the other three in pronunciation in each of the following
questions.
Question 1: A. charity B. chaos C. champion D. chin
Question 2: A. through B. thought C. enormous D. taught

Mark the letter A, B, C, or D on your answer sheet to indicate the word that differs from
the other three in the position of primary stress in each of the following questions.
Question 3: A. computer B. domestic C. substantial D. dominate
Question 4: A. difficulty B. participate C. appropriate D. relationship

Mark the letter A, B, C, or D on your answer sheet to indicate the underlined part that
needs correction in each of the following questions.
Question 5: It is believed that in the near future robots will be used to doing things such as
cooking.
A. in the near future B. It is believed C. such as D. be used to doing
Question 6: Students suppose to read all the questions carefully and find out the answers to
them.
A. out B. suppose C. all the questions D. them
Question 7: After driving for twenty miles, he suddenly realised that he has driven in the wrong
direction.
A. suddenly realised B. has driven C. After driving D. in

Mark the letter A, B, C, or D on your answer sheet to indicate the correct answer to
each of the following questions.
Question 8: He lost in the election because he is a weak and ________ leader.
A. decision B. undecided C. undecisive D. indecisive
Question 9: Only in Japan ________ the high levels of western countries.
A. industrialization has reached B. industrialization is reached
C. has industrialization reached D. is industrialization reached
Question 10: Staying in a hotel costs ________ renting a room in a dormitory for a week.
A. twice more than B. as much twice
C. more than twice as D. twice as much as
Question 11: ________ increases in population in underdeveloped countries, a lot of problems
arise including health care and social evils.
A. In spite of B. In stead of C. Despite D. Due to
Question 12: Lorie is very thin, ________ her young sister, who is quite fat.
A. unlike B. dissimilar C. dislike D. unlikely
Question 13: I’d rather you ________ in here.
A. don’t smoke B. shouldn’t smoke C. didn’t smoke D. not smoke
Question 14: He did not share his secrets with other people but he ________ in her.
A. confessed B. concealed C. confided D. consented
Question 15: The new magazine about women ________ tomorrow.

40
A. comes down B. comes off C. comes on D. comes out
Question 16: I do not remember ________ anyone that kind of message. It must have been
someone else.
A. to have sent B. sending C. having sending D. to send
Question 17: I think you must be ________ me for someone else.
A. confusing B. reminding C. mistaking D. considering
Question 18: The reason why this game attracts so many youngsters is that ________ other video
games, this one is far more interesting.
A. comparing to B. in compared with C. on comparison to D. in comparison with
Question 19: Population explosion seems to surpass the ability of the earth to meet ________
food.
A. the demanding of B. the demand of C. the demand for D. the demanding for

Mark the letter A, B, C, or D on your answer sheet to indicate the most suitable
response to complete each of the following exchanges.
Question 20: The waitress is apologizing to her customer for her carelessness .
~ Waitress: “Oop! I'm sorry for stepping on your foot.”
~ Customer: “ ________ ”
A. You don't mind B. You’re welcome C. That's fine D. Never mind
Question 21: John is disappointed with the result of his job interview .
~ John: “That's the 16th job interview I've failed. What should I do?"
~ Anna: “ ________ ”
A. Don't give over B. Don't give out C. Don't give up D. Don't give on

Mark the letter A, B, C, or D on your answer sheet to indicate the word that is CLOSEST
in meaning to the underlined part in each of the following questions.
Question 22: In most countries, compulsory military service does not apply to women.
A. superior B. mandatory C. beneficial D. constructive
Question 23: Corn, domesticated by the American Indians, was brought to Europe by
Columbus.
A. trained B. cultivated C. implanted D. reared

Mark the letter A, B, C, or D on your answer sheet to indicate the word that is
OPPOSITE in meaning to the underlined part in each of the following questions.
Question 24: She could not hide her dismay at the result.
A. disappointment B. depression C. happiness D. pessimism
Question 25: “Please speak up a bit more, Jason. You’re hardly loud enough to be heard from the
back,” the teacher said.
A. visible B. edible C. eligible D. inaudible

Mark the letter A, B, C, or D on your answer sheet to indicate the sentence that is
closest in meaning to each of the following questions.
Question 26: It was not only cold but it also snowed a few days ago.
A. Was it not only cold but it also snowed a few days ago.
B. It was not only cold but did it also snow a few days ago.
C. Not only it was cold but did it also snow a few days ago.
D. Not only was it cold but it also snowed a few days ago.
41
Question 27: I would rather you wore something more formal to work.
A. I’d prefer you wearing something more formal to work.
B. I’d prefer you should wear something more formal to work.
C. I’d prefer you to wear something more formal to work.
D. I’d prefer you wear something more formal to work.
Question 28: "Cigarette?" he asked. "No, thanks," I said.
A. He asked if I was smoking, and I denied at once.
B. He mentioned a cigarette, so I thanked him.
C. He asked for a cigarette, and I immediately refuse.
D. He offered me a cigarette, but I promptly declined.
Mark the letter A, B, C, or D on your answer sheet to indicate the sentence that best
combines each pair of sentences in the following questions.
Question 29: What has happened? You look as if you have been in the wars .
A. You look like an old soldier.
B. You are wearing many medals.
C. You look as though something unpleasant has happened to you.
D. You look as though you have been fighting.
Question 30: The agreement ended six-month negotiation. It was signed yesterday .
A. The agreement which ends six-month negotiation was signed yesterday.
B. The agreement which was signed yesterday lasted six months.
C. The negotiation which lasted six months was signed yesterday.
D. The agreement which was signed yesterday ended six-month negotiation.

Read the following passage and mark the letter A, B, C, or D on your answer sheet to
indicate the correct word or phrase that best fits each of the numbered blanks.
Why do people like to chew gum? Some people say they like the taste. (31) ________ say they
can think better if they chew gum. Some people chew it when they have some boring work to do.
Others chew gum when they are nervous. Gum is a mixture of things. For many years, gum
companies made gum from chicle. Chicle is a natural gum from a tree in Mexico and Central
America. Now companies use plastic and rubber made from petroleum (32) ________ of chicle.
Gum must be soft so that you can chew it. A softener keeps it soft. The gum company makes the
softener from vegetable oil. A sweetener makes the gum sweet. The sweetener is usually sugar.
Then the company (33) ________ the flavour.
Thomas Adams made the first gum from chicle in 1836. (34) ________ , chewing gum was not
new. The Greeks chewed gum from a tree more than 2,000 years ago. Mayan Indians in Mexico
chewed chicle. Indians in the Northeastern United States taught Europeans to chew gum from a
tree there. People first made bubble gum in 1928. Children like to (35) ________ bubble with
bubble gum. Some university students do too.
Question 31: A. The other B. Others C. The others D. Other
Question 32: A. aside B. apart C. inside D. instead
Question 33: A. puts B. places C. adds D. fits
Question 34: A. However B. Moreover C. But D. Though
Question 35: A. turn B. set C. pass D. blow

Read the following passage and mark the letter A, B,C, or D on your answer sheet to
indicate the correct answer to each of the questions from 36 to 42.

42
Telecommuting is a form of computer communication between employees’ homes and offices.
For employees whose jobs involve sitting at a terminal or word processor entering data or typing
reports, the location of the computer is of no consequence. If the machine can communicate
over telephone lines, when the work is completed, employees can dial the office computer and
transmit the material to their employers. A recent survey in USA Today estimates that there are
approximately 8,7 million telecommuters. But although the numbers are rising annually, the trend
does not appear to be as significant as predicted when Business Week published “The Portable
Executive” as its cover story a few years ago. Why hasn’t telecommuting become more popular?
Clearly, change simply takes time. But in addition, there has been active resistance on the part
of many managers. These executives claim that supervising the telecommuters in a large work
force scattered across the country would be too difficult, or, at least, systems for managing them
are not yet developed, thereby complicating the manager’s responsibilities. It is also true that
employees who are given the option of telecommuting are reluctant to accept the opportunity. Most
people feel that they need regular interaction with a group, and many are concerned that they will
not have the same consideration for advancement if they are not more visible in the office setting.
Some people feel that even when a space in their homes is set aside as a work area, they never
really get away from the office.
Question 36: How many Americans are involved in telecommuting?
A. More than predicted in Business Week. B. More than 8 million.
C. Fewer than estimated in USA Today. D. Fewer than last year.
Question 37: The phrase "of no consequence" means ________
A. of no use B. of no good C. unimportant D. irrelevant
Question 38: The author mentions all of the following as concerns of telecommuting EXCEPT
________
A. the opportunities for advancement B. the different system of supervision
C. the lack of interaction with a group D. The work place is in the home
Question 39: The word "them" which is bold and italic refers to ________
A. systems B. telecommuters C. executives D. responsibilities
Question 40: The reason why telecommuting has not become popular is that the employees _______
A. needn't regular interaction with their families
B. are worried about the promotion if they are not seen at the office
C. like that a work area in their home is away from the office
D. are ignorant of telecommuting
Question 41: It can be inferred from the passage that the author is ________
A. a telecommuter B. the manager C. a statistician D. a reporter
Question 42: When Business Week published "The Portable Executive", it implied that ____
A. systems for managing telecommuters were not effective
B. there was resistance on the part of many managers about telecommuting
C. the trend for telecommuting was optimistic
D. most telecommuters were satisfied with their work

Read the following passage and mark the letter A, B,C, or D on your answer sheet to
indicate the correct answer to each of the questions from 43 to 50.
Marianne Moore (1887-1972) once said that her writing could be called poetry only because there
was no other name for it. Indeed, her poems appear to be extremely compressed essays that
happen to be printed in jagged lines on the page. Her subjects were varied: animals, labourers,
artists, and the craft of poetry. From her general reading came quotations that she found striking or
43
insightful. She included these in her poems, scrupulously enclosed in quotation marks, and
sometimes identified in footnotes. Of this practice, she wrote, "Why many quotation marks?" I am
asked ... When a thing has been so well that it could not be said better, why paraphrase it? Hence,
my writing is, if not a cabinet of fossils, a kind of collection of flies in amber." Close observation and
concentration on detail and the methods of her poetry.
Marianne Moore grew up in Kirkwood, Missouri, near St. Lois. After graduation from Bryn Mawr
College in 1909, she taught commercial subjects at the Indian School in Carlisle, Pennsylvania.
Later, she became a librarian in New York City. During the 1920’s she was editor of The Dial, an
important literary magazine of the period. She lived quietly all her life, mostly in Brooklyn, New
York. She spent a lot of time at the Bronx Zoo, fascinated by animals. Her admiration of the
Brooklyn Dodgers - before the team moved to Los Angeles - was widely known.
Her first book of poems was published in London in 1921 by a group of friends associated with
the Imagist movement. From that time on, her poetry has been read with interest by succeeding
generations of poets and readers. In 1952, she was awarded the Pulitzer Prize for her Collected
Poems. She wrote that she did not write poetry for money or fame. To earn a living is needful, but
it can be done in routine ways. ‘One writes because one has a burning desire to objectify what it is
indispensable to one's happiness to express.”
Question 43: What is the passage mainly about?
A. The influence of the imagists on Marianne Moore. B. Essayists and poets of the 1920's.
C. The use of quotations in poetry. D. Marianne Moore's life and work.
Question 44: Which of the following can be inferred about Moore's poems?
A. They are better known in Europe than the United States. C. They were all published in The Dial.
B. They do not use traditional verse forms. D. They tend to be abstract.
Question 45: According to the passage Moore wrote about all of the following EXCEPT ___
A. artists B. animals C. fossils D. workers
Question 46: Where did Marianne Moore grow up?
A. In Carlisle, Pennsylvania B. In Kirkwood
C. In New York City D. In Los Angeles
Question 47: The author mentions all of the following as jobs held by Moore EXCEPT_____
A. commercial artist B. teacher C. magazine editor D. librarian
Question 48: Where did Moore spend most of her adult life?
A. In Kirkwood B. In Brooklyn C. In Los Angeles D. In Carlisle
Question 49: The word "succeeding" is closest to ________
A. inheriting B. prospering C. diverse D. later
Question 50: The word "it" refers to ________
A. writing poetry B. becoming famous C. earning a living D. attracting readers
~~THE END~~

PRACTICE TEST 22
Mark the letter A, B, C, or D to indicate the word whose underlined part differs from the
other three in pronunciation in each of the following questions.
Question 1: A. grouse B. brown C. blouse D. growth
Question 2: A. barracks B. series C. means D. headquarters

Mark the letter A, B, C, or D to indicate the word that differs from the other three in the
position of primary stress in each of the following questions.
44
Question 3: A. profile B. surpass C. persuade D. exchange
Question 4: A. preference B. obstacle C. practising D. supporter

Mark the letter A, B, C, or D to indicate the underlined part that needs correction in
each of the following questions.
Question 5: Food prices have raised so rapidly in the past few months that some families have
been forced to alter their eating habits.
A. so rapidly B. eating habits C. raised D. that
Question 6: At least three-quarters of that book on famous Americans are about people who lived
in the nineteenth century.
A. lived B. famous C. are D. three-quarters
Question 7: According to the terms of the contract, the job would have been finished yesterday.
A. the job B. would have been C. According to D. of
Mark the letter A, B, C, or D to indicate the correct answer to each of the following
questions.
Question 8: I think the _________ thing would be to take a taxi home. I’m tired of walking.
A. senseless B. sensational C. sensitive D. sensible
Question 9: They had a _________ candlelit dinner last night and she accepted his proposal of
marriage.
A. romance B. romantic C. romantically D. romanticize
Question 10: A whistle is the _________ for the football players to begin the match.
A. communication B. instance C. attention D. signal
Question 11: A _________ is an official document that you receive when you have completed a
course of study or training.
A. vocation B. subject C. certificate D. grade
Question 12: Tom looks so frightened and upset. He _________ something terrible.
A. must experience B. should have experienced
C. can have experienced D. must have experienced
Question 13: The last of these reasons is _________ the most important.
A. so B. very C. far D. by far
Question 14: _________ we arrived at the hotel when there was a power cut.
A. Not only had B. Hardly had C. No sooner had D. Little had
Question 15: The temperatures _________ take place vary widely for different materials.
A. at which they melt and freeze B. which melting and freezing
C. which they melt and freeze D. at which melting and freezing
Question 16: Jane’s very modest, always _________ her success.
A. playing down B. turning around C. keeping down D. pushing back
Question 17: Neither the director nor the assistant _________ yet.
A. hasn’t come B. haven’t come C. has come D. have come
Question 18: The school Principal suggested that he.....a scholarship.
A. was awarded B. would be awarded C. be awarded D. must be awarded
Question 19: Sometimes in a bad situation, there may still be some good things. Try not to “throw
out the _________ with the bathwater.”
A. fish B. duck C. baby D. child

Mark the letter A, B, C, or D to indicate the most suitable response to complete each of
the following exchanges.
45
Question 20: Two friends Geogre and Frankie are talking to each other.
- Geogre: “In my opinion, action films are exciting.”
- Frankie: “_________”
A. Yes. Congratulations! B. There’s no doubt about it.
C. What an opinion! D. You shouldn’t have said that.
Question 21: Hoa is asking Hai, who is sitting at a corner of the room, seeming too shy .
- Hoa: “Why aren’t you taking part in our activities? ________________”
- Hai: “Yes. I can. Certainly.”
A. What is the matter with you? B. Shall I take your hat off?
C. Can I help you? D. Can you help me with these decorations?

Mark the letter A, B, C, or D to indicate the word that is CLOSEST in meaning to the
underlined part in each of the following questions.
Question 22: The US Congress office building is adjacent to the Capitol building.
A. within B. next to C. far from D. behind
Question 23: Think about the interviewer’s comments because they may help you prepare better
when you are called for the next job interview.
A. compliments B. criticism C. character D. remarks

Mark the letter A, B, C, or D to indicate the word that is OPPOSITE in meaning to the
underlined part in each of the following questions.
Question 24: Names of people in the book were changed to preserve anonymity.
A. reveal B. conserve C. presume D. cover
Question 25: The motorist felt that the ticket for infraction was unwarranted.
A. conscientious B. inadvertent C. inevitable D. justified

Mark the letter A, B, C, or D to indicate the sentence that is closest in meaning to each
of the following questions.
Question 26: What you have been saying is beside the point.
A. You were honest to have said about the point like that.
B. What you have been saying is quite irrelevant.
C. You have been talking too much about the point.
D. What you have been saying is beyond my expectation.
Question 27: Impressed as we were by the new cinema, we found it rather expensive .
A. The new cinema was more expensive than we expected.
B. We were not impressed by the new cinema at all because it looked rather expensive.
C. We weren't as much impressed by the new cinema's look as its cost.
D. We were very impressed by the new cinema, but found it rather expensive.
Question 28: They would never have accepted his money if they had known his plans .
A. They took the money he offered them without realising his purposes.
B. They knew what he wanted to do, so they refused his money.
C. They agreed with his wishes because they were glad to have his money.
D. They didn’t know his plans and never took money from him.

Mark the letter A, B, C, or D to indicate the sentence that best combines each pair of
sentences in the following questions.
Question 29: You don’t try to work hard. You will fail the exam.
46
A. Unless you don’t try to work hard, you will fail in the exam.
B. Unless you try to work hard, you won’t fail in the exam.
C. Unless you try to work hard, you will fail in the exam.
D. Unless do you try to work hard, you will fail in the exam.
Question 30: The coffee was not strong. It didn’t keep us awake.
A. The coffee was very strong, but it couldn’t keep us awake.
B. We were kept awake because the coffee was strong.
C. The coffee was not strong enough to keep us awake.
D. The coffee was so hot that it didn’t keep us awake.

Read the following passage and mark the letter A, B, C, or D to indicate the correct
word or phrase that best fits each of the numbered blanks.
No one can say when sports began. Since it is impossible to (31)_________ a time when children
did not spontaneously run races or wrestle, it is clear that children have always included sports in
their play, but one can only speculate about the emergence of sports as autotelic physical contests
for (32)_________. Hunters are depicted in prehistoric art, but it cannot be known (33)_________
the hunters pursued their prey in a mood of grim necessity or with the joyful abandon of
sportsmen. It is certain, (34)_________, from the rich literary and iconographic evidence of all
ancient civilizations that hunting soon became an end in itself at least for royalty and nobility.
Archaeological evidence also indicates that ball games were common among ancient peoples as
different as the Chinese and the Aztecs. If ball games were contests rather than (35)_________
ritual performances, such as the Japanese football game kemari, then they were sports in the most
rigorously defined sense. That it cannot simply be assumed that they were contests is clear from
the evidence presented by Greek and Roman antiquity, which indicates that ball games had been
for the most part playful pastimes like those recommended for health by the Greek physician Galen
in the 2nd century AD.
Question 31: A. think B. see C. have D. imagine
Question 32: A. people B. children C. adult D. society
Question 33: A. when B. whether C. how D. why
Question 34: A. therefore B. so C. consequently D. however
Question 35: A. competitive B. competitively C. non-competitive D. competition

Read the following passage and mark the letter A, B,C, or D to indicate the correct
answer to each of the questions from 36 to 42.
A number of factors related to the voice reveal the personality of the speaker.
The first is the broad area of communication, which includes imparting information by use of
language, communicating with a group or an individual and specialized communication through
performance. A person conveys thoughts and ideas through choice of words, by a tone of voice that
is pleasant or unpleasant, gentle or harsh, by the rhythm that is inherent within the language itself,
and by speech rhythms that are flowing and regular or uneven and hesitant, and finally, by the
pitch and melody of the utterance. When speaking before a group, a person's tone may indicate
uncertainty or fright, confidence or calm. At interpersonal levels, the tone may reflect ideas and
feelings over and above the words chosen, or may belie them. Here, the participant’s tone can
consciously or unconsciously reflect intuitive sympathy or antipathy, lack of concern or interest,
fatigue, anxiety, enthusiasm or excitement, all of which are usually discernible by the acute listener.
Public performance is a manner of communication that is highly specialized with its own
techniques for obtaining effects by voice and/or gesture. The motivation derived from the text, and
47
in the case of singing, the music, in combination with the performer's skills, personality, and ability
to create empathy will determine the success of artistic, political, or pedagogic communication.
Second, the voice gives psychological clues to a person's self-image, perception of others, and
emotional health. Self-image can be indicated by a tone of voice that is confident, pretentious, shy,
aggressive, outgoing, or exuberant, to name only a few personality traits. Also the sound may give
a clue to the façade or mask of that person, for example, a shy person hiding behind an
overconfident front. How a speaker perceives the listener's receptiveness, interest, or sympathy in
any given conversation can drastically alter the tone of presentation, by encouraging or
discouraging the speaker. Emotional health is evidenced in the voice by free and melodic sounds of
the happy, by constricted and harsh sound of the angry, and by dull and lethargic qualities of the
depressed.
Question 36: What does the passage mainly discuss?
A. The function of the voice in performance B. Communication styles
C. The production of speech D. The connection between voice and personality
Question 37: What does the author mean by staring that, "At interpersonal levels, tone may
reflect ideas and feelings over and above the words “chosen" in line 8-9?
A. Feelings are expressed with different words than ideas are.
B. The tone of voice can carry information beyond the meaning of words.
C. A high tone of voice reflects an emotional communication.
D. Feelings are more difficult to express than ideas.
Question 38: The word “Here” in line 9 refers to _______
A. interpersonal interactions B. the tone C. ideas and feelings D. words chosen
Question 39: According to the passage, an exuberant tone of voice may be an indication of a
person's ________
A. general physical health B. personality
C. ability to communicate D. vocal quality
Question 40: According to the passage, an overconfident front may hide _______
A. hostility B. shyness C. friendliness D. strength
Question 41: The word “drastically” in line 21 is closest in meaning to _______
A. frequently B. exactly C. severely D. easily
Question 42: According to the passage, what does a constricted and harsh voice indicate?
A. lethargy B. depression C. boredom D. anger

Read the following passage and mark the letter A, B,C, or D to indicate the correct
answer to each of the questions from 43 to 50.
Walt Disney was born in 1901 in Chicago, but soon moved to a small farm near Marceline,
Missouri. From this rural and rather humble beginning, he later became one of the most famous
and beloved motionpicture producers in history. Although he died in 1966, his name and artistic
legacy continue to influence the lives of millions of people throughout the world.
After several years of barely making ends meet as a cartoon artist operating from his Los
Angeles garage, Disney had his first success in 1928 with his release of a Mickey Mouse cartoon.
Through the next decade, he continued to produce a number of cartoons, and developed more of
his highly profitable and enduring creations such as Donald Duck and Pluto. In the later 1930s, he
issued the first full-length cartoon film. Snow White became an instant commercial and critical
success. This was only the first of many films, both animated and not, produced by Disney and his
studio. But as renowned as the Disney name is for cartoons and movie, it is probably best known
for a string of spectacular amusement and theme parks. Starting with California’s Disneyland in
48
1955, and culminating with the fantastically successful Disney World and EPCOT Center in Florida,
Disney became a household name. In recent years, the theme park concept has become
international, with openings in Tokyo and Paris. With the continuing success of Disney, the
creations of future theme parks are under discussion.
Question 43: Which of the following is the best title for the passage ?
A. Walt Disney and His Legacy B. The History of Disney World and Disneyland
C. Walt Disney’s Boyhood Years D. Walt Disney and Animated Cartoons
Question 44: The word “humble” in paragraph 1 is closet in meaning to _________
A. extraordinary B. modest C. brave D. professional
Question 45: What is the author’s attitude towards the accomplishments of Walt Disney ?
A. critical B. approving C. ambivalent D. spiteful
Question 46: According the passage, which of the following is TRUE?
A. Disney first achieved success after his death.
B. Mickey Mouse was Disney’s only cartoon creation.
C. Snow White was the first full-length cartoon film.
D. Disney’s first concern was always profitability.
Question 47: The phrase “barely making ends meet” in paragraph 2 is closest in meaning to
_________
A. trying new businesses B. producing only a few cartoons
C. not making much money D. meeting personal failure
Question 48: It can be inferred from the passage that _________
A. Snow White was Disney’s most successful film
B. Disney created cartoon movies and “non-action” movies
C. The Tokyo theme park is in financial difficulty
D. The California theme park is now closed
Question 49: The word “concept” in paragraph 3 is closest in meaning to _________
A. idea B. location C. birth D. demand
Question 50: In future years it is most likely that _________
A. the remaining theme parks will also close B. Disney will produce only cartoons
C. the Disney name will stay well known D. the Paris theme park will become successful
~~THE END~~

PRACTICE TEST 23
Mark the letter A, B, C, or D on your answer sheet to indicate the word whose underlined part
differs from the other three in pronunciation in each of the following questions.
Question 1: A. chamber B. ancient C. danger D. ancestor
Question 2: A. smoothly B. southern C. breath D. airworthy

Mark the letter A, B, C, or D on your answer sheet to indicate the word that differs from the
other three in the position of primary stress in each of the following questions.
Question 3: A. favourable B. allergic C. permanent D. supermarket
Question 4: A. reinforcementB. abnormality C. supplementary D. confidence

Mark the letter A, B, C, or D on your answer sheet to indicate the underlined part that
needs correction in each of the following questions.
Question 5: There are vast, open grazing lands, an area that receives little rain than the farming
49
region west of the Missouri River.
A. west of B. little rain C. vast, open grazing D. an area
Question 6: Many war battles for nation independence were fought in the North than in any other
regions.
A. than B. fought C. Many D. nation independence
Question 7: Not long after Galileo’s time, Sir Isaac Newton invented another kind of telescope
which he used mirrors instead of lenses.
A. instead of B. another kind of C. which he used D. Not long after

Mark the letter A, B, C, or D on your answer sheet to indicate the correct answer to
each of the following questions.
Question 8: During the height of the season, tourists arrive in to see Shakespeare's birthplace.
A. loads B. shoals C. droves D. flocks
Question 9: She set some money each month for her holiday.
A. about B. aside C. back D. up
Question 10: When to explain his mistake, the new employee cleared his throat nervously.
A. asking B. asked C. to be asking D. to be asked
Question 11: In my to win a place at university, I am now under a lot of study pressure.
A. improvement B. support C. confidence D. attempt
Question 12: no money would be wasted, we will use energy more efficiently.
A. so that B. in order that C. in order to D. in order for
Question 13: Somebody who is foolish or stupid is .
A. soft in the head B. over the moon C. riding high D. pushing up daisies
Question 14: Do you think doing the household chores is the of the women only?
A. responsibly B. responsible C. responsibility D.responsive
Question 15: She should in the garage when we came around, which would
explain why she didn't hear the bell.
A. work B. be working C. have worked D. have been working
Question 16: Sarah delivered a(n) appeal to the court and asked for mercy.
A. sentimental B. emotional C. affectionate D. sensational
Question 17: There are several means of mass communication. The newspaper is one. Television
is .
A. another B. other C. the another D. the other
Question 18: “Wild-life-preserves“ are areas where wild animals are within their natural
environment.
A. hunted B. threatened C. protected D. discarded
Question 19: The company fell deeper and deeper into the and then went bankrupt.
A. black B. green C. yellow D. red

Mark the letter A, B, C, or D on your answer sheet to indicate the most suitable
response to complete each of the following exchanges.
th
Question 20: Marx is giving Pam a present on his 18 birthday party.
- Marx: “I have bought you a toy. Happy birthday to you!”
- Pam: “ ”
A. What a lovely toy! Thanks. B. Have a nice day!
C. The same to you! D. What a pity!
Question 21: Tom is talking to Mary.
50
- Tom: “This medicine tastes horrible!”
- Mary: “ , it will cure your cough.”
A. Whatever it tastes B. Come what may
C. How much horrible is it D. Be that as it may

Mark the letter A, B, C, or D on your answer sheet to indicate the word that is CLOSEST
in meaning to the underlined part in each of the following questions.
Question 22: When the protestor entered the meeting clad only in a beach tower, the audience
was dumbfounded.
A. speechless B. excited C. content D. applauding
Question 23: S. Mayo Hospital in New Orleans was so named in recognition of Dr. Mayo’s
outstanding humanitarianism.
A. exhaustive B. producing C. spreading D. collecting

Mark the letter A, B, C, or D on your answer sheet to indicate the word that is
OPPOSITE in meaning to the underlined part in each of the following questions.
Question 24: His creer advancement was slow and he did not gain any promotion until he was
40, when he won the position of the company’s Chief Excutive.
A. progress B. elevation C. rise D. decrease
Question 25: At times, I look at him and wonder what is going on in his mind.
A. Sometimes B. Always C. Hardly D. Never

Mark the letter A, B, C, or D on your answer sheet to indicate the sentence that is
closest in meaning to each of the following questions.
Question 26: “You shouldn’t have leaked our confidential report to the press, Frank!” said Jane.
A. Jane suspected that Frank had leaked their confidential report to the press.
B. Jane criticized Frank for having disclosed their confidential report to the press.
C. Jane accused Frank of having cheated the press with their confidential report.
D. Jane blamed Frank for having flattered the press with their confidential report.
Question 27: It’s easy for Lisa to get tickets for the show.
A. Lisa didn’t have any trouble in getting tickets for the show.
B. Lisa found no difficulty in getting tickets for the show.
C. Lisa didn’t have any trouble in getting tickets for the show as she used to.
D. Lisa had no trouble in getting tickets for the show.
Question 28: I remember giving you a five-pound note.
A. Whether I gave you a five-pound note or not, I can remember.
B. I can't remember whether I gave you a five-pound note or not.
C. I did gave you a five-pound note, and I could remember it.
D. I remember I have given you a five-pound note.

Mark the letter A, B, C, or D on your answer sheet to indicate the sentence that best
combines each pair of sentences in the following questions.
Question 29: My sister worries so much about fitness that she wastes a lot of time and money.
A. My sister wastes a lot of time and money though she worries so much about fitness.
B. My sister worries about fitness so that she wastes a lot of time and money.
C. Worrying too much about fitness, my sister wastes a lot of time and money.
D. Fitness worried, my sister wasted a lot of time and money.
51
Question 30: He joined the army in 1998. He was soon after promoted to the rank of captain.
A. He was promoted to the rank of captain in 1998 and joined the army.
B. Promoted to the rank of captain in 1998, he joined the army soon later.
C. He worked as a captain in the army and had a promotion in 1998.
D. Joining the army in 1998, he was soon promoted to the rank of captain.

Read the following passage and mark the letter A, B, C, or D on your answer sheet to
indicate the correct word or phrase that best fits each of the numbered blanks.
Oxford is a city with such a mind- blowing reputation that many who come here find
themselves intimidated by the place and can’t wait to leave. While others, taking to it like a (31)
_________ to water. Find themselves return again and again, the college lawns provide a gorgeous
backdrop to serious study, and in the light night, on a sunny winter’s morning says, one feels
as if one is (32)_________on air, such is the sense of unreality. Oxford may like to pretendthat it
is at the intellectual hubof things but in many ways, it is no more than a sleepy backwaterwhere to
mix metaphors, transitory students, the creamtheir generation, wait in the wingsallowing their
talents to (33)_________before moving off into the industrial or political fast-lane. Much of this is a
myth, of course. Hardship and hard work are very much part and parcelof student life. The
(34)_________get through the three years’ hard grindby simply putting their shoulders to the
(35)_________ before going on to fairly average jobs. Only for the tiny minority is Oxford the first
step on the ladder to fame and future.
Question 31: A. fish B. duck C. boat D. swimmer
Question 32: A. flying B. gliding C. floating D. swimming
Question 33: A. flourish B. open C. spread D. float
Question 34: A. level- B. hot-headed C. hot-blooded D. kind-hearted
Question
headed 35: A. cart B. wheel C. engine D. boat

Read the following passage and mark the letter A, B, C, or D on your answer sheet to
indicate the correct answer to each of the questions from 36 to 42.
The Winterthur Museum is a collection and a house. There are many museums devoted to the
decorative arts and many house museums, but rarely in the United States is a great collection
displayed in a great country house. Passing through successive generations of a single family,
Winterthur has been a private estate for more than a century. Even after the extensive renovations
made to it between 1929 and 1931, the house remained a family residence. This fact is of
importance to the atmosphere and effect of the museum. The impression of a lived-in house is
apparent to the visitor: the rooms look as if they were vacated only a short while ago whether
by the original owners of the furniture or the most recent residents of the house can be a matter of
personal interpretation. Winterthur remains, then, a house in which a collection of furniture and
architectural elements has been assembled. Like an English country house, it is an organic
structure; the house, as well as the collection and manner of displaying it to the visitor, has
changed over the years. The changes have coincided with developing concepts of the American
arts, increased knowledge on the part of collectors and students, and a progression toward the
achievement of a historical effect in period-room displays. The rooms at Winterthur have followed
this current, yet still retained the character of a private house.
The concept of a period room as a display technique has developed gradually over the years in an
effort to present works of art in a context that would show them to greater effect and would give
them more meaning for the viewer. Comparable to the habitat group in a natural history museum,
the period room represents the decorative arts in a lively and interesting manner and provides an
52
opportunity to assemble objects related by style, date, or place of manufacture.
Question 36: What does the passage mainly discuss?
A. Historical furniture contained in Winterthur
B. How Winterthur compares to English country houses
C. Elements that make Winterthur an unusual museum
D. The reason that Winterthur was redesigned
Question 37: The phrase “devoted to” in line 1 is closest in meaning to .
A. specializing in B. sentimental about C. surrounded by D. successful in
Question 38: What happened at Winterthur between 1929 and 1931?
A. The old furniture was replaced B. The estate became a museum
C. The owners moved out D. The house was repaired
Question 39: What does the author mean by stating “The impression of a lived-in house is
apparent to the visitor”?
A. Few people visit Winterthur
B. The furniture at Winterthur looks comfortable
C. Winterthur does not look like a typical museum
D. Winterthur is very old
Question 40: The word “it” refers to .
A. collection B. English country house C. visitor D. Winterthur
Question 41: According to the passage, objects in a period room are related by all of the following
EXCEPT .
A. place of manufacture B. date C. past ownership D. style
Question 42: What is the relationship between the two paragraphs in the passage?
A. Paragraph 2 explains a philosophy of art appreciation that contrasts with that explained in
Paragraph 1.
B. Paragraph 2 explains a term that was mentioned in Paragraph 1.
C. Each paragraph describes a different historical period
D. Each paragraph describes a different approach to the display of objects in a museum

Read the following passage and mark the letter A, B, C, or D on your answer sheet to
indicate the correct answer to each of the questions from 43 to 50.
Very few people, groups, or governments oppose globalization in its entirety. Instead, critics of
globalization believe aspects of the way globalization operates should be changed. The debate over
globalization is about what the best rules are for governing the global economy so that its
advantages can grow while its problems can be solved.
On one side of this debate are those who stress the benefits of removing barriers to
international trade and investment, allowing capital to be allocated more efficiently and giving
consumers greater freedom of choice. With free-market globalization, investment funds can move
unimpeded from the rich countries to the developing countries. Consumers can benefit from
cheaper products because reduced taxes make goods produced at low cost from faraway places
cheaper to buy. Producers of goods gain by selling to a wider market. More competition keeps
sellers on their toes and allows ideas and new technology to spread and benefit others.
On the other side of the debate are critics who see neo-liberal policies as producing greater
poverty, inequality, social conflict, cultural destruction, and environmental damage. They say that
the most developed nations – the United States, Germany, and Japan – succeeded not because of
free trade but because of protectionism and subsidies. They argue that the more recently successful
economies of South Korea, Taiwan, and China all had strong state-led development strategies that
53
did not follow neo-liberalism. These critics think that government encouragement of “infant
industries” – that is, industries that are just beginning to develop – enables a country to become
internationally competitive.
Furthermore, those who criticize the Washington Consensus suggest that the inflow and
outflow of money from speculative investors must be limited to prevent bubbles. These bubbles are
characterized by the rapid inflow of foreign funds that bid up domestic stock markets and property
values. When the economy cannot sustain such expectation, the bubbles burst as investors panic
and pull their money out of the country.
Protests by what is called the anti-globalization movement are seldom directed against
globalization itself but rather against abuses that harm the rights of workers and the environment.
The question raised by nongovernmental organizations and protesters at WTO and IMF gatherings
is whether globalization will result in a rise of living standards or a race to the bottom as
competition takes the form of lowering living standards and undermining environmental regulations.
st
One of the key problems of the 21 century will be determining to what extent markets should
be regulated to promote fair competition, honest dealing, and fair distribution of public goods on a
global scale.
From “Globalization” by Tabb, William K., Microsoft ® Student 2009 [DVD]
Question 43: It is stated in the passage that .
A. the protests of globalization are directed against globalization itself
B. the United States, Germany, and Japan succeeded in helping infant industries
C. suppoters of globalization stress the benefits of removing trade barriers
D. critics of globalization say that the successful economies are all in Asia
Question 44: Supporters of free-market globalization point out that .
A. consumers can benefit from cheaper products
B. there will be less competition among producers
C. taxes that are paid on goods will be increased
D. investment will be allocated only to rich countries
Question 45: The word “allocated” in the passage mostly means “ ”.
A. removed B. solved C. offered D. distributed
Question 46: The phrase “keeps sellers on their toes” means “ ______”.
A. makes sellers responsive to any changes B. allows sellers to stand on their own feet
C. forces sellers to go bare-footed D. prevents sellers from selling new products
Question 47: According to critics of globalization, several developed countries have become rich
because of .
A. their neo-liberal policies B. their help to developing countries
C. their prevention of bubbles D. their protectionism and subsidies
Question 48: Which of the following is NOT mentioned in the passage?
A. Critics believe the way globalization operates should be changed.
B. The anti-globalization movement was set up to end globalization.
C. Some Asian countries had strong state-led economic strategies.
D. Hardly anyone disapproves of globalization in its entirety.
Question 49: The debate over globalization is about how .
A. to use neo-liberal policies for the benefit of the rich countries
B. to govern the global economy for the benefit of the community
C. to spread ideas and strategies for globalization
D. to terminate globalization in its entirely
Question 50: The author seems to be globalization that helps promote economy and raise
54
living standards globally.
A. supportive of B. indifferent to C. pessimistic about D. opposed to

THE END

PRACTICE TEST 24
Mark the letter A, B, C, or D on your answer sheet to indicate the word whose
underlined part differs from the others.
Question 1: A. coexist B. notify C. prohibit D. frozen
Question 2: A. danger B. devastate C. parade D. rational

Mark the letter A, B, C, or D on your answer sheet to indicate the word that differs from
the others.
Question 3: A. enthusiast B. philosopher C. education D. enlightenment
Question 4: A. develop B. rearrange C. determine D. distinguish
Mark the letter A, B, C, or D on your answer sheet to indicate the underlined part that
needs correction in each of the following questions.
Question 5: Average world temperatures have risen on half a degree Celsius since the
A B C D
mid-nineteenth century.
Question 6: John’s wisdom teeth were troubling him, so he went to a dental surgeon to see
A B
about having them pull.
C D
Question 7: The early periods of aviation in the US was marked by exhibition flights made
A B
by individual fliers or by teams of performers at county fairs.
C D

Mark the letter A, B, C, or D on your answer sheet to indicate the correct answer to
each of the following questions.
Question 8: Alright, Johnny , it’s time you ______ to bed
A. are going to go B. would go C. went D. will be going
Question 9: Tom refuses ___________Mark his address.
A. give B. giving C. to give D. gave
Question 10: My relative, ______ you met yesterday , is a talented pianist
A. that B. whose C. whom D. A & C
Question 11: Your fare, accommodation and meals, are all _________ in the price of this holiday.
A. composed B. collected C. enclosed D. included
Question 12: If you borrow my bike , don’t get it dirty, ______?
A. shall you B. will you C. do you D. can you
Question 13: You can go where you like _________ you come back bore dark.
A. although B. as long as C. or else D. despite
Question 14: I’m sure he wouldn’t mind if we_____ early.
A. arrive B. arriving C. arrived D. had arrived
Question 15: ______ he had to deliver the performance in front of the whole class.
55
A. No sooner had he finished his rehearsal than
B. No sooner than he had his rehearsal finished
C. No sooner than he had finished his rehearsal
D. No sooner he had finished his rehearsal than
Question 16: The living conditions in the rural areas of the country ______ during the past five
years.
A. had been improved B. have been improved C. had improved D.were improved
Question 17: _____colleges and____universities are the main institutions that provide tertiary
education.
A. The / Ø B. Ø / the C. The/the D. Ø / Ø
Question 18: "Those eggs of different colors are so beautiful!" -"Yes, they_____in Russia".
A. were painted B. were paint C. were painting D. painted
Question 19: If you_____ so busy, I would have shown you how to play.
A. aren’t B. weren’t C. hadn’t been D. wouldn’t be

Mark the letter A, B, C, or D on your answer sheet to indicate the most suitable
response to complete each of the following exchanges.
Question 20: Huy and Steve are classmates . They are talking about their sport hobby
Huy: Don’t you like watching football?
Steve: ______________.
A. Yes, I don’t B. No, I do. I like it a lot C. It’s nice D. Yes , I love it
Question 21: This afternoon Hoa is visiting her home town.
Mark: “Don’t fail to send your family my regards”
Hoa: ______________.
A. You’re welcome B. It’s my pleasure C. Good idea, thanks D. Thanks, I will

Mark the letter A, B, C, or D on your answer sheet to indicate the word(s) CLOSEST in
meaning to the underlined word(s) in the following questions.
Question 22: In the world of models, every model girl tries every trick in the book to stop
others from succeeding.
A. every available method B. all kinds of tricks C. all styles of writing D. every story slot
Question 23: The natives were angry when foreigners came to their country and took over their
land
A. tourists B. locals C. members D. migrants

Mark the letter A, B, C, or D on your answer sheet to indicate the word(s) OPPOSITE in
meaning to the underlined word(s) in the following questions.
Question 24: “I made a mess of the exam. But I think I should pass, as I only need 50%.
A. to make a big mistake B. to change your plan
C. to loose confiddence D. to do at one’s best
Question 25: Most universities have trained counselors who can reassure and console students
who have academic or personal problems.
A. discourage B. please C. sympathize D. satisfy

Mark the letter A, B, C, or D on your answer sheet to indicate the sentence that is
closest in meaning to each of the following questions.
Question 26: The challenges facing the new committee leader are numerous .
56
A. There are numerous challenges facing the new committee leader.
B. The new committee leader was ready to take numerous challenges.
C. To succeed, the new committee leader had to face numerous challenges.
D. Numerous challenges are going to face with the new committee leader.
Question 27: It is not until a Vietnamese girl getting 18 years old that she is allowed to get
married legally.
A. A Vietnamese girl is not allowed to get married legally only when she gets18 years old.
B. They never allow a Vietnamese girl to get married legally when she is 18 years old.
C. The legal allowance for a Vietnamese girl to get married will be issued in 18 years.
D. A Vietnamese girl is allowed to get married legally only when she gets 18 years old.
Question 28: “Don’t leave the house until I get back, Jack” said his sister.
A. Jack’sister told him not to leave the house when she got back.
B. Jack’s sister told him to stay at home till she got back .
C. Jack’s sister told him to stay at home when she got back.
D. Jack’s sister told him not to go out until she gets back.

Mark the letter A, B, C, or D on your answer sheet to indicate the sentence that best
combines each pair of sentences in the following questions.
Question 29: The driver in front stopped so suddenly.Therefore , the accident happened
A. If the driver in front didn’t stop so suddenly , the accident wouldn’t happen.
B. If the driver in front hadn’t stopped so suddenly, the accident wouldn’t have happened.
C. If the driver in front hadn’t stopped so suddenly, the accident would have happened.
D. If the driver in front had stopped so suddenly, the accident would have happened.
Question 30: A drug may affect several functions, even though it’s targeted at only one.
A. Despite various other uses, a drug usually has a function for a special effect.
B. A drug is taken for a specific purpose, but it may have a range of other effects.
C. The functions expected of a drug are various even if it is used for a specific disease.
D. However effective a drug may be, its functions have to be several.

Read the following passage and mark the letter A, B, C, or D on your answer sheet to
indicate the correct word or phrase that best fits each of the numbered blanks from 31
to 35.
The current global extinction rate is estimated at about 20,000 species per year, exponentially
greater than the background extinction rate. Many (31) _____ believe that we are in the middle of
the greatest mass
extinction episode since the disappearance of the dinosaurs 65 million years ago.
From what is known about present-day populations and from (32) _____ theory, the change in
either the physical or the biological environment is the key to extinction. However, the vulnerability
of a species depends (33) _____ a wide variety of factors, such as its total population size,
geographical distribution, reproductive ability, ecological relations with other species, and genetic
characteristics. For example, more emphasis is put on the greater vulnerability of species that
reproduce slowly as contrasted with those that reproduce (34) _____. Other factors, such as food-
plant specialization, may make many fast reproducers more vulnerable than species that reproduce
more slowly. No matter (35) _____ fast an insect species that depends on a certain plan can
reproduce; it will still go extinct if that plant's habitat is destroyed.
It has been estimated that about one half billion species have lived at one time or another, and
today's existing species are only 2% of those that have ever evolved. The other 98% have either
died out or evolved into something sufficiently different to be called a new species.
Question 31. A. biology B. biologists C. biological D. biologically

57
Question 32. A.evolve B. evolution C. evolutionary D. evolutionist
Question 33. A. in B. on C. at D. about
Question 34. A. cheerfully B. cheaply C. lastly D. rapidly
Question 35. A. how B. what C. when D. which

Read the following passage and mark the letter A, B, C, or D on your answer sheet to
indicate the correct answer to each of the questions from 36 to 42.
It is hard to get any agreement on the precise meaning of the term "social class". In everyday
life, people tend to have a different approach to those they consider their equals from which they
assume with people they consider higher or lower than themselves in social scale. The criteria we
use to 'place' a new acquaintance, however, are a complex mixture of factors. Dress, way of
speaking, area of residence in a given city or province, education and manners all play a part.
In ancient civilizations, the Sumerian, for example, which flourished in the lower Euphrates valley
from 2000 to 5000 B.C. social differences were based on birth, status or rank, rather than on
wealth. Four main classes were recognized. These were the rulers, the priestly administrators,
the freemen (such as craftsmen, merchants or farmers) and the slaves.
In Greece, after the sixth-century B.C., there was a growing conflict between the peasants and
the aristocrats, and a gradual decrease in the power of the aristocracy when a kind of ‘middle class’
of traders and skilled workers grew up. The population of Athens, for example, was divided into
three main classes which were politically and legally distinct. About one-third of the total population
were slaves, who did not count politically at all, a fact often forgotten by those who praise Athens
as the nursery of democracy.
The next main group consisted of resident foreigners, the, ‘metics’ who were freemen, though
they too were allowed no share in political life. The third group was the powerful body of ‘citizens”,
who were themselves divided into sub-classes.
The medieval feudal system , which flourished in Europe from the ninth to the thirteenth century
, gave rise to a comparatively simple system based on birth. Under the King , there were two main
classes – lords and “vassals” , the latter with many subdivisions.
In the later Middle Ages, however, the development of a money economy and the growth of
cities and trade led to the rise of another class, the ‘burghers’ or city merchants and mayors.
These were the predecessors of the modern middle classes. Gradually high office and occupation
assumed importance in determining social position, as it became more and more possible for a
person born to one station in life to move to another. This change affected the towns more than
the country areas, where remnants of feudalism lasted much longer.
Question 36. According to the passage, we evaluate other people's social position by ________ .
A. questioning them in great details
B. their dress, manners, area of residence and other factors
C. finding out how much their salary is
D. the kind of job they do
Question 37. The four main classes of Sumerian civilization ________.
A. did not include slaves B. took little account of financial standing
C. took little account of status or rank D. were not clearly defined
Question 38. The decline of the Greek aristocracy's power in the sixth century B.C ________ .
A. caused international conflicts in the area
B. lasted for only a short time
C. was assisted by a rise in the number of slaves
D. coincided with the rise of a new "middle class" of traders and peasants
58
Question 39. Athens is often praised as the nursery of democracy ________.
A. even though slaves were allowed to vote
B. because its three main classes were politically and legally distinct.
C. in spite of its heavy dependence on slave labor
D. because even very young children could vote
Question 40. The word "predecessors" in the last paragraph is closest in meaning to:
A. supporters B. authorities C. descendants D. ancestors
Question 41. The word "remnants" in the third paragraph is most likely to correspond to:
A. remains B. opponents C. clothing D. garments
Question 42. The passage is mainly about ________.
A. the human history B. the division of social classes in the ancient world
C. the social life in ancient Greece D. the modern society
Read the following passage and mark the letter A, B, C, or D on your answer sheet to
indicate the correct answer to each of the questions from 43 to 50.
Simply being bilingual doesn’t qualify someone to interpret. Interpreting is not only a
mechanical process of converting one sentence in language A into the same sentence in language
B. Rather, it’s a complex art in which thoughts and idioms that have no obvious counterparts from
tongue to tongue _ or words that have several meanings must be quickly transformed in such a
way that the message is clearly and accurately expressed to the listener.
At one international conference, an American speaker said, “You can’t make a silk purse
out of a sows ear”, which meant nothing to the Spanish audience. The interpretation was, “A
monkey in a silk dress is still a monkey” - an idiom the Spanish understood and that expressed the
same idea.
There are 2 kinds of interpreters, simultaneous and consecutive. The former, sitting in a
separated booth, usually at a large multilingual conference, speaks to listeners wearing
headphones, interpreting what a foreign language speaker says actually a sentence behind.
Consecutive interpreters are the ones most international negotiations use. They are employed for
smaller meetings without sound booths and headphones. Consecutive interpretation also
requires two-person teams. A foreign speaker says his piece while the interpreter, using a
special shorthand, takes notes and during a pause, tells the client
what was said.
Question 43: What is the purpose of the passage?
A. To explain the scope of interpreting
B. To differentiate between simultaneous and consecutive interpreters.
C. To state the qualifications of an interpreter.
D. To point out the importance of an interpreter.
Question 44: The author implies that most people have the opinion that the skill of interpreting is ______.
A. simpler than it really is B. very complex and demanding
C. highly valued and admired D. based on principles of business
Question 45. The example “You can’t make a silk purse out of a sows ear” is used to______.
A. stress the importance of word for word translation
B. point out the difference in attributes of animals in English and Spanish
C. emphasize the need for translation of the meaning of what is said
D. show the differences in language A and language B
Question 46. A precondition of being a translator is ______.
A. being a linguist B. being bilingual
C. working well with people D. being able to use high-tech equipment
59
Question 47. Which of the following would a consecutive interpreter be used for?
A. A large meeting of many nations.
B. A translation of a foreign book.
C. A business transaction between 2 foreign speakers.
D. An interpretation of a major literary work.
Question 48. The word “converting” is closest in meaning to __________
A. understanding B. changing C. reading D. concluding
Question 49. The word “ the former” in the last paragraph refers to_____.
A. simultaneous interpreters. B. the conference
C. consecutive interpreters. D. both A & B.
Question 50. What is a difference mentioned between a simultaneous interpreter and a
consecutive interpreter?
A. The money they are paid. B. The size of group with whom they work.
C. Their proficiency in the language. D. The type of dictionary they use.
~~THE END~~

PRACTICE TEST 25
Mark the letter A, B, C, or D on your answer sheet to indicate the word whose
underlined part differs from the other three in pronunciation in each of the following
questions.
Question 1. A. sweet B. sweat C. beastie D. creature
Question 2. A. neophyte B. geocentric C. leopard D. leotard

Mark the letter A, B, C or D on your answer sheet to indicate the word that differs from
the rest in the position of the main stress in each of the following questions:
Question 3: A. environmentB. satisfy C. imagine D. attractive
Question 4: A. biology B. ability C. interactive D. inaccurate

Mark the letter A, B, C or D on your answer sheet to indicate the underlined part that
needs correction in each of the following questions.
Question 5. John announced that he could not longer tolerate the conditions of the contract under
which he was working.
A. that    B. not longer    C. of    D. under which
Question 6. Mary was determined to leave the office by 4:30 for catching the early train home.
A. to leave     B. by     C. for catching     D. home
Question 7. The little boy’s mother bought him a five – speeds racing bicycle for his birthday.
A. The B. him C. five – speeds D. for

Mark the letter A, B, C or D on your answer shhet to indicate the sentence that best
combines each pair of sentences in the following questions.
Question 8. Jim told me that last year; the company celebrated their tenth anniversary with an
elaborate dinner party that was held in an old castle.
A. Last year, according to Jim, there was an elaborate banquet which took place in an old castle in
honour of the company's tenth anniversary.
B. According to Jim, the ancient castle provides a wonderful place to arrange any special occasion,
such as the party his company held there on their tenth anniversary.
60
C. Jim says that since last year, his company has been preparing for an elaborate banquet in an old
castle to celebrate their tenth anniversay.
D. The company where Jim works held an elaborate party in an old castle last year to celebrate
their tenth anniversary.
Question 9. It isn't just that the level of education of this school is high. It's that it's also been
consistent for years.
A. The level of education in this school, which is usually quite high, shows only slight variations
from year to year.
B. The standard of education is not high in this school, but at least all the students are at the same
level.
C. Not only are the standards of education good in this school, but it has maintained those
standards over the years.
D. It isn't fair to deny that this school is successful, as it has had the same high standards for many
years now.

Mark the letter A, B, C, or D on your answer sheet to indicate the most suitable
response to complete each of the following questions
Question 10. “Excuse me, is anybody sitting here?” – “__________________.”
A. Sorry, the seat is taken.     B. Yes, I am so glad   
C. No, thanks.         D. Yes, yes. You can sit here.
Question 11. - "Do you mind if I use your phone?" - "__________________."
A. Yes, certainly B. Sorry. I have no idea
C. Not at all. Help yourself D. You can say that again

Choose the sentence which is closest in meaning to the given one


Question 12. The writer Kate Millett was forced to stay in a psychiatric hospital by her family.
A.Problems with her family made the writer Kate Millett go to a psychiatric hospital.
B.Together with her family, the writer Kate Millett was made to go into a psychiatric hospital.
C.The writer Kate Millett's family made her stay in a psychiatric hospital.
D.The family of the writer Kate Millett convinced her to remain in a psychiatric hospital.
Question 13. Susan congratulated Jane on having beaten Fiona at the chess tournament.
A. Jane beat Fiona at the chess tournament, and so Susan suggested celebrating this victory.
B. For her victory over Fiona at the chess tournament, Jane was complimented by Susan.
C. Susan was the first to congratulate Jane when she won a victory over Fiona at the chess
tournament.
D. Jane won the chess tournament by finally beating Fiona, so Susan congratulated her for it.
Question 14. In the 1996 presidential election in the US, the majority of Americans abstained
from voting.
A. Most Americans chose not to vote for any presidential candidate in the 1996 election in the US.
B. The US presidential election had to be held again in 1996, as the majority of Americans refused
to vote
C. In the US presidential election of 1996, many Americans decided not to cast a vote.
D. The majority of American voters cast a vote in the US presidential election of 1996.

Find the word which is SYNONYMOUS in meaning with the underlined one:
Question 15. The interviewer spends most of the interview trying to confirm the impression
given by the candidate in the first few moments.
61
A. verify B. misrepresent C. recollect D. conclude
Question 16. Experiments have shown that in selecting personnel for a job, interviewing is at best
a hindrance, and may even cause harm
A. assistance B. encouragement C. procedure D. interference

Find the word which is ANTONYMOUS in meaning with the underlined one:
Question 17. The notice should be put in the most conspicuous place so that all the students
can be well-informed.
A. easily seen B. concealed C. popular D. suspicious
Question 18: In some countries, the disease burden could be prevented through environmental
improvements.
A. something to suffer B. something sad
C. something to entertain D. something enjoyable

Mark the letter A, B, C, or D on your answer sheet to indicate the correct answer to
each of the following questions.
Question 19: It’s a formal occasion so we’ll have to __________ to the nines- no jeans and
pullovers this time!
A. hitch up B. put on C. wear in D. get dressed up
Question 20.  When I bought the shoes, they ___________ me well but later they were too tight
at home
A. matched   B. fitted   C. suited   D. went with
Question 21: Stay out of this problem and do not interfere, please! It’s none of your _______!
A. work B. job C. business D. action
Question 22: There seems to be a large _________ between the number of people employed in
service industries, and those employed in the primary sectors.
A. discriminate B. discretion C. discrepancy D. distinguish
Question 23: ________, human beings have relatively constant body temperature.
A. Alike all mammal B. Alike all mammals C. Like all mammals D. Like all mammal
Question 24: According to a recent survey, ________ doctors do not have a personal physician.
A. a large amount of B. large amount of C. a large number ofD. large number of
Question 25: A washing machine of this type will certainly________normal domestic use.
A. stand up for B. come up with C. get on to D. take down with
Question 26: It’s essential that every student _____ the exam before attending the course.
A. pass B. passes C. would pass D. passed
Question 27: I decided to go to the library as soon as I _______
A. finish what I did B. would finish what I did
C. finished what I did D. finished what I was doing
Question 28: We all agree that she is _____ student in our class.
A. the cleverest B. a more clever C. cleverest D. most clever
Question 29: Michael looked deeply hurt and surprised when _____.
A. scolding B. scolded C. having scolded D. to scold
Question 30: The boat was sailing north when a terrible storm _____.
A. broke B. would break C. had broken D. was breaking

Read the following passage and mark the letter A, B, C, or D on your answer sheet to
indicate the correct word for each of the blanks.
62
Organic farmers also use alternatives (31) ……… pesticides; for example they may rely on
natural predators of certain insect pests. (32) ……… the need arises, they can buy the eggs and
larvae of these natural predators and introduce them into their crop fields. They use (33) …………
techniques to control pests as well, like planting certain crops together because one crop repels the
other's pests. Organic farmers do not need a lot of land;(34) ……… organic farming is perfectly (35)
……… to small farms and is relatively inexpensive. Finally, many organic farmers' average yields
compare favorably with other farmers' yields.
Question 31. A. of B. to C. for D. from
Question 32. A. Then B. If C. Because D. Though
Question 33. A. others B. another C. the others D. other
Question 34. A. instead B. in one way C. on one hand D. in fact
Question 35. A. suitable B. open C. likely D. suited

Read the following passage and mark the letter A, B, C or D on your answer sheet to
indicate the correct answer to each of the questions
The immune system is equal in complexity to the combined intricacies of the brain and nervous
system. The success of the immune system in defending the body relies on a dynamic regulatory
communications network consisting of millions and millions of cells. Organized into sets and
subsets, these cells pass information back and forth like clouds of bees swarming around a hive.
The result is a sensitive system of checks and balances that produces an immune response that is
prompt, appropriate, effective, and self-limiting.
At the heart of the immune system is the ability to distinguish between self and non-self. When
immune defenders encounter cells or organisms carrying foreign or non-self molecules, the immune
troops move quickly to eliminate the intruders. Virtually every body cell carries distinctive molecules
that identify it as self. The body’s immune defenses do not normally attack issues that carry self-
markers. Rather, immune cells and other body cells coexist peaceably in a state known as self-
tolerance. When a normally functioning immune system attacks a non-self molecule, the system has
the ability to remember the specifics of the foreign body. Upon subsequent encounters with the
same species of molecules, the immune system reacts accordingly. With the possible exception of
antibodies passed during lactation, this so-called immune system memory is not inherited. Despite
the occurrence of a virus in your family, your immune system must learn from experience with the
many millions of distinctive non-self molecules in the sea of microbes in which we live. Learning
entails producing the appropriate molecules and cells to match up with and counteract each non-
self invader.
Any substance capable of triggering an immune response is called an antigen. Antigens are not
to be confused with allergens, which are most often harmless substances (such as ragweed pollen
or cat hair) that provoke the immune system to set off the inappropriate and harmful response
known as allergy. An antigen can be a virus, a bacterium, a fungus, a parasite, or even a portion or
product of one of these organisms. Tissues or cells from another individual (except an identical
twin, whose cells carry identical self markers) also act as antigens; because the immune system
recognizes transplanted tissues as foreign, it rejects them. The body will even reject nourishing
proteins unless they are first broken down by the digestive system into their primary, non-antigenic
building blocks. An antigen announces its foreignness by means of intricate and characteristic
shapes called epitopes, which protrude from its surface. Most antigens, even the simplest microbes,
carry different kinds of epitopes on their surface; some may even carry several hundred. Some
epitopes will be more effective than others at stimulating an immune response. Only in abnormal
situations does the immune system wrongly identify self as non-self and execute a misdirected
63
immune attack. The result can be so-called autoimmune disease such as rheumatoid arthritis or
systemic lupus erythematosis. The painful side effects of these diseases are caused by a person’s
immune system actually attacking itself.
Question 36. After you have had the chicken pox, your immune system will be able to do all of the
following EXCEPT __________.
A. prevent your offspring from infection by the chicken pox virus
B. remember previous experiences with the chicken pox virus
C. match up and counteract non-self molecules in the form of the chicken pox virus
D. distinguish between your body cells and that of the chicken pox virus
Question 37. Which of the following best expresses the main idea of this passages ?
A. The human body presents an opportune habitat for microbes.
B. One of the immune system’s primary functions is the allergic response.
C. An antigen is any substance that triggers an immune response.
D. The basic function of the immune system is to distinguish between self and non-self.
Question 38. Why would tissue transplanted from father to daughter have a greater risk of being
detected as foreign than a tissue transplanted between identical twins ?
A. The age of the twins’ tissue would be the same and, therefore, less likely to be rejected.
B. The identical twins’ tissue would carry the same self-markers and would, therefore, be less likely
to be rejected.
C. The twins’ immune systems would remember the same encounters with childhood illnesses.
D. The difference in the sex of the father and daughter would cause the tissue to be rejected by
the daughter’s immune system.
Question 39. What is the meaning of the underlined word ‘ intricacies’ as it is used in the first
sentence of the passage ?
A. elaborate interconnections B. comprehensive coverage
C. inherent perplexity D. confusion of pathways
Question 40. What is the specific term for the substance capable of triggering an inappropriate or
harmful immune response to a harmless substance such as ragweed pollen?
A. microbe B. antigen C. allergen D. autoimmune disease
Question 41. How do the cells in the immune system recognize an antigen as foreign or non-self ?
A. through an allergic response
B. through blood type
C. through fine hairs protruding from the antigen surface
D. through characteristic shapes on the antigen surface
Question 42. The immune cells and other cells in the body coexist peaceably in a state known as
__________.
A. self-tolerance B. harmony C. tolerance D. equilibrium
Question 43. What is the analogy used to describe the communications network among the cells
in the immune system ?
A. A sea of microbes B. The immune system’s memory
C. Bees swarming around a hive D. Immune troops eliminating intruders

Read the following passage and mark the letter A, B, C, or D on your answer sheet to
indicate the correct answer to each of the questions
   Who talk more – men or women? Most people believe that women talk more. However, linguist
Deborah Tannen, who has studied the communication style of men and women , says that this is a
stereotype . According to Tannen, women are more verbal – talk more – in private situations ,
64
where they use conversation as the “glue” to hold relationships together. But, she says, men talk
more in public situations, where they use conversation to exchange information and gain status.
Tannen points out that we can see these differences even in children. Little girls often play with one
“best friend”, their play includes a lot of conversation. Little boys often play games in groups; their
play usually involves more doing than talking. In school, girls are often better at verbal skills, boys
are often better at mathematics.
    A recent study at Emory University helps to shed light on the roots of this difference.
Researchers studied conversation between children age 3-6 and their parents. They found evidence
that parents talk very differently to their son than they do to their daughters. The startling
conclusion was that parents use more language with their girls. Specifically, when parents talk with
their daughters , they use more descriptive language and more details. There is also far more talk
about emotions, especially sadness, with daughters than with sons. 
Question 44. Which sentence best expresses the main idea of the first paragraph?
A. Little girls and little boys have different ways of playing.
B. Women talk more than men.       
C. It’s stereotype that women talk more than men .
D. Women talk more in private , and men talk more in publiC.
Question 45. Which  word  is similar in meaning to  “glue”  in line 3 ?
A. means      B. sticky substance              C.  game                            D.  rope
Question 46. Which sentence best expresses the main idea of the second paragraph?
A. Researchers have studied  the conversations of children and their parents.
B. Parents do not talk much about sadness with their sons.
C. Study at Emory University can help to explain the differences between communication styles of
boys and girls.
D. An Emory University found that parents talk more with their daughters than with their sons.
Question 47. The word they in line 3 refers to
A. situations.                B. women              C. men and women              D. men
Question 48. Which of the following statements can be inferred from the first paragraph ?
A. Men and women have different styles of talking , which may begin in childhood.
B. Men are more sociable than women.
C. According to Tannen, the belief that women talk more is partly right but most wrong and
oversimplified.
D. Women talk more in some situations, men talk more in others.
Question 49. Which word could best replace “startling” ?
A. beginning              B. annoying     C. surprising         D. interesting             
Question 50. Which of the following statements is TRUE about the passage?
A. Parents use more language to talk with their daughters.
B. Boys don’t like showing their emotions.
C. Parents give more love to their daughters than their sons.
D. Girls are thought to be more talkative than boys.
~~THE END~~

PRACTICE TEST 26
Mark the letter A, B, C, or D on your answer sheet to indicate the word whose underlined
part differs from the other three in pronunciation in each of the following questions.
Question 1: A. laughed B. practiced C. awarded D. jumped
65
Question 2: A. compete B. intend C. medal D. defend

Mark the letter A, B, C, or D on your answer sheet to indicate the word that differs from
the other three in the position of primary stress in each of the following questions.
Question 3: A. lucrative B. authentic C. departure D. romantic
Question 4: A. residential B. repetition C. mountaineer D. television

Mark the letter A, B, C, or D on your answer sheet to indicate the underlined part that
needs correction in each of the following questions.
Question 5: Digital clocks, however precise, they cannot be perfectly accurate because the earth’s
rotation changes slightly over years.
A. slightly B. they C. the D. perfectly
Question 6: You shouldn’t criticize him in front of his friends. It was insensitive of you.
A. It B. insensitive C. shouldn’t criticize D. in front of
Question 7: On the floor of the Pacific Ocean is hundreds of flat-topped mountains more than a mile
beneath sea level.
A. hundreds of B. is C. more than D. On

Mark the letter A, B, C, or D on your answer sheet to indicate the correct answer to
each of the following questions.
Question 8: She locked the door of her room all day yesterday to avoid .
A. disturbing B. being disturbed C. to disturb D. to be disturbed
Question 9: For many days on end, authorities of District 1 by vice chairman Doan Ngoc Hai have
aggressively tackled all violations by bringing down illegal structures on the sidewalk to "reclaim
pavement for pedestrians".
A. leads B. leading C. led D. lead
Question 10: I don’t think T-shirts and jeans will ever go of style among young people.
A. down B. away C. off D. out
Question 11: Many people are trying their best to study and work in the hope that they will
fame and fortune in the near future.
A. lose B. collect C. achieve D. remove
Question 12: When he came to the counter to pay, he found that he had cash his
credit card with him.
A. neither/ nor B. either/ or C. both/ and D. not/ neither
Question 13: There’s a law that says you throw trash on the street.
A. might not B. couldn’t C. don’t have to D. must not
Question 14: Despite all the evidence, he wouldn’t admit that he was in the .
A. fault B. error C. wrong D. slip
Question 15: English is used by pilots to ask for landing in Cairo.
A. instructions B. instruct C. instructors D. instructive
Question 16: Her husband bought her when he went on holiday in Singapore last week.
A. a beautiful silk yellow scarf B. a beautiful yellow silky scarf
C. a beautiful yellow scarf silk D. a beautiful yellow silk scarf
Question 17: The teacher told the pupils that salt water at a lower temperature than fresh
water.
A. froze B. would freeze C. freeze D. freezes
Question 18: Learning English isn’t so difficult once you .
A. get on it B. get down to it C. get off it D. get down with it

66
Question 19: I could sit here and argue with you till __________ come home, but it wouldn’t solve
anything.
A. the horses B. the sheep C. the cows D. the chickens

Mark the letter A, B, C, or D on your answer sheet to indicate the most suitable
response to complete each of the following exchanges.
Question 20: Jack is apologizing to his teacher for his being late.
- Jack: “I’m sorry I’m late.”
- Teacher: “ .”
A. Never mind. Go ahead. B. Good to hear that.
C. No worries. Come in, please. D. Don’t say so again.
Question 21: Thomas and Peter are meeting after a long time.
- Thomas: “How have you been recently?”
- Peter: “ ”
A. I am going on holiday next week. B. By car, usually.
C. I am working here. D. Pretty busy, I think.

Mark the letter A, B, C, or D on your answer sheet to indicate the word or phrase that is
CLOSEST in meaning to the underlined part in each of the following questions.
Question 22: “Edwards seems like a dog with two tails this morning”. “Haven’t you heard the
news? His wife gave birth to a baby boy early this morning.”
A. very proud B. very exhausted C. extremely dazed D. extremely pleased
Question 23: The National Institute of Mental Health is conducting far-reaching research to
determine the psychological effects of using drugs.
A. refined B. extensive C. prevalent D. tentative

Mark the letter A, B, C, or D on your answer sheet to indicate the word that is OPPOSITE
in meaning to the underlined part in each of the following questions.
Question 24: The company empowered her to declare their support for the new event. As a matter
of fact, she now has more power to do what she wants than ever before.
A. forbade B. disallowed C. helped D. ordered
Question 25: About 95 percent of all animals are invertebrates which can live anywhere, but most,
like the starfish and crabs, live in the ocean.
A. with ribs B. without ribs C. with backbones D. without backbones

Mark the letter A, B, C, or D on your answer sheet to indicate the sentence that is closest
in meaning to each of the following questions.
Question 26: No sooner had he been appointed to the post than the new director fell ill.
A. Immediately after his appointment to the post, the new director fell ill.
B. The new director had no sooner been appointed to the post when he fell ill.
C. After the new director fell ill, he had been appointed to the new post.
D. Because he had been appointed to the post, the new director fell ill.
Question 27: I’d rather be hungry than eat that food.
A. I am hungry and I want to eat that food.
B. I prefer being hungry to eating that food.
C. I can’t stand being hungry so I’ll eat that food.
D. I am hungry enough to eat that food.
Question 28: “It can't be Mike who leaked the document, it might be Tom.” said our manager.
A. Our manager suspected Tom of having leaked the document, not Mike.
67
B. Our manager blamed Tom for having leaked the document instead of Mike.
C. Our manager showed his uncertainty about who leaked the document: Mike or Tom.
D. Our manager made it clear that Tom was the one who leaked the document, not Mike.

Mark the letter A, B, C, or D on your answer sheet to indicate the sentence that best
combines each pair of sentences in the following questions.
Question 29: The agreement ended six-month negotiation. It was signed yesterday.
A. The agreement which ends six-month negotiation was signed yesterday.
B. The agreement which was signed yesterday lasted six months.
C. The negotiation which lasted six months was signed yesterday.
D. The agreement which was signed yesterday ended six-month negotiation.
Question 30: You’d better take the keys. It’s possible I’ll come home late.
A. You’d better take the keys as I possibly come home late.
B. If I come home late, you’d better take the keys.
C. You’d better take the keys in case I come home late.
D. I’ll probably come home late so that you’d better take the keys.

Read the following passage and mark the letter A, B, C, or D on your answer sheet to
indicate the correct word or phrase that best fits each of the numbered blanks.
A trend that has emerged recently is the sharing of childcare (31)________ between husband and
wife. Young couples will try to arrange their work schedules so that they work opposite hours or shifts
in order that one parent is always home with the children. Since childcare is expensive, this saves
money for the young couple trying to establish themselves and provide a secure environment for the
family. Husband and wife may also share household chores. Some fathers are just as capable as
mothers at cooking dinner, changing and bathing the baby, and doing the laundry.
In some cases, the woman’s salary is for family (32) __________ and the father becomes the
“househusband." These cases are still fairly rare. One positive trend, however, is that fathers seem to
be spending more time with their children. In a recent survey, 41% of the children sampled said they
spend equal time with their mothers and fathers. “This is one of our most significant cultural changes,”
says Dr. Leon Hoffman, who co-directs the Parent Child Center at the New York Psychoanalytic Society.
In practice, for over 30 years, Hoffman has found "a very dramatic difference in the involvement of the
father in everything from care-taking to general decision (33) ________ around kids' lives.”
Another factor has recently been added to the childcare formula. The number of people who work
from home nearly full-time rose 23% from the last decade. The (34) of technology -
computers, faxes, teleconferencing - has made it easier for at-home workers to be constantly in touch.
Will this new flexibility in the workforce bring a positive change for the (35)________ of children? Only
time will tell.
Question 31: A. abilities B. possibilities C. techniques D. responsibilities
Question 32: A. payment B. expenses C. fares D. fees
Question 33: A. making B. creating C. holding D. giving
Question 34: A. accessible B. accessibly C. access D. accessibility
Question 35: A. well-being B. security C. comfort D. interests

Read the following passage and mark the letter A, B, C, or D on your answer sheet to
indicate the correct answer to each of the questions from 36 to 42.
Smart cards and mobile phones are becoming an increasingly popular way to make all sorts of
payments. Even now, in Japan thousands of transactions, from paying rail tickets to picking up the
groceries, take place every day with customers passing their handsets across a small flat-screen
device. And predictions in the world of finance reckon that payments using mobile phones will have
68
risen to more than $50 billion in the very near future.
What's the appeal of e-cash? Compared to cheques or credit cards, it offers the speed of cash,
but more so. It takes just one tenth of a second to complete most transactions and as no change is
required, errors in counting are eliminated. Fraud and theft are also reduced and for the retailer, it
reduces the cost of handling money. Sony's vision of having a chip embedded in computers. TVs and
games consoles means that films, music and games can be paid for easily and without having to input
credit card details.
And what about the future of the banks? Within their grip on the market, banks and credit-card
firms want to be in a position to collect most of the fees from the users of mobile and contactless-
payment systems. But the new system could prove to be a "disruptive technology" as far as the banks
are concerned. If payments for a few coffees, a train ticket and a newspaper are made every day by a
commuter with a mobile, this will not appear on their monthly credit card statements but on their
mobile phone statements. And having spent fortunes on branding, credit-card companies and banks
do not want to see other payment systems gaining popularity. It's too early to say whether banks will
miss out and if so, by how much. However, quite a few American bankers are optimistic They feel
there is reason to be suspicious of those who predict that high-street banks may be a thing of the
past. They point out that Internet banking did not result in the closure of their high-street branches as
was predicted. On the contrary, more Americans than ever are using local branches. So, whether we'll
become a totally cash-free society remains open to contention.
Question 36: What is the main idea of the first paragraph?
A. The absence of traditional payment methods
B. Predictions of future payment methods
C. Japan's advanced forms of payment
D. The increasing popularity of new payment methods
Question 37: Why does the author mention “a small flat-screen device” in the first paragraph?
A. to exemplify the e-cash system B. to criticize the e-cash system
C. to praise the e-cash system D. to inform the e-cash system
Question 38: Which of the following is NOT true about the strong point of e-cash?
A. faster speed B. no fraud C. fewer mistakes D. reduced cost
Question 39: The word “embedded” in the second paragraph is closest in meaning to .
A. manufactured B. isolated C. integrated D. generated
Question 40: The author mentions the case of commuters in the third paragraph to illustrate .
A. the modern technology of the e-cash system
B. a possible drawback of the system
C. the banks' cooperation with credit-card companies
D. the transferability of the system
Question 41: What does the author think may happen in the future?
A. Daily expenses on drinks and tickets will appear on phone statements.
B. Banks will collect their fees through credit-card companies.
C. Americans will no longer go to their local bank branches.
D. Credit-card companies and banks will want to promote cash.
Question 42: How does the writer seem to feel about the future of banks?
A. neutral B. uncertain C. optimistic D. pessimistic

Read the following passage and mark the letter A, B, C, or D on your answer sheet to
indicate the correct answer to each of the questions from 43 to 50.
Cooperation is the common endeavor of two or more people to perform a task or reach a jointly
cherished goal. Like competition and conflict, there are different forms of cooperation, based on
group organization and attitudes.
69
In the first form, known as primary cooperation, group and individual fuse. The group contains
nearly all of each individual’s life. The rewards of the group’s work are shared with each member.
There is an interlocking identity of individual, group and task performed. Means and goals become
one, for cooperation itself is valued.
While primary cooperation is most often characteristic of preliterature societies, secondary
cooperation is characteristic of many modern societies. In secondary cooperation, individuals devote
only part of their lives to the group. Cooperation itself is not a value. Most members of the group feel
loyalty, but the welfare of the group is not the first consideration. Members perform tasks so that they
can separately enjoy the fruits of their cooperation in the form of salary, prestige, or power. Business
offices and professional athletic teams are examples of secondary cooperation.
In the third type, called tertiary cooperation or accommodation, latent conflict underlies the
shared work. The attitudes of the cooperating parties are purely opportunistic: the organization is
loose and fragile. Accommodation involves common means to achieve antagonistic goals: it breaks
down when the common means cease to aid each party in reaching its goals. This is not, strictly
speaking, cooperation at all, and hence the somewhat contradictory term antagonistic cooperation is
sometimes used for this relationship.
Question 43: What is the author’s main purpose in the first paragraph of the passage?
A. To urge readers to cooperate more often
B. To offer a brief definition of cooperation
C. To explain how cooperation differs from competition and conflict
D. To show the importance of group organization and attitudes
Question 44: The word “cherished” in paragraph 1 is closest in meaning to .
A. defined B. agreed on C. prized D. set up
Question 45: The word “fuse” in paragraph 2 is closest in meaning to .
A. explore B. unite C. evolve D. react
Question 46: Which of the following statements about primary cooperation is supported by
information in the passage?
A. It is usually the first stage of cooperation achieved by a group of individuals attempting to
cooperate
B. It is most commonly seen among people who have not yet developed reading and writing skills
C. It is an ideal that can never be achieved
D. It was confined to prehistoric times
Question 47: According to the passage, why do people join groups that practice secondary
cooperation?
A. To experience the satisfaction of cooperation
B. To associate with people who have similar backgrounds
C. To get rewards for themselves
D. To defeat a common enemy
Question 48: Which of the following is an example of the third form of cooperation as it is defined in
the fourth paragraph?
A. Students form a study group so that all of them can improve their grades
B. Members of a farming community share work and the food that they grow
C. Two rival political parties temporarily work together to defeat a third party
D. A new business attempts to take customers away from an established company
Question 49: The word “fragile” in paragraph 4 is closest in meaning to .
A. involuntary B. poorly planned C. inefficient D. easily broken
Question 50: Which of the following best describes the overall organization of the passage?
A. The author provides a number of concrete examples and then draws a conclusion
B. The author presents the points of view of three experts on the same topic
70
C. The author compares and contrasts two types of human relations
D. The author describes a concept by analyzing its three forms
THE END

PRACTICE TEST 27
Read the following passage and mark the letter A, B, C, or D on your answer sheet to
indicate the correct answer to each of the questions.
People travel for a lot of reasons. Some tourists go to see battlefields or religious shrines. Others are
looking for culture, or simply want to have their pictures taken in front of famous places. But most
European tourists are looking for a sunny beach to lie on.
Northern Europeans are willing to pay a lot of money and put up with a lot of inconveniences for the
sun because they have so little of it. Residents of cities like London, Copenhagen, and Amsterdam
spend a lot of their winter in the dark because the days are so short, and much of the rest of the year
in the rain. This is the reason why the Mediterranean has always attracted them. Every summer, more
than 25 million people travel to Mediterranean resorts and beaches for their vacation. They all come
for the same reason, sun!
The huge crowds mean lots of money for the economies of Mediterranean countries. Italy's 30,000
hotels are booked solid every summer. And 13 million people camp out on French beaches, parks and
roadsides. Spain’s long sandy coastline attracts more people than anywhere else. 37 million tourists
visit yearly, or one tourist for every person living in Spain .
But there are signs that the area is getting more tourists than it can handle. The Mediterranean is
already one of the most polluted seas on earth. And with increased tourism, it's getting worse. The
French can’t figure out what to do with all the garbage left by campers around St. Tropez. And in
many places, swimming is dangerous because of pollution.
None of this, however, is spoiling anyone’s fun. The Mediterranean gets more popular every year
with tourists. Obviously, they don’t go there for clean water and solitude. They tolerate traffic jams
and seem to like crowded beaches. They don’t even mind the pollution. No matter how dirty the water
is, the coastline still looks beautiful. And as long as the sun shines, it's still better than sitting in the
cold rain in Berlin, London, or Oslo.
(Adapted from "Modern Sun Worshippers")
Question 1: The latter half of the last sentence in paragraph 3, "or one tourist for every person
living in Spain" means......
A. every person living in Spain has to take care of a tourist annually.
B. every year almost as many tourists visit Spain as there are people living in that country.
C. all the 37 million people living in Spain are tourists.
D. every Spanish is visited by a tourist every year.
Question 2: The word "tolerate" in paragraph 5 is closest in meaning to.......
A. exclude B. reject C. neglect D. endure
Question 3: According to the passage, which of the following countries attracts more tourists than
the others?
A. France B. Greece C. Spain D. Italy
Question 4: In paragraph 2, cities like London, Copenhagen, and Amsterdam are mentioned .......
A. to tell us how wealthy their people are.
B. to show that they are not good cities in terms of geography and climate.
C. to suggest that these cities lack places of historic interest and scenic beauty.
D. to prove that they have got more tourism than they handle.
71
Question 5: According to the passage, which of the following factors might spoil the tourists’ fun at
Mediterranean resorts and beaches?
A. Rainy weather B. Crowded buses C. Traffic jams D. Polluted water
Question 6: The word "solid" in paragraph 3 means most nearly the same as.......
A. seeming to be hard to book B. having no spaces inside
C. having less people than normal D. being uncomfortable to live in
Question 7: The writer seems to imply that Europeans travel mostly for the reason that ......
A. they would like to take pictures in front of famous sites.
B. they want to see historic remains or religious spots.
C. they are interested in different cultural traditions and social customs,
D. they wish to escape from the cold, dark and rainy days back at home.

Mark the letter A, B, C, or D on your answer sheet to indicate the word(s) CLOSEST in
meaning to the underlined word(s) in each of the following questions.
Question 8: ~ "Edwards seems like a dog with two tails this morning."
~ "Haven’t you heard the news? His wife gave birth to a baby boy early this morning."
A. very proud B. extremely pleased C. extremely dazed D. very exhausted
Question 9: Shyness, the most common form of social anxiety, occurs when a person's
apprehensions are so great that they inhibit his making an expected or desired social response.
A. trigger B. defer C. restrain D. discharge

Read the following passage and mark the letter A, B, C, or D on you r answer sheet to
indicate the correct answer to each of the questions.
While some forecasting methods had limited success predicting the 1997 El Nino a few months in
advance, the Columbia University researchers say their method can predict large El Nino events up to
two years in advance. That would be a boon for governments, farmers and others seeking to plan for
the droughts and heavy rainfall El Nino can produce in various parts of the world.
Using a computer, the researchers matched sea-surface temperatures to later El Nino occurrences
between 1980 and 2000 and were then able to anticipate El Nino events dating back to 1857, using
prior sea-surface temperatures. The results were reported in the latest issue of the journal Nature.
The researchers say their method is not perfect, but Biyan Weare, a meteorologist at the University
of California, Davis, who was not involved in the work, said it "suggests El Nino is indeed predictable."
"This will probably convince others to search around more for even better methods," he added.
The new method "makes it possible to predict El Nino at long lead times," said lead author Dake
Chen of Columbia University’s Lamont-Doherty Earth Observatory. "Other models also use sea-surface
temperatures, but they have not looked as far back because they need other data, which is only
available for recent decades," Chen said.
The ability to predict the warming and cooling of the Pacific is of immense importance. The 1997 El
Nino, for example, caused an estimated $20 billion in damage worldwide, offset by beneficial effects in
other areas, said David Anderson, of the European Centre for Medium-Range Weather Forecasts in
Reading, England. The 1877 El Nino, meanwhile, coincided with a failure of the Indian monsoon and a
famine that killed perhaps 40 million in India and China, prompting the development of seasonal
forecasting, Anderson said.
When El Nino hit in 1991 and 1997, 200 million people were affected by flooding in China alone,
according to a 2002 United Nations report. While predicting smaller El Nino events remains tricky, the
ability to predict larger ones should be increased to at least a year if the new method is confirmed,
Anderson wrote in an accompanying commentary. El Nino tends to develop between April and June
72
and reaches its peak between December and February. The warming tends to last between 9 and 12
months and occurs every two to seven years. Chen said the new forecasting method does not predict
any major El Nino events in the next two years, although a weak warming toward the end of this year
is possible.
(Source: http://www.nbcnews.com)
Question 10: The word "tricky" in paragraph 8 is closest in meaning to.......
A. difficult B. normal C. cunning D. common
Question 11: Which of the following can be the best title of the passage?
A. Evidence: Effects of El Nino on human lives B. Study: El Nino can be predicted
C. El Nino: Millions of people threatened with hunger D. Research: How Does the El Nino Develop?
Question 12: The Columbia University researchers studied the relationship between the past El Nino
occurrences and......
A. the droughts and heavy rainfall B. sea-surface temperatures
C. wind direction and ocean current D. some forecasting methods
Question 13: The word "immense" in paragraph 6 is closest in meaning to.......
A. main B. special C. essential D. enormous
Question 14: The word "a boon" in paragraph 1 probably means.......
A. scientific evidence B. living condition C. good news D. new information
Question 15: Which of the following is NOT true according to the passage?
A. The method used by the Columbia University researchers can predict El Nino a few months in
advance.
B. Due to the severe damage El Nino can cause to human life and property, many researchers have
devoted themselves to El Nino-related research.
C. According to a 2002 United Nations report, the flooding in China caused by El Nino in 1991 and
1997 affected 200 million Chinese people.
D. The 1877 El Nino happened at the same time as a failure of the Indian monsoon and a famine
killed perhaps 40 million in India and China.
Question 16: It is stated in the passage that Bryan Weare.......
A. made a contribution to predicting El Nino, which was highly praised by other meteorologists.
B. had set up a special institute in America to study El Nino.
C. worked at the California University, but he used to live at the Columbia University.
D. didn’t do research on El Nino with researchers.
Question 17: According to the passage, it takes about......for El Nino to reach its peak.
A. 8 months B. less than 3 months C. 4 months D. 9 to 12 months

Mark the letter A, B, C, or D on your answer sheet to indicate the word that differs from
the other three in the position of primary stress in each of the following questions.
Question 18:A. brochure B. achieve C. purchase D. teacher
Question 19:A. inherit B. authentic C. elegant D. departure

Mark the letter A, B, C, or D on your answer sheet to indicate the underlined part that
needs correction in each of the following questions.
Question 20: Besides rainfall in the desert is low, it is one of the most important climatic factors in
the formation of desert erosion features.
A. it is B. is low C. Besides D. desert erosion
Question 21: Regardless of the homes from which students come, every one of whom usually has at
least one person who cares.
73
A. from which B. whom C. the homes D. cares
Question 22: Of all seashore plants, seaweeds are best able to tolerate long periods out of water,
followed by long periods covering by water.
A. covering B. best C. Of D. by

Mark the letter A, B, C, or D on your answer sheet to indicate the word whose underlined
part differs from the other three in pronunciation in each of the following questions.
Question 23: A. finished B. pleased C. packed D. punched
Question 24: A. pretty B. president C. protection D. delegate

Mark the letter A, B, C, or D on your answer sheet to indicate the word(s) OPPOSITE in
meaning to the underlined word(s) in each of the following questions.
Question 25: But what most prevents women from reaching the boardroom, say bosses and
headhunters, is lack of hands-on experience of a firm’s core business.
A. impractical B. untested C. unavailable D. insignificant
Question 26: Nowadays, it was rather easy to buy a modern TV, and it does pack a punch to bring
to life some really awesome visuals.
A. prevent something from coming into use B. make bad things happen
C. produce the results that aren’t intended D. have little effect on something

Mark the letter A, B, C, or D on your answer sheet to indicate the sentence that is closest
in meaning to each of the following questions.
Question 27: Alice and Charles did not decide to move to a bigger house until after the birth of their
second child.
A. Only when Alice and Charles had their second child did they decide to move to a bigger house.
B. Only when Alice and Charles had their second child that they decided to move to a bigger house.
C. Only when Alice and Charles had their second child, they decided to move to a bigger house.
D. Only when had Alice and Charles had their second child than they decided to move to a bigger
house
Question 28: The spectators were so excited to see the race that they did not pay so much attention
to that little incident.
A. The spectators paid full attention to the race with excitement, so they didn't realize that little incident.
B. So excited were the spectators to see the race that they didn’t pay so much attention to that little incident.
C. Such excitement were the spectators to see the race that they did not pay attention to that little incident.
D. The spectators did not pay so much attention to that little incident because they were excited to see the
race.
Question 29: Mr. Brown said, "Could you please wait here about half an hour ?"
A. Mr. Brown asked me if I was pleased to wait there about half an hour.
B. Mr. Brown asked me if I had been able to wait there about half an hour.
C. Mr. Brown asked me if I could wait there about half an hour.
D. Mr. Brown asked me to wait there about half an hour.

Mark the letter A, B, C, or D on your answer sheet to indicate the sentence that best
combines each pair of sentences in the following questions.
Question 30: It is raining cats and dogs outside. I want to climb the mountain today but I can't.
A. It is the heavy rain that prevents me from climbing the mountain today.
B. The heavy rain makes it posssible for me to continue going climbing today.

74
C. I find it rather hard to climb the mountain despite a heavy downpour outside,
D. I can’t cany out my mountain climbing because there are many cats and dogs falling out the sky.
Question 31: The most successful candidates are not always the best educated. They are the best
trained in the technique.
A. The most successful candidates are not only the best educated but also the best trained in the technique.
B. The most successful candidates are people who are trained well in the technique, although well educated.
C. The most successful candidates are the best at training in the technique, if not they will need education.
D. The most successful candidates are not always the best educated, but the best trained in the technique.

Read the following passage and m ark the letter A, B, C, or D on your answer sheet to
indicate the correct word or phrase that best fits each of the numbered blanks.
TAKE A LOOK AT THE WORK
"To ...(32)... that students will be successful in school, parents need to look for indicators of school
bonding," said principal Maiy Smith. "Talk of friends, ...(33)... about school, and a sense of belonging
to a class and the school are indicators of that." "They also need to look for signs of academic
engagement," added Smith. By looking at the work their child brings home from school, they can often
see indicators that he or she "is becoming an independent learner and ...(34)... age-appropriate
responsibility for personal success."
"Parents can tell a lot about a teacher’s curriculum by looking at the work their children bring home,"
said Tim Messick. "If teachers present a curriculum ...(35)... has real-life connections and actively
engages kids, and if those kids can articulate at the end of the day some of the exciting learning they
have experienced, parents will be very happy."
"Parents should also expect to see assignments marked and returned promptly," said Chris Rose. "If
it is obvious that only the students are doing the work, it ...(36)... a very negative impression. If that’s
the case - if the parents don’t think the teacher is doing his job - maybe they’re right."
(Adapted from: http://www.educationworld.com)
Question 32: A. ensure B. approve C. insure D. prove
Question 33: A. enthuse B. enthusiastic C. enthusiasm D. enthusiastically
Question 34: A. setting B. assuming C. maintaining D. solving
Question 35: A. that B. whom C. who D. what
Question 36: A. takes B. creates C. draws D. trusts

Mark the letter A, B, C, or D on your answer sheet to indicate the correct answer to each
of the following questions.
Question 37: ......had the curtain been raised than the light went out.
A. Only when B. No sooner C. Hardly D. Scarcely
Question 38: Although the producer is......to the reactions of his audience, he feels he must portray
a 'real' situation.
A. respectful B. enthusiastic C. sarcastic D. sympathetic
Question 39: A washing machine of this type will certainly......normal domestic use.
A. get on to B. stand up to C. take down with D. come up with
Question 40: Don’t park your car before the office building, ......you will have to pay for a fine.
A. unless B. therefore C. because D. or
Question 41: Buy me a newspaper on your way back, .......?
A. will you B. don’t you C. can’t you D. do you
Question 42: I am.......tired to think about that problem now.
A. far too B. very C. nearly D. much more

75
Question 43: ........so aggressive, we'd get on much better.
A. Weren’t she B. Had she not C. If she weren’t D. She was not
Question 44: The teacher told the pupils that salt water......at a lower temperature than fresh water.
A. freeze B. would freeze C. freezes D. froze
Question 45: ......in the atmosphere is the temperature falling below freezing.
A. What produces frost B. Frost produces C. What is frost produced D. Frost is produced
Question 46: The Jackson won’t contribute to the Red Cross unless you.......
A. stand on your own feet B. go down on your knees C. are on your last legs D. keep your head
Question 47: He has declared.......building a new bridge because the budget can’t afford it.
A. against B. about C. in D. to
Question 48: People should not have children unless they are.......to being responsible parents.
A. promised B. committed C. involved D. allowed

Mark the letter A, B, C, or D on your answer sheet to indicate the most suitable response
to complete each of the following exchanges.
Question 49: Ann and Matthew are talking about the effects of global warming .
~ Ann: "Does the global warming worry you?" ~ Matthew: "............."
A. I can’t bear to think about it. B. Oh, it’s hotter and hotter.
C. I don’t like hot weather. D. What a shame!
Question 50: The teacher asked students to discuss the situation :
"Which is better, supermarkets or traditional markets?"
~ Jennifer: "I believe that supermarkets are much better than traditional markets."
~ Katherine: ".......... Each has its own features."
A. I can’t help thinking the same. B. I disagree with you.
C. That’s completely true. D. I couldn’t agree with you more.
THE END

PRACTICE TEST 28
Mark the letter A, B, C, or D on your answer sheet to indicate the word whose underlined
part differs from the other three in pronunciation in each of the following questions.
Question 1: A. crucial B. partial C. material D. financial
Question 2: A. choice B. achieve C. each D. chemistry

Mark the letter A, B, C, or D on your answer sheet to indicate the word that differs from
the other three in the position of primary stress in each of the following questions.
Question 3: A. adventure B. attendance C. opponent D. penalty
Question 4: A. interviewer B. preparation C. economics D. education

Mark the letter A, B, C, or D on your answer sheet to indicate the underlined part that
needs correction in each of the following questions.
Question 5: Higher education in the United States specifically refers to post -secondary institutions,
A B
that offer associate degrees, master degrees or Ph.d. degrees or equivalents.
C D
Question 6: I’m not used to go to school late. I prefer punctuation.
A B C D
76
Question 7: Many young people lack skills, good education, and financial to settle in the urban areas
A B C
where many jobs are found.
D

Mark the letter A, B, C, or D on your answer sheet to indicate the correct answer to each
of the following questions.
Question 8: I am flying to the States tonight. I ______ you a ring if I can find a phone.
A. will give B. would give C. could give D. have given
Question 9: There are two types of higher education in _______ UK: higher general education and
higher vocational education.
A. a B. an C. the D. Ø
Question 10: At first the children enjoyed the game but quite soon _______ novelty.
A. died out B. wore off C. went off D. died out
Question 11: When he graduated __________ university, he had been famous __________ his first
novel.
A. from- for B. for- from C. for- at D. at- for
Question 12: ______ the fifth largest among the night planets that make up our solar system.
A. The Earth is B. The Earth being C. That the Earth is D. Being the Earth
Question 13: ________ but he also proves himself a good athlete.
A. Not only did he show himself a good student B. Not only he showed himself a good student
C. He did not show himself only a good student D. A good student not only showed himself
Question 14: Parents should ___________their children from computer games.
A. discourage B. encourage C. encouragement D. courage
Question 15: After the guests __________, I ___________ to bed.
A. left/went B. had left/went C. had left/had gone D. left/had gone
Question 16: People who are out of work are________.
A. employee B. unemployment C. employed D. unemployed
Question 17: Do you think doing the household chores is the__________of the women only?
A. responsibly B. responsible C. responsibility D. responsive
Question 18: The woman _________ name I don’t remember is a doctor.
A. which B. whom C. who D. whose
Question 19: It’s becoming ______ to find a job.
A. most and more difficult B. more difficult and more
C. more difficult than D. more and more difficult

Mark the letter A, B, C, or D on your answer sheet to indicate the most suitable respone to
complete each of the following exchanges.
Question 20: “I have bought you a toy. Happy birthday to you!” –“__________”
A. What a lovely toy! Thanks. B. Have a nice day! C. The same to you! D. What a pity!
Question 21: Mai: “Wow, I’ve never seen such a nice cell phone, Nam”
Nam: “_______________________”.
A. I agree with you B. Oh, I don’t know
C. You’re welcome D. Thank you. I’m glad you like it.

Mark the letter A, B, C, or D on your answer sheet to indicate the word(s) CLOSEST in
meaning to the underlined word(s) in each of the following questions.
77
Question 22: When the protestor entered the meeting clad only in a beach tower, the audience was
dumbfounded.
A. speechless B. excited C. content D. applauding
Question 23: His new work has enjoyed a very good review from critics and readers.
A. opinion B. viewing C. look D. regard

Mark the letter A, B, C, or D on your answer sheet to indicate the word(s) OPPOSITE in
meaning to the underlined word(s) in each of the following questions.
Question 24: The motorist felt that the ticket for infraction was unwarranted.
A. conscientious B. inadvertent C. inevitable D. justified
Question 25: On November 25 1972, something dreadful happened on board of the brigantine Mary
Celeste, causing all crew members to hastily abandon the ship.
A. hold on B. stay on C. take care of D. save for
Mark the letter A, B, C, or D on your answer sheet to indicate the sentence that is closest
in meaning to each of the following questions.
Question 26: She has played the piano for 5 years.
A. She didn’t play the piano 5 years ago. B. She started playing the piano 5 years ago.
C. She played the piano 5 years ago. D. The last time she played the piano was 5 years ago.
Question 27: He got a terrible punishment from his father.
A. His father was got a terrible punishment from him. B. He was terribly punished by his father.
C. He punished terribly from his father. D. His father was got a terrible punishment by him.
Question 28: The mother asked her son, "Why didn't you do the exercises?"
A. The mother asked her son why he had not done the exercises.
B. The mother asked her son why he did not do the exercises.
C. The mother asked her son why hadn’t he done the exercises.
D. The mother asked her son why didn't he do the exercises.

Mark the letter A, B, C, or D on your answer sheet to indicate the sentence that best
combines each pair of sentences in the following questions.
Question 29: Sue lives in the house. The house is opposite my house.
A. Sue lives in the house which is opposite my house.
B. Sue lives in the house who is opposite my house.
C. Sue lives in the house where is opposite my house.
D. Sue lives in the house and which is opposite my house.
Question 30: They arrived late, so they didn’t have gotten good seats.
A. If they had arrived earlier, they might have gotten good seats.
B. The late arrivers still had good seats.
C. However late they arrived, they had very good seats.
D. Unless they arrived early, they wouldn’t have good seats

Read the following passage and mark the letter A, B, C, or D on your answer sheet to
indicate the correct word or phrase that best fits each of the numbered blanks.
The University of Oxford, informally called "Oxford University", or simply
"Oxford", (31) ______ in the city of Oxford, in England, is (32) ______ oldest university in the English-
speaking world. It is also considered as one of the world's leading (33) ______ institutions. The
university traces, its roots back to at least the end of the 11th century, (34) ______ the exact date of
foundation remains unclear. Academically, Oxford is consistently ranked in the world's top ten
78
universities. The University is also open (35) ______ overseas students, primarily from American
universities, who may (36) _____ in study abroad programs during the summer months for more than
a century, it has served as the home of the Rhodes Scholarship, (37) ______ brings highly
accomplished students from a number of countries to study at Oxford as (38) ______ The University
of Oxford is also a place where many talented leaders from all over the world used to study. Twenty-
five British Prime Ministers attended Oxford, including Margaret Thatcher and Tony Blair. At (39)
______ 25 other international leaders have been educated at Oxford, and this number includes King
Harald V of Norway and King Abdullah II of Jordan. Bill Clinton is the first American President to attend
Oxford. Forty-seven Nobel (40) __ winners have studied or taught at Oxford.
Question 31. A. put B. placed C. located D. stood
Question 32. A. a B. an C. the D. Ø
Question 33. A. learning B. academic C. graduating D. scholar
Question 34. A. although B. because C. since D. if
Question 35. A. to B. for C. from D. up
Question 36. A. write B. name C. enroll D. require
Question 37. A. that B. where C. whose D. which
Question 38. A. postgraduates B. postgraduated C. postgraduation D. postgraduating
Question 39. A. last B least C. late D. lately
Question 40. A. present B. gift C. medal D. prize

Read the following passage and mark the letter A, B, C, or D on your answer sheet to
indicate the correct answer to each of the questions.
From our start in 1961, WWF has worked to protect endangered species. We're ensuring that
the world our children inherit will be home to elephants, tigers, giant pandas, whales and other wildlife
species, as well as people.
WWF safeguards hundreds of species around the world, but we focus special attention on our
flagship species: giant pandas, tigers, polar bears, endangered whales and dolphins, rhinos, elephants,
marine turtles and great apes. These species not only need special measures and extra protection in
order to survive, they also serve as umbrella species: helping them helps numerous other species that
live in the same habitats.
WWF and its partners have a number of projects around the world to reduce Human-Wildlife
Conflict and improve the livelihoods of the people affected.
In addition to our flagship animals, we work to protect numerous species in peril around the
world that live within our priority ecoregions. Large predators like snow leopards and grizzly bears,
migratory species like whooping cranes and songbirds, and a host of other species facing threats also
benefit from WWF's conservation efforts. Our wildlife trade experts at TRAFFIC work to ensure that
trade in wildlife products doesn't harm a species, while also fighting against illegal and unsustainable
trade contact your nearest regional centre.
Question 41. WWF is an. international.........................................................
A. society B. organization C. group D. union
Question 42. WWF focusses special attention on…………………………
A, all animals everywhere B. giant pandas and endangered species
C. wildlife in general D. flagship species
Question 43. WWF and its parteners also carry out some projects to……….
A. cut down human- wildlife conflicts and improve livelihoods of people affected.
B. increase human- wildlife conflicts and improve livelihoods of people affected.
C. help human- wildlife conflicts and improve livelihoods of people affected.
79
D. react human- wildlife conflicts and improve livelihoods of people affected.
Question 44. WWF protects not only flagship animals but also…………..
A. all other species in peril B. endandered species
C. wildlife D. some other species
Question 45. Wildlife trade experts are sure ………………………………
A. not to do any harm to a species. B. to do harm to a species.
C. to do illegal and sustainable trade. D. to do legal and unsustainable trade.

Read the following passage and mark the letter A, B, C, or D on your answer sheet to
indicate the correct answer to each of the questions.
How can scientists predict earthquakes? Earthquakes are not scattered anywhere but happen in
certain areas. They happen in places where pieces of the earth’s surface meet. This pattern causes
them to shake the same places many times. Foe example, earthquakes often occur on the west-coast
of North and South America, around the Mediterranean Sea, and along the Pacific coast of Asia.
Another way tp predict earthquakes is to look for changes in the earth’s surface, like a sudden
drop of water level in the ground. Some people say animals can predict earthquakes. Before
earthquakes people have been chickens sitting in the trees, fish jumping out of the water, snakes
leaving their holes and other animals acting strangely.
After an earthquake happens, people can die from lack of food , water, and medical supplies.
The amount of destruction caused by an earthquake depends on where it happens, what time it
happens, and how strong it is. It also depends on types of building, soil conditions and population. Of
the 6000 earthquakes in the earth each year, only about fifteen cause great damage and many deaths.
Question 46: The passage mainly discusses .........................
A. strange animal behaviors B. how to predict earthquakes
C. the damage caused by earthquakes D. how strong earthquakes are
Question 47: Earthquakes happen in certain areas where....................
A. many buildings are built B. the soil conditions are stable
C. pieces of the earth’s surface meet. D. the population is large
Question 48: Earthquakes often happen along..............................
A. the east-coast of South America B. the Pacific coast of Asia
C. the coast of Australia D. the east-coast of North America.
Question 49: After an earthquake, as a result of ......................... people can die.
A. lack of food B. lack of information C. lack of knowledge D. lack of friendship
Question 50: Looking for .......................... can help predict earthquakes.
A. water currents B. changes in the earth’s surface
C. water beneath the earth’s surface D. drops of water
THE END

PRACTICE TEST 29
Mark the letter A, B, C, or D on your answer sheet to indicate the word whose underlined
part differs from the other three in pronunciation in each of the following questions.
Question 1: A. trained B. performed C. proved D. walked
Question 2: A. education B. woman C. rain D. airplane

Mark the letter A, B, C, or D on your answer sheet to indicate the word that differs from
the other three in the position of primary stress in each of the following questions.

80
Question 3: A. mathematics B. engineering C. scientific D. laboratory
Question 4: A. certificate B. necessary C. economy D. geography

Mark the letter A, B, C, or D on your answer sheet to indicate the underlined part that
needs correction in each of the following questions.
Question 5: The boss laughed when the secretary has told him that she really needed a pay rise.
A B C D
Question 6: My friends didn’t drink any beer since we came to live here.
A B C D
Question 7: The guitarist about who I told you yesterday is coming here tomorrow
A B C D

Mark the letter A, B, C, or D on your answer sheet to indicate the correct answer to each
of the following questions.
Question 8: He ___________ very quickly when I met him yesterday.
A. was walking B. would walk C. had walked D. has walked
Question 9: _______ colleges and _______ universities are the main institutions that provide tertiary
education.
A. The / Ø B. Ø / the C. The / the D. Ø / Ø
Question 10: ______________ in Shanghai than in any other city in China.
A. More living people B. More people live
C. More people living D. It has more people
Question 11: Nam wanted to know what time _________ .
A. the movie began C. the movie begins
B. did the movie begin D. does the movie begin
Question 12: Last night a tornado swept through Rockvill. It ____________ everything in its path.
A. destroyed B. was destroyed C. was being destroyed D. had been destroyed
Question 13: He is very sucesssful _____ his job.
A. for B. with C. in D. of
Question 14: High school students should be ______ for their future jobs before leaving school.
A. ill-spoken B. well-prepared C. ill-prepared D. well-spoken
Question 15: Could you fill out this ______ form?
A. application B. applicable C. applicant D. applying
Question 16: My father decided to ______ smoking after he had been smoking for ten years.
A. take up B. get over C. give up D. put away
Question 17: Your house needs ________ before the new year.
A. paint B. painting C. to paint D. painted
Question 18: The harder you learn,___________
A. the more is your English B. the gooder your English will be
C. the better becomes your English D. the better your English will be
Question 19: All that rubbish will have to be ............. at once.
A. get rid of B. got rid of C. gets rid of D. getting rid of

Mark the letter A, B, C, or D on your answer sheet to indicate the most suitable respone to
complete each of the following exchanges.
Question 20: Maria: "Thanks for the lovely evening." Diana: "______."
A. No, it's not good B. Oh, that's right
C. Yes, it's really great D. I'm glad you enjoyed it
81
Question 21: Pat: "Would you like something to eat?" Kathy: "______. I'm not hungry now."
A. Yes, I would B. No, thanks C. No, no problem D. Yes, it is

Mark the letter A, B, C, or D on your answer sheet to indicate the word(s) CLOSEST in
meaning to the underlined word(s) in each of the following questions.
Question 22: - Because of the fog,the flight was delayed for several hours.
A. began B. started C. held up D. continued
Question 23: -At haft past six, the alarm clock went off
A. came B. rang C. disappeared D. exploded

Mark the letter A, B, C, or D on your answer sheet to indicate the word(s) OPPOSITE in
meaning to the underlined word(s) in each of the following questions.
Question 24: The consequences of the typhoon were disastrous due to the lack of precautionary measures
A. physical B. severe C. beneficial D. damaging
Question 25: Vietnam’s admission to the World Trade Organization (WTO) has promoted its trade
relations with other countries.
A. balanced B. restricted C. expanded D. boosted

Mark the letter A, B, C, or D on your answer sheet to indicate the sentence that is closest
in meaning to each of the following questions.
Question 26: My sister is often sick because she doesn’t do physical exercise.
A. If my sister does physical exercise, she won’t often be sick.
B. If my sister isn’t physical exercise, she does sick.
C. If my sister did physical exercise, she wouldn’t often be sick.
D. If my sister wasn’t physical exercise, she would do sick.
Question 27: "Don't forget to go to the supermarket after work", he said.
A. He requested me not forget to go to the supermarket after work.
B. He asked me not to forget to go to the supermarket after work.
C. He reminded me to go to the supermarket after work.
D. He told me that I shouldn't forget to go to the supermarket after work.
Question 28: Without transportation, our modern society could not exist.
A. Our modern society could not exist if there is no transportation.
B. Our modern society will not exist without having transportation.
C. If there were no transportation, our modern society would not exist.
D. If transportation no longer exists, our modern society will not exist.

Mark the letter A, B, C, or D on your answ er sheet to indicate the sentence that best
combines each pair of sentences in the following questions.
Question 29: I don't have enough money with me now; otherwise I would buy that coat.
A. If I didn't have enough money now, I wouldn't buy that coat.
B. If I didn't have enough money with me now, I would buy that coat.
C. If I had enough money with me now, I would buy that coat.
D. If I had enough money with me now, I wouldn't buy that coat.
Question 30: He is not flexible in his work; that is why he doesn't have many friends.
A. Such is his inflexibility in his work that he doesn't have many friends.
B. Inflexibility in his work is not why he doesn't have many friends.
C. Having almost no friends makes him inflexible in his work.
D. He is not popular with many people although he is flexible in his work.

82
Read the following passage and mark the letter A, B, C, or D on your answer sheet to
indicate the correct word or phrase that best fits each of the numbered blanks.
The United State has many different types of families. While most American families are
traditional, (31) ____ a father, mother and one or more children, 22 percent of all American families in
1980 were dead by one parent, usually a woman. In a few families in the United States, there are no
children. These (32)____ couples may believe that they would not make good parents, they may want
freedom the responsibilities of childrearing; or, perhaps they (33) _____ physically able to have
children. Other families in the United States have one adult (34)____ a stepparent. A stepmother or
step father is a person who joins a family by marrying a father or mother.
Americans tolerate and accept these different types of families. In the United States, people have
the right to privacy and Americans do not believe in telling other Americans what types of family group
they must belong to. They respect each other's choices regarding family groups. Families are very
important (35)____ Americans.
Question 31: A. consisting of B. consisting C. consist of D. in concluding of
Question 32: A. childlike B. childish C. childless D. childhood
Question 33: A. weren't B. can't C. are not D. couldn't
Question 34: A. who is B. is C. which has D. which is
Question 35: A. to B. of C. for D. in

Read the following passage and mark the letter A, B, C, or D on your answer sheet to
indicate the correct answer to each of the questions.
Education is another area of social life in which information technology is changing the way we
communicate. Today's college students may not simple sit in a lecture or a library to learn about their
field. Through their computers and the wonders of virtual reality they can participate in lifelike
simulated experiences. Consider the following scenario of the future of education made possible
through developments in information technology.
For children over the age of 10, daily attendance at schools is not compulsory. Some of the
older children attend school only once or twice a weekly to get tutorial support or instruction from a
teacher. For the most part, pupils are encouraged to work online from home. Students should
complete a minimum number of study hour per year; however, they may make up these hours by
studying at home at times that suit their family schedule. They can log on early or late in the day and
even join live classes in other countries. In order to ensure that each student is learning adequately,
computer software will automatically monitor the number hours a week each student studies on line as
well as that student's learning materials and assessment activities. Reports will be available for parents
and teachers. The software can then identify the best learning activities and conditions for each
individual student and generate similar activities. It can be also identify areas of week achievement
and produce special programs adjusted to the students' needs.
Question 36: What is the topic of the passage?
A. Students don't have to go to school any more
B. The effect of information technology on education
C. Students can know about their weak aspects to focus
D. Computer software will make sure students learn at home
Question 37: How many times are children who are older than 10 required to go to school weekly?
A. Three B. No time C. Once or twice D. Four
Question 38: Who/what counts the number of hours per week that students spend learning?
A. Teacher B. Parents C. Virtual reality D. Computers
Question 39: What can't the software do?
A. Find out the best activities for the students B. Design materials for the students
83
C. Identify weaknesses of the students D. Monitor the time the students learn
Question 40: What is NOT MENTIONED as a benefit of information technology to the students?
A. Students can learn at times that suit their schedule B. Students can stay at home to learn
C. Students' learning time won't be monitored D. Students' weak achievement can be identified.
Read the following passage and mark the letter A, B, C, or D on your answer sheet to
indicate the correct answer to each of the questions.
The food we eat seems to have profound effects on our health. Although science has made enormous
steps in making food more fit to eat, it has, at the same time, made many foods unfit to eat. Some
research has shown that perhaps eighty percent of all human illnesses are related to diet and forty
percent of cancer is related to the diet as well, especially cancer of the colon. People of different
cultures are more prone to contact certain illnesses because of the characteristic foods they consume.
That food is related to illness is not a new discovery. In 1945, government researchers realized that
nitrates nitrites (commonly used to preserve color in meat) as well as other food additives caused
cancer. Yet, these carcinogenic additives remain in our food, and it becomes more difficult all the
time to know which ingredients on the packaging label of processed food are helpful or harmful.
The additives that we eat are not all so direct. Farmers often give penicillin to cattle and poultry, and
because of this, penicillin has been found in the milk of treated cows.
Sometimes similar drugs are administered to animals not for medical purposes, but for financial
reasons. The farmers are simply trying to fatten the animals in order to obtain a higher price on the
market. Although the Food and Drug Administration (FDA) has tried repeatedly to control these
procedures, the practices continue.
A healthy diet is directly related to good health. Often we are unaware of detrimental substances we
ingest. Sometimes well-meaning farmers or others who do not realize the consequences add these
substances to food without our knowledge.
Question 41: How has science done to disservice to people?
A. As a result of scientific intervention, some potentially harmful substances have been added to our food
B. The scientists have preserved the color of meats, but not of vegetables
C. It caused a lack of information concerning the value of food
D. Because of science, disease caused by contaminated food has been virtually eradicated.
Question 42: The word “prone” is nearest meaning to_________.
A. healthy B. unlikely C. supine D. predisposed
Question 43: The word “carcinogenic” is closest in meaning to_________.
A. trouble-making B. money-making C. cancer-causing D. colorretaining
Question 44: What are nitrates used for?
A. They preserve the color of meat B. They preserve flavor in package food
C. There are objects of research D. They cause the animals become fatter
Question 45: FDA means____________.
A. Federal Dairy Additives B. Food and Drug Administration
C. Final Difficult Analysis D. Food Direct Additives
Question 46: All of the following statements are TRUE except
A. Drug are always given to animals for medical reasons
B. Food may cause forty percent of the cancer in the world
C. Researchers have known about the potential hazard of food additives for more than 45 years
D. Some of the additives in our food are added to the food itself and some are given to the living animals
Question 47: What is best title for this passage?
A. The food you eat can affect your health B. Harmful and Harmless substances in food
C. Avoiding injurious substances in food D. Improving health through a Natural Diet
Question 48: The word “fit” could be best replaced by which of the following?
84
A. suitable B. tasty C. athletic D. adaptable
Question 49: The word “these” refers to___________?
A. researchers B. nitrates and nitrites C. meats D. colors
Question 50: The word “additives” is closest meaning to__________.
A. begin substance B. natural substance C. dangerous substance D. added substance
THE END
PRACTICE TEST 30
Mark the letter A, B, C, or D on your answer sheet to indicate the word that differs from
the rest in the position of the main stress in each of the following questions.
Question 1:A. entertainment B. manufacture C. recommendation D. supplementary
Question 2:A. generosity B. international C. isolation D. facilities
Mark the letter A, B, C, or D on your answer sheet to indicate the word whose underlined
part ispronounced differently from that of the rest in each of the following questions.
Question 3:A. catastrophe B. become C. propose D. survive
Question 4:A. pleasure B. conclusion C. measure D. comparison
Mark the letter A, B, C, or D on your answer sheet to indicate the sentence that best
combines each pair of sentences in the following questions.
Question 5: Her husband died. When she heard the news, she fainted.
A. On hearing the news of her husband’s death, she fainted.
B. When she heard the deadly news of her husband, she fainted.
C. When hear the news from her dead husband, she fainted.
D. On hearing the news of her dead husband, she fainted.
Question 6: He cannot practice scuba diving. He has a weak heart.
A. The reason why he cannot practice scuba diving is that he has a weak heart.
B. The reason why he cannot practice scuba diving is because he has a weak heart.
C. Scuba diving makes him suffer from having a weak heart.
D. The fact that he has a weak heart cannot stop him from practicing scuba diving.
Read the following passage and mark the letter A, B, C, or D on your answer sheet to
indicate the correct answer to each of the following questions.
During the nineteenth century, women in the United States organized and participated in a large
number of reform movements, including movements to reorganize the prison system, improve
education, ban the sale of alcohol, grant rights to people who were denied them, and, most
importantly, free slaves. Some women saw similarities in the social status of women and slaves.
Woman like Elizabeth Cady Stanton and Lucy Stone were feminists and abolitionists who supported
the rights of both women and blacks. A number of male abolitionists who supported the rights of both
women and blacks. A number of male abolitionists, including William Lloyd Garnison and Wendell
Phillips, also supported the rights of women to speak and to participate equally with men in anti-
slavery activities. Probably more than any other movement abolitionism offered women a previously
denied entry into politics. They became involved primarily in order to better their living conditions and
improve the conditions of others.
When the civil war ended in 1865, the Fourteenth and Fifteenth Amendments to the Constitution
adopted in 1868 and 1870 granted citizenship and suffrage to blacks but not to women. Discouraged
but resolved, feminists worked tirelessly to influence more and more women to demand the right to
vote. In 1869, the Wyoming Territory had yielded to demiulds by feminists, but the states on the Fast
Coast resisted more stubbornly than before. A women’s suffrage bill had been presented to every
Congress since 1878, but it continually failed to pass until 1920, when the Nineteenth Amendment
granted women the right to vote.
85
Question 7: With what topic is the passage primarily concerned?
A. Women’s suffrage. B. Abolitionists.
C. The Wyoming Territory. D. The Fourteenth and Fifteenth Amendments.
Question 8: The word "it" in line 16 refers to.......
A. Congress B. Nineteenth Amendment C. bill D. vote
Question 9: According to the passage, why did women become active in politics ?
A. To support Elizabeth Cady Stanton for president.
B. To be elected to public office.
C. To improve the conditions of life that existed at the time.
D. To amend the Declaration of Independence.
Question 10: The word "suffrage" in line 12 could best be replaced by which of the following ?
A. freedom from bondage B. citizenship C. pain D. the right to vote
Question 11: What does the Nineteenth Amendment guarantee?
A. Citizenship for women B. Citizenship for blacks C. Voting rights for women D. Voting rights for blacks
Question 12: What had occurred shortly after the Civil War?
A. The eastern states resisted the end of the war.
B. A women’s suffrage bill was introduced in Congress.
C. Black people were granted the right to vote.
D. The Wyoming Territory was admitted to the Union.
Question 13: When were women allowed to vote throughout the United States ?
A. after 1878 B. after 1866 C. after 1920 D. after 1870
Question 14: The word "ban" in line 3 most nearly means to........
A. publish B. prohibit C. limit D. encourage

Mark the letter A, B, C, D on your answer sheet to indicate the correct answer to each of
the following questions.
Question 15: The number of homeless people after the flood........dramatically.
A. had increased B. are increasing C. increase D. has increased
Question 16: Neither of them offered to help you, ........?
A. did it B. did they C. did you D. didn’t you
Question 17: As I was walking along the street, I saw.....$100 note on......pavement
A. a - the B. the - a C. a - a D. the - the
Question 18: I think my car has been stolen. .....the police.
A. am not allowed to phone B. am supposed to phone
C. should be phoned D. had better phone
Question 19: ......the alarm go off, please leave the building at once.
A. Should B. If C. As D. Unless
Question 20: By the end of the week, you must decide.......or not you want to apply for this post.
A. if B. otherwise C. whether D. rather
Question 21: Teaching small children requires.......patience.
A. a lot B. a few C. a number of D. a great deal of
Question 22: The weather is getting.......due to the effects of global warming.
A. more and more badly B. worse and worse C. less and less D. belter and better
Question 23: HCR is one of the biggest international and......caring for refugees.
A. agencies B. federations C. groups D. societies
Question 24: He talked as if he.......where she was.
A. is known B. know C. had known D. would know
86
Question 25: ........after the World War II, the United Nations has been actively carrying out its
convention to stop wars and bring peace to nations worldwide.
A. Having established B. To be established C. To establish D. Established
Question 26: We finally.......at eight o’clock in the morning.
A. put out B. set off C. took after D. took care of

Mark the letter A, B, C, or D on your answer sheet to indicate the word(s) that is CLOSEST
in meaning to the underlined word(s) in each of the following questions.
Question 27: David Beckham is one of the celebrities who are still popular after their retirement,
A. individuals B. rich people C. bachelors D. famous people
Question 28: The integration of several schools has decreased the number of academic options in
our community.
A. disunion B. division C. separation D. combination

Mark the letter A, B, C, or D on your answer sheet to indicate the most suitable response
to complete each of the following exchanges.
Question 29: "Oh no, I’m late for my appointment. ............."
A. Pleased to meet you. B. Catch you later. C. Nice to see you again. D. Catch you now.
Question 30: ~ Nam: "Do you have the time please?" ~ Lan: "............"
A. I am busy right now. I am afraid, B. I am sorry I don’t have a watch here.
C. Yes, I have some time for you. D. I am very sorry. I am already late.

Mark the letter A, B, C, or D on your answer sheet to indicate the word(s) that is
OPPOSITE in meaning to the underlined word(s) in each of the following questions.
Question 31: He was way behind with his rent. As a result, he was evicted from his apartment.
A. ejected B. kicked out C. admitted D. dismissed
Question 32: We will find a temporary shelter for the victims of the fire until they have a place.
A. permanent B. brief C. unstable D. shortly

Mark the letter A, B, C, D on your answer sheet to indicate the underlined part that needs
correction in each of the following questions.
Question 33: There are many benefits that we may be gained by actually taking the time to read a
book instead of sitting in front of the TV or doing some other forms of mindless entertainment.
A. actually B. or C. may be gained D. instead of
Question 34: The purpose of traveller’s checks is to protect travellers from theft and accidental lost
of money.
A. lost B. of C. theft D. is
Question 35: It is educational that helps enrich the mind and widens our knowledge.
A. the B. knowledge C. enrich D. educational

Read the following passage and choose the correct word that best fits each of the
numbered blanks.
Harnessing the wind is one of the cleanest, most sustainable ways to ...(36)... electricity. Wind
power produces no toxic emissions and none of the heat-trapping emissions that contribute ...(37)...
global warming. This, and the fact that wind power is one of the most abundant and ...(38)... cost-
competitive energy resources, makes it a viable ...(39)... to the fossil fuels that harm our health and

87
threaten the environment. ...(40)... its many benefits and significantly reduced costs, wind power is
poised to play a major role as we move towards a sustainable energy future.
Question 36:A. generate B. control C. direct D. manufacture
Question 37:A. to B. into C. on D. for
Question 38:A. increasingly B. increased C. increasing D. increase
Question 39:A. difference B. reference C. alternative D. solution
Question 40:A. Despite B. As for C. Because of D. Thanks to
Read the following passage and mark the letter A, B, C, or D on your answer sheet to
indicate the correct answer to each of the following questions.
With the advent of the computer age, it was [rashly] predicted by computer manufacturers that
technology would help to save trees and create a paperless office. Unfortunately, far from cutting
down on the amount of paper used, we now used 10% more paper than we did before, a lot of which
is used for printing out web pages and emails.
Despite all this, computer companies are still msisting that conventional books will soon disappear,
to be replaced by the handheld e-book. For one particular version of the e-book available at the
moment, you hold your laptop like a book and read the book that you have bought and downloaded
over the internet. It is very doubtful, however, whether this means that the ordinary reader is going to
give up using books made of paper easily. Trying to read an e-book in bright sunshine on a beach,
with the sun reflecting off the ocean and when its batteries may be running dovm is hardly a recipe for
a relaxed and carefree vacation. Basically, there is nothing as easy to use as the common paperback
novel and it will take a lot to replace it!
There may still be something to be said for e-books when it comes to weighty academic volumes that
students need to refer to for their studies. Instead of having to carry them around or physically go to a
library to read them, they would be able to download the sections they needed and use them on-
screen or print them out.
Changes will probably take place anyway in conventional publishing, with the incorporation of digital
printing techniques. It may soon be possible to have a particular book printed on demand over the
Net, using digital copies of the work provided by the publisher. Perhaps in this way we will eventually
be able to reduce our consumption of paper.
Question 41: What disadvantage of the e-book does the author point out ?
A. It has to have a power supply B. It can't be taken onto the beach
C. It's rather heavy to hold D. It's not relaxed
Question 42: In what way does the author suggest that the amount of paper we use might be
reduced?
A. by only having a book printed when someone requests it
B. by involving publishers in the printing of books
C. by creating a paperless office
D. by selling books over the Internet
Question 43: The word "rashly" in the passage is closest in meaning to.......
A. increasingly B. surely C. uncertainly D. quickly
Question 44: According to the passage, e-books.......
A. will be used more for leisure reading B. are unlikely to be used by anyone
C. are not at all a good idea D. may be useful when reading for reference
Question 45: What argument did the computer makers use to support the widespread use of
computers?
A. Offices would find their work cut by 10%
B. Their use would have significant ecological value
88
C. Conventional books would gradually die out
D. Trees would no longer have to be cut down to make paper
Question 46: The word "they" in the passage refers to.......
A. e-books B. volumes C. students D. studies
Question 47: In the passage, the author says that.....
A. laptops are a convenient way to read a novel
B. conventional books will soon become a thing of the past
C. readers will easily get used to e-books
D. the general reader is reluctant to give up using conventional books

Mark the letter A, B, C, D on your answer sheet to indicate the sentence that is closest in
meaning to each of the following questions.
Question 48: It’s not worth trying to make her change her mind.
A. There’s no point in trying to make her change her mind.
B. It’s no point in trying to make her change her mind.
C. There’s no point to try to make her change her mind.
D. It’s no point for trying to make her change her mind.
Question 49: We were surprised because she suddenly came back.
A. She suddenly came back, which almost surprised us.
B. She suddenly came back, which was very surprismg.
C. That she suddenly came back made us surprised.
D. That she suddenly came back made us surprise.
Question 50: You must never take your helmet off while you are riding a motorcycle.
A. When you are riding a motorcycle, wearing a helmet is not a must.
B. Helmet must be worn at all times when you are riding a motorcycle.
C. If you are riding a motorcycle, you needn't wear a helmet.
D. You needn't wear a helmet whenever you are riding a motorcycle.

GOOD
LUCK ON
YOUR
EXAM

89

You might also like